Clin Pharm: Intro, Rheum, EENT, GI, ID, Behavioral Health, Endo, Derm, OB/GYN, Neuro, Fluids/Electrolytes, Pulm, Peds, Cardio/Hemo, Pain Manangment

Réussis tes devoirs et examens dès maintenant avec Quizwiz!

Good Prescribing Practice: Indication

Encouraged, seldom practiced Helps confirm appropriateness of medication Reminds pt of drug's purpose Facilitates communication between health care providers

A 22-year-old female presents to your office with a history of a rash for one week. The rash is annular in appearance, with a pale center and is slightly erythematous. There is no history of allergies and no illness in family members. Vaccinations are up to date; the family recently returned from a vacation in the northeast. Physical examination is unremarkable except for the rash. Labs have been ordered. No meds. NKDA.

Erythema migrans= early lyme disease- stage 1 Doxycycline 100 mg po BID x 21 days; kids less than 8, pregnancy use amoxicillin. If PCN allergic use, macrolide. Some literature suggests 14 days is enough, but controversial.

What are the exceptions to "acute pain"?

Exceptions include: cancer, palliative care, terminal conditions, or traumatic injury with an Injury Severity Score of 9 or greater. There are no prescribing limitations for these conditions. For the treatment of pain other than acute pain, a prescriber must indicate "NONACUTE PAIN" on a prescription for an opioid drug listed as a Schedule II controlled substance.

CC: " I have stomach pain and cramping" 30-year-old female presents to your office with epigastric pain and tenderness. Notes that pain started after purchasing food from a local food-truck 2 weeks ago. Notes that she initially had some vomiting but that has now improved. Denies any n/v/d. Denies any constipation. Patient notes epigastric tenderness that is constant cramping pain (4/10). Worsens with eating/drinking (7/10). Admits to early satiety. Denies any melena or hematochezia. Pt has been taking famotidine (Pepcid) and Tums with minimal relief. ■PMHx: none ■MEDS: Famotidine 10 mg as needed ■ALLERGIES: None ■PE (Pertinent positives): +epigastric pain with light palpation, +guarding. No rebound, +hyperactive bowel sounds ■LABS: H pylori stool positive What would you give with Clarithromycin & Amoxicillin ?

H.Pylori

CC: "My husband has been confused and disoriented for the past 2 days" HPI: 70-year-old male with a history of cirrhosis is brought to the office by his wife. She notes that he is disoriented and confused for the last 2 days. Patient has been sleeping more throughout the day and will fall asleep mid-conversation. Recently released from the hospital where he was admitted for LE edema, worsening renal function and difficulty breathing. While in the hospital he had 3L fluid removed from his abdomen due to ascites. Was discharged to follow up with GI but that isn't until next week. Wife wants to know what can be done. ■PMHx: Cirrhosis 1 yr, CKD Stage IV, Osteomyelitis ■MEDS: Doxycycline, Torsemide, HCTZ, Oxycodone, Lactulose ■ALLERGIES - None ■VITALS - BP 120/68mmHg LA sitting, RR 14bpm, HR 68bpm, Temp 98.8F Oral ■LABS: Hgb 10.4 g/dL, Ammonia 176 mcg/dL What med would you give besides Rifaximin?

Hepatic encephalopathy

CC: "Rash on my ear that hurts" HPI: 50 y/o M presents with painful rash on his right ear x 1 week and radiates to his face. States he has had a burning pain for the last 10 days, but the rash showed up within the last 3-5 days. Pt. denies any new soaps, washes, or exposure to any new detergents. Rash has been spreading along ear. Pt. states the pain is a constant burning sensation (10/10). Touching the area makes it worse. Has tried OTC ibuprofen without any relief. Pt. denies any recent travel or fever. PMH: Hyperlipidemia, GERD, HTN Meds: Amlodipine 5mg PO daily, Protonix 40mg PO daily, Crestor 10 mg PO daily Surg Hx: Right Rotator cuff repair Soc Hx: Denies smoking, drinks 2 beers daily, no illicit drugs. Bus driver occupation. Vitals BP 140/94mmHg HR 99bpm RR 17bpm O2sat 98%RA Temp 98.6F Oral What other meds besides Valacyclovir?

Herpes Zoster Oticus Contraindications: None Adverse Rxns: N/V/D? Take with food. Counsel on steroids causing jittery-ness Safety/Monitoring: None

CC - "It hurts when I swallow x 2 days" A 6 yo Asian female is accompanied by her mother today for pain with swallowing x 2 days. Mom notes that patient has had fever of up to 103F Tympanic. Mom has given ibuprofen and Tylenol with improvement of fever. Mom notes that recently was here 1 week ago with older brother and he was positive for a throat infection. Wants to get patient checked prior to sending her back to school. ◦Allergies - NKDA ◦Medications - None ◦Vitals: Ht 44" Wt 45lbs Temp 99F Tympanic (Tylenol 1 hour ago) All other vitals normal ◦PHYSICAL EXAM ◦HEENT: H: normocephalic, E: Sclera/conjunctiva normal, non-injected E: Right/Left TM Pearly grey no bulging or effusion, N - Rhinorrhea noted T - +posterior oropharynx injected +tonsillar exudate b/l +ant cervical lymphadenopathy ◦CARDIO - S1S2 noted, no murmurs, rubs or gallops ◦RESPIRATORY - CTA B/L Ok, but what if pt has an anaphylactic allergic reaction to PCN?

Strep Azithromycin 12mg/kg every 24 hours x 5 days (MAX - 500mg/dosage) ◦45lbs ◦45lbs/2.2kg = 20.45kg ◦12mg*20.45kg = 245.45mg/day ◦200mg/5mL = 6.1mL ◦Pt to take 6mL daily x 5 days ◦6mL daily x 5 = 30mL

CC: " My throat hurts" HPI: 32 y/o F presents with sore throat x 3 days. Pt. states she has difficulty swallowing due to pain and has a strong odor coming from her mouth. Pain is sharp and constant 7/10 and worsens with swallowing (10/10). Pt is 26 weeks gestation and called her OB/GYN but was sent to the urgent care. Pt. complains of fever but has not taken her temperature. Pt. states she is using OTC Tylenol with no relief. PMH: Stress/Anxiety Surg Hx: Appendectomy (child 12 y/o) Social Hx: No tobacco/alcohol/illicit drug usage Meds: Prenatal vitamins, OTC Tylenol 500mg Allergies: NKDA Vitals - BP 132/82mmHg HR 98bpm RR 18bpm O2sat 99% RA Temp 100.2F Oral Tests Ordered: Rapid Strep with culture Rapid Strep - positive Culture - pending

Strep pharyngitis Contraindications: Pt is pregnant Adverse Rxns: N/V/D? Take with food Safety/Monitoring: Pregnancy Cat B (safe for baby!) Other Considerations: - Free at Publix - no need to worry about cost - Safe for baby - Able to be crushed & mixed with food (in case pt has nausea/difficulty eating due to pregnancy) 2nd line w/ allergics- Cephalsporins: Cephalexin

A 65 yo female with history of DM and HTN develops acute onset left sided facial droop, left arm and leg weakness. 911 was called and arrives within 15 minutes. EMS notes BP 200/110 What is your most likely diagnosis?

Stroke (CVA) What interventions should be provided in the field? - ASA -Antihypertensive? Clonidine? -Should 911 be concerned about closet hospital? -Monitor vitals - blood sugars, BP, O2 levels, reassurance -TIME ONSET - needs to be documented -PE findings DOCUMENTED (Facial droop, drift) Pt arrives in the ED with unchanged blood pressure, unchanged neurologic exam What key components of the hx should be obtained? -Time of onset -Head Trauma -PMHx - stroke? Aneurysm? AVM? -Any anticoagulants within 14 days? -Seizures? Pharmacologic interventions? Statin

A 65 yo female with history of DM and HTN develops acute onset left sided facial droop, left arm and leg weakness. 911 was called and arrives within 15 minutes. EMS notes BP 200/110 Patient has a NIHSS score of 8. ECG shows NSR Glucose 140, Platelets 200K PT/PTT are normal Head CT is read as 'normal' What is your next therapeutic step?

Stroke (CVA) t-PA How is t-PA administered? -0.9mg/kg in a 1:1 dilution MAX 90mg 10% initial bolus oer 1-2 min and the rest over 60minutes -Monitor BP -DO NOT give ASA or HEPARIN!!!!! How is suspected intracranial hemorrhage managed? -Discontinue t-PA -Obtain IMMEDIATE CT -Check labs (PT/PTT, platelet count, fibrinogen level) -Prepare fibrinogen, cryoprecipitate and platelets Call Neurosurgeon ON-CALL The patient received t-PA and within one hour her strength was markedly improved. She was admitted to the stroke unit where she was monitored and began early rehabilitation She was discharged home one week later with minimal left sided weakness She was told to follow up with PCP for BP monitoring/Treatment Back working in the primary sector, what do I do with my patients? w/in 4.5 hrs of last known normal per AHA/ASA

Pathophysiology of Pain

Nociceptive 2 subtypes at the tissue level •Visceral (organs) •Somatic (skin, muscles, joints, bones and connective tissue) Neuropathic •Injury to the somatosensory nervous system •Phantom limb pain, peripheral neuropathy, post herpetic neuralgia

CC "My right ear hurts!" HPI: 40 y/o F presents to your office with right ear pain x 2 days. Pt. states she is a flight attendant and has been experiencing this sharp ear pain (8/10) for the last 2 days. She states the pressure makes it worse (10/10). She admits to "feeling hot" but denies checking her temperature. She denies any ear discharge. Ibuprofen helps the pain briefly but only for a short time ~2 hours. PMH: Uterine Fibroids Surg Hx: C-section (10 years ago) Soc Hx: Smokes ½ PPD x10 years Meds: Zolpidem 5mg PRN Allergies: PCN (Hives) Vitals BP 118/88mmHg HR 84bpm RR 16bpm 02sat 98% RA Temp 99.2 F Oral How would you tx?

OM

CC: "I have excruciating pain in my left ear." HPI: 17 y/o F was swimming at the pool and the beach all last week while on vacation in the Florida keys. She states since last night she has severe ear pain 10/10 and she can't even put her head on the pillow because it hurts so bad. She states she "feels warm" but has not checked for a fever. She also feels some "gooey" stuff is coming from her ear. She has taken Goody's powder with no relief. PMH: Asthma Surg Hx: None Meds: Albuterol 90mcg/spray MDI PRN Allergies: Azithromycin (Hives) Soc Hx: Denies alcohol, drugs, or smoking Vitals BP 112/78mmHg HR 72bpm RR16bpm O2sat 99% RA Temp 98.9F Oral Positive Tragal and Helix tenderness. Otoscopic exam limited - unable to assess TM Ofloxacin Optic?

Otitis Externa Optic can go in the ears, but otic can NEVER go in the eyes!!!! Contraindications: none Adverse Reactions: skin irritation - return to office Safety/Monitoring: none

CC: "I have excruciating pain in my left ear." HPI: 17 y/o F was swimming at the pool and the beach all last week while on vacation in the Florida keys. She states since last night she has severe ear pain 10/10 and she can't even put her head on the pillow because it hurts so bad. She states she "feels warm" but has not checked for a fever. She also feels some "gooey" stuff is coming from her ear. She has taken Goody's powder with no relief. PMH: Asthma Surg Hx: None Meds: Albuterol 90mcg/spray MDI PRN Allergies: Azithromycin (Hives) Soc Hx: Denies alcohol, drugs, or smoking Vitals BP 112/78mmHg HR 72bpm RR16bpm O2sat 99% RA Temp 98.9F Oral PE: Positive Tragal and Helix tenderness. Otoscopic exam limited - unable to assess TM What would you prescribe?

Otitis Externa Optic can go in the ears, but otic can NEVER go in the eyes!!!! Contraindications: none Adverse Rxns: Skin irritation - return to office Safety/Monitoring: none

CC: "I have excruciating pain in my left ear." HPI: 17 y/o F was swimming at the pool and the beach all last week while on vacation in the Florida keys. She states since last night she has severe ear pain 10/10 and she can't even put her head on the pillow because it hurts so bad. She states she "feels warm" but has not checked for a fever. She also feels some "gooey" stuff is coming from her ear. She has taken Goody's powder with no relief. PMH: Asthma Surg Hx: None Meds: Albuterol 90mcg/spray MDI PRN Allergies: Azithromycin (Hives) Soc Hx: Denies alcohol, drugs, or smoking Vitals BP 112/78mmHg HR 72bpm RR16bpm O2sat 99% RA Temp 98.9F Oral Positive Tragal and Helix tenderness. Otoscopic exam limited - unable to assess TM Medications?

Otitis Externa Consider cost and look how 2 letters can change the cost of the Rx Use Cortisporin and Ciprodex with caution with ?ruptured TM Due to limited exam and cost Ofloxacin written

CC "I think my kid has worms" x 1 week 3 yo Hispanic male brought into your office for possible worms. Mom notes that patient goes to daycare but has been complaining of pain in his rectum at night. Mom states child is constantly "picking" at the area during the day but has been awaking at night due to discomfort. Denies any N/V/D, loss of appetite. Vitals - all within normal What do you suspect is the dx?

Pinworms What is the special test that can be done? ◦ Scotch tape test

Prescribing Errors: Poor handwriting

Poor handwriting contributed to a medication dispensing error that resulted in a patient with depression receiving the antianxiety agent Buspar instead of Prozac

A 75 yo Caucasian woman arrives in the ED after a fall down the front steps of her building. She c/o intense pain in her L hip. When she tried to stand, she was unable to put weight on her L leg. The pt had been previously healthy & had no significant PMHx. Lab tests were performed including TSH, urine studies, blood chemistries, CBC, and CMP. These were all within normal limits. A hip radiograph was performed and a fx of the femoral neck was clearly visible. Additional tests were performed including a bone density test. The pt was found to have a T-score of -2.9 at her spine and T score of -2.4 at her hip. The patient has an ORIF repair of her left hip and is discharged to a rehab center.

Post-menopausal osteoporosis Any bisphosphonate- 1st line tx ( alendronate, risedronate, ibandronate) This drug will be lifelong, won't recheck for at least a year minimum She'll have to come back after 6 mos for her well check

Good Prescribing Practice: Quantity of the drug

Prescribe only necessary quantity •May want to prescribe 1-2 ml extra for children's suspensions to allow for some spillage Write for specific quantities rather than time period (for example: dispense #30 vs. dispense for 1 month) Calculate: quantity = frequency per day x treatment days Writing out "Dispense # X" is helpful

PA Prescribing Laws in Fl with prescribing license

If a PA has a prescribing license, that PA may dispense drugs and sample drugs if their supervising physician has delegated prescribing authority to the PA. Dispensing medication is not the same as prescribing medication •Prescribing implies that the patient was given a written prescription by a practitioner to have filled at a pharmacy, or a prescription was called in to a pharmacy under a supervising physician's name. •Dispensing means the patient was given the medication in the office for fee, or at no charge as with dispensing sample medication. Prescribing of medication must be noted in the medical record Written prescriptions by PAs must be filled in a pharmacy licensed in the state of Florida •Out of state pharmacies that ship or mail prescriptions to residents of Florida must be licensed by the state as a nonresident pharmacy..... •So yes, PAs can write prescriptions to be mailed out of state if the pharmacy where it will be filled is licensed in Florida. PAs with prescribing privileges can prescribe any medication used within the scope of their supervising physician(s) practice except for those medications specifically excluded by law

What Fluids are Available? (crystalloids, electrolyte/salt based)

Routine fluids Sodium Chloride •0.9% - 154 mEq Na/L •0.45% - 77 mEq Na/L •0.22% - 38.5 mEq Na/L Dextrose - 5%, 10% •Considered free water due to rapid uptake/metabolism of dextrose Lactated ringers (273 mOsm/L) •Sodium - 130 mEq/L •Potassium - 4 mEq/L •Calcium - 2.7 mEq/L •Chloride - 109 mEq/L •Lactate - 28 mEq/L LR mostly in the OR -Mimics blood electrolytes and natural blood Normal lactate in the blood: 0 - Gets metabolized into bicarb to help with acidosis Other Fluids (colloids): More protein based to help with oncotic pressure Albumin •5% - used for volume replacement •25% - used to replace protein deficits Starches •Hetastarch •Dextran

Goals of Fluids

Routine goals •Replace and/or maintain fluid balance •Replace and/or maintain electrolyte balance •Provide calories to prevent catabolism (w/ dextrose) Other potential goals •Correct or maintain serum pH (w/ NaHCO3) •Provide oncotic pressure (w/ serum proteins) •Prevent precipitation of metabolic byproducts/drugs in renal tubule: E.g., rhabdomyolysis, methotrexate Notes: 4 calories for every 1 gram of dextrose 0 calories for sodium chloride

Correcting Fluid/Electrolyte Problems

If it occurred quickly (< 48 hrs) and the pt is symptomatic •Fix it quickly - little adaptation has occurred •Altered mental status w/ hyponatremia à 3% saline If it occurred slowly (>48 hrs) and/or pt is asymptomatic •Take your time - adaptation has likely occurred •Chronic hyponatremia --> over correction with 3% saline --> central pontine myelinolysis

Sliding scales - let's break it down...

If normal reactivity to insulin - ◦1 unit taken for every 50 points over goal If my goal is --> 120-150 for blood sugar reading... ◦150-200 --> 1 unit ◦200-250 --> 2 units ◦250-300 --> 3 units...and so on If Insulin resistant OR high dose basal insulinà ◦1 unit for every 25 points over goal If my goal is --> 120-150 for blood sugar reading... ◦150-175 --> 1 unit ◦175-200 --> 2 units ◦200-225 --> 3 units ... and so on You normally stop at >400 (call the office) or 5-6 units

When prescribing for pain use a stepwise approach

If pt has been prescribed non-opioid medication and still has the pain without much improvement Add an opiate If opiate doesn't provide realistic improvement - increase dosage/send to pain mgmt./complete tests to look for other underlying issues

CC "my child has a rash x 1 week" A father brings in his 5-year-old African American son for a rash on his face for the last week. States that his son has been picking at the area for the last week and it seems to be spreading. Patient denies any pain or itching, but dad notes that patient will pick at the area. Denies any new lotions, soaps or detergents. Denies any recent contacts with the same rash. No new pets in the home. ◦Allergies: NKDA ◦Medications: None ◦Vitals: HT 43" WT 40lbs HR 84 bpm RR - 20bpm BP 100/60mmHg LA sitting ◦PHYSICAL EXAM: ◦Skin - golden crusts noted below nostrils R>L and along top lip. No vesicles noted. No hives, excoriations or eczematous lesions noted Dx? Tx?

Impetigo What do you want to write? ◦Bactroban (Mupirocin) ointment

Prescribing Laws in Florida

In the US, prescribing power is governed by state legislation The state regulates the prescriptive powers of the following groups: •PAs, NPs, optometrists, pharmacists

Pt returns to the office for bp check. He is still having elevated bp. No sxs otherwise and feels well overall BP in office 156/92 mmHg RA sitting Lab results returned and Kidney function is within normal. What's next?

Increase HCTZ to 25mg Write new Rx Reiterate to patient about DASH diet and dietary measures to aid with BP

Same pt returns 3 wks later and states he has made dietary changes and has been trying to exercise. Admits to taking medication daily and denies missing any dosages. Has not been monitoring bp at home. Denies any chest pain, shortness of breath, pedal edema, palpitations, dizziness, headaches Labs have also returned, and you want to discuss them with the patient VITALS: BP 152/96 LA sitting, 156/94 RA sitting HR 86bpm, RR 14bpm Abnormal lab results Total Cholesterol 262mg/dL LDL 168mg/dL HDL 28mg/dL TRIGS 278mg/dL What do you do?

Increase the amlodipine to 10mg daily and have patient return in 2 weeks Add a statin - but beware which one! - Amlodipine has interactions with simvastatin(Zocor) and high dosage atorvastatin (Lipitor)

Fluid Balance

Intake should match output (generally) Average healthy adult: 30-40 mL/kg/day Note: Why when you admit pt monitor Inputs & outputs (I's & O's) Reasons for fluid gains (overhydration)? •CHF, high water intake, Liver disease, IV overdose on fluids, retention of fluid Reasons for fluid losses (dehydration)? •DI, surgery, hemorrhaging, excess, dialysis, N/V/D, burns, heat stroke, diuretics What occurs with serum sodium and plasma osmolality in each case? •Dehydration: Na+ decreases = hyponatremia à plasma osmality decreases as well What hormone responds directly to this change? •ADH

What if I can't get an IV?

Intraosseous access - good for emergencies (esp in peds) Hypodermoclysis - non emergent cases •Facilitated by hyaluronidase (Hylenex) - 150 units SQ

A 47-year-old male comes to the office after returning from his cruise. Notes he has a history of gout and believes he is having a flareup due to his diet while aboard the ship. Notes that the pain has improved significantly since this weekend (8/10à4/10) but is still there. Is currently taking 4 OTC ibuprofen 200mg tablets every 8 hours. Wants to know what else he can do. PMHx: Gout VITALS: WNL

Is there a better option for a medication that can be written? Remember to treat the diagnosis -NSAIDs are not all equal. -When it comes to gout, indomethacin is a better NSAID option -Write him a new prescription -Counsel him on not taking indomethacin with ibuprofen -Counsel him on diet -Possibly add allopurinol for uric acid control

Body Fluids

Salty banana

PA Prescribing Laws in Florida

Know the law before you prescribe! •Simply stated, if a PA violates the statutes they may be investigated, and if found guilty of a violation, they may be fined, disciplined, or lose their license.... The Board of Medicine tends to frown upon PAs who carry around pre-signed prescriptions by their attendings and then fill in the medications It is a third-degree felony to call oneself a PA if one does not have a current valid license from the Florida Department of Health. The State of Florida takes this very seriously and will vigorously prosecute violators. The Board of Medicine issues each PA a license and a prescriber number after applying for the same, and the PA can then prescribe according to law. •If you do NOT have a prescribing number/license in hand from the Board, you CANNOT prescribe The PA must clearly identify to the patient that he/she is a PA and must inform the patient of his/her right to see the physician prior to a medication being prescribed or dispensed

Insulin

Know your insulin! Diabetes is everywhere and treated in every specialty Lifestyle changes are great, but it is a necessity to be able to walk with patients through insulin control. Explaining to them why it is important and allowing them to understand why it is necessary will increase compliance with the medication and willingness to make those lifestyle changes.

Sarah, a 45 y/o Caucasian female, presents requesting to have her blood sugar tested - "it must be running low because I don't feel well." PMHx: T2DM x 6 months, HTN x 12 yrs, dyslipidemia x 12 yrs, morbid obesity x 20 yrs Meds: Metformin 1000 mg po BID with food, Lisinopril 20 mg po once a day, pravastatin 40 mg po once a day All: Penicillin - hives SHx: married, 2 teenagers, sales associate, quit smoking 10 yrs ago, moderate alcohol - 3 to 4 beer or wine a week ROS: Daily nocturia, polyuria, polydipsia. No nausea, vomiting or diarrhea, no signs or symptoms of hypoglycemia, no paresthesia's, weakness or tingling. No chest pain or dyspnea. She admits to a 10 lb weight gain over the past 6 months. PE: NAD, alert & oriented x 3, marked obesity, VSS BP 154/90mmHg LA sitting, P 90bpm, R 18bpm, T 98.0F Oral, Wt 265 lbs, BMI 36kg/m2 HEENT - PERRLA, EOMI, no retinal hemorrhages or exudates, nose and throat clear without exudates or lesions Neck - supple, no lymphadenopathy, no thyromegaly Lungs - CTA bilat CV - RRR, no murmurs, gallops or rubs ABD - soft, obese, nontender, normal BS EXT - pulses 2+ throughout, no edema Neuro - CN II-XII intact, DTR's 2+ throughout, feet with normal vibratory and light touch sensation (monofilament normal) Labs: Na - 138, K - 3.7, Cl - 103, CO2 - 31 BUN - 16, Creat - 0.9, CA - 9.4, Phos - 3.3 AST - 16, ALT - 19, Alk Phos - 62, T. Bili - 0.4 Random glucose - 243 (high) HgA1C - 10.0% (high); Previous HgA1C - 8.9 (6 months ago) Fasting lipids - Chol - 244 (high) LDL - 141 (high) HDL - 58 Trig - 225 (high) UA - 2+ glucose, neg ketones, 1+ protein What insulin?

Lantus? Why? Why 24 units? 24 units is based upon the calculation of 0.2-0.4U per kg -Pt is 265lbs -265lb/2.2 = 120.45kg -120.45kilograms*0.2units = 24Units Lantus chosen due to cost - without insurance Lantus is $203 with coupons at pharmacies. The other choices of toujeo and Levemir are >$400.

CC - "I need to have her checked for lice" 5 yo Asian female seen in office today. Mom notes that nurse called from school for patient to be picked up due to "possible lice". Patient admits to some itching of her scalp, but mom notes that she needs to have her checked to return to school. Pt recently had a sleepover with her friends but mom hasn't heard if any of the other children have been diagnosed with lice. •PMHx: None •FamHx: Mom 34 (no history), Dad 37 (neg history), MGM 57 (HTN), PGM 62 (DM2) •Medications: None Allergies: None Counseling? Tx?

Lice Discuss about combing out hair & continuing to look for nits •Wash all beeding & vacuum at hoom •PT is able to return to school after tx (note written for rx) Permethrin 1% lotion Apply to scalp & let sit for 10 minutes then rinse. May repeat in 9-10 days # 1 bottle (one) Refills: 0 (zero)

CC "I have a cough" A 55-year-old male presents to your ER with fever, chest pain, and cough. 5 days ago he had a "cold" with cough and congestion and over the last 2 days has been getting worse. Notes at home his temp ranges from 101-103F. He takes Tylenol and ibuprofen with relief of his fever. Admits to chills, fatigue, congestion, cough with phlegm (rust colored). Notes sharp stabbing pain (8/10) in his right lower chest with deep breathing. Coughing makes the pain worse (10/10) and sitting still improves the pain (3/10). Denies Tylenol and ibuprofen aiding with pain. ■PMHx - none ■Social Hx - 30 pack year smoking hx. No alcohol or illicit drug usage ■Vitals - BP 110/60mmHg RA sitting, HR of 110, RR of 28, Temp 101.9F Oral, O2 sat 88% RA ■Pulmonary exam is significant for crackles and rhonchi over the right lower lung fields. Cardiovascular exam is significant for tachycardia but with regular rhythm and no murmurs, gallops, or rubs. The remainder of his exam is normal. ■You order a gram stain with sputum culture, CBC, EKG, and CXR. ■The CBC shows a markedly elevated WBC . ■The EKG shows a sinus tachycardia at a rate of 110. ■The CXR shows a dense infiltrate of the right lower lobe with a small pleural effusion.

List at least 4 possible causative agents of this patient's pneumonia. - Strep pneumoniae, Haemophilus influenza, Legionella spp, Chlamydia pneumoniae, Klebsiella pneumoniae, Staph aureus, Mycoplasma pneumoniae, anaerobes Of those, which is most likely, and what would you expect to see on gram stain? - Strep pneumoniae, gram positive diplococci How would you treat this patient? - Antibiotics IV(Ampicillin 2gm IV every 6 hours) and O2 while in the hospital and discharge home on oral antibiotics when stable Because, strep pneumonia is usually pcn susceptible. Discharge home with high dosage amox and follow up with PCP in 1 week. Will need to continue treatment until fever free - minimum 5 days If concerned of PCN resistance - azithromycin or other fluoroquinolones would be better options.

Good Prescribing Practice: Selecting the drug

Medication allergies Availability Available forms Cost Patient compliance Patient's desire (or your desire) for brand name instead of generic (DAW- dispense as written)

CC: " My stomach hurts" HPI - 62-year-old man present to the office with c/o stomach pain for the last 3 weeks and dark black foul-smelling stools x 2 days. Pain is a gnawing epigastric pain. Pain worsens with all food (8/10) and improves with not eating (2/10). Has not changed his diet within the last few months. Denies symptoms starting with any potluck or eating out. Denies any nausea, vomiting, diarrhea or constipation. Denies any history of taking pepto-bismol. Pt is compliant with daily metformin and ASA. Notes hx of osteoarthritis and states that over the last 2 months has had to take his ibuprofen twice daily everyday and sometimes will take a 3rd dosage. Pt doesn't always take ibuprofen with food. Admits to fatigue that has worsened over the last 2 weeks. PMHx: Type 2 DM-10 yrs, Osteoarthritis-15 yrs Meds: Metformin 500 mg twice daily, EC aspirin 325 mg daily - compliant - Ibuprofen 600 mg twice daily as needed Vitals: BP 110/72mmHg RA sitting, HR 106bpm, RR 16bpm, Temp 98.8F Oral Labs: Hgb 9.2 mg/dL with microcytic anemia PE(pertinent positives): pallor, tachycardia

NSAID induced gastric ulcer? Or PUD Select appropriate medication - PPI chosen - omeprazole written Select non-pharmacologic therapy/interventions. - Pt to decrease the amount of ibuprofen he is taking. Ideally, pt will stop the ibuprofen, but if he is needing to continue for pain mgmt. of arthritis, make sure he is taking it with food and even possibly discuss with patient about changing ibuprofen to a different medication. - Also, if taking ASA - determine risk vs benefits if he should continue with ASA and then again readdress arthritis pain control with patient in regard to NSAID -Refer to GI for endoscopy to assess PUD and complete H pylori test. What are the contraindications or side effects of this medication? - Monitor for long term usage of PPI Describe the cost of the medication. -Omeprazole is affordable and covered by most insurances

Body Fluids: Osmotic Pressure

Na+ responsible for extracellular space (275-295 mOsm/kg) K+ responsible for intracellular space Normal lab values: Na: 135-145 K: 3.5-5.5 Hypo - dehydration Hyper - cerebral edema

PA Practice Laws FL

New graduates are eligible for a temporary license to practice until they sit for the next available NCCPA examination. New grads must report their passing score to the BOM, who will then issue a regular license The temporary license shall expire 30 days after receipt of the NCCPA examination scores. An applicant failing the proficiency examination is no longer temporarily licensed but may reapply for a 1-year extension of temporary licensure. If the PA fails the second attempt, they cannot apply for a license until they have successfully passed the exam and they can no longer practice with a temporary license. The NCCPA allows six attempts to pass the PANCE examination. Must wait 90 days between attempts only allowed three attempts in one year.

CC - "my eyes are itching and red x 1 month" Mother brings in her 7-year-old Asian son due to itchy eyes and cough. States that over the last few years he has been complaining of itching eyes as well as runny notes that worsens in the spring. Mom given the patient Benadryl if the symptoms are severe with relief. Patient has not been on a daily medication. Family history of allergies and asthma. Patient denies any personal history of allergies or asthma. Mom states that she tries to keep house clean and doesn't allow the pets in patient's bedroom ◦Allergies: NKDA ◦Medications: Children's Benadryl OTC as needed for symptoms ◦PHYSICAL EXAM: ◦HEENT: H: normocephalic, E: Conjunctiva injected b/l with cobblestone like changes noted of interior of lower lid b/l. E: Right/Left TM Pearly grey no bulging or effusion, N - Rhinorrhea +pale blue boggy turbinates noted b/l T - +postnasal drip noted. No tonsillar exudate or lymphadenopathy noted ◦CARDIO - S1S2 noted, no murmurs, rubs or gallops ◦RESPIRATORY - CTA B/L Tx besides Loratadine?

Seasonal Allergies

Hazel C., a 30-year-old female, demonstrates a subtle onset of the following sxs over the last 4 months: ◦Dull facial expression with droopy eyelids ◦Puffiness of the face and periorbital swelling ◦Sparse, dry hair with dry, scaly skin ◦Evidence of intellectual impairment ◦Lethargy, constipation, and hypothermia ◦Change of personality ◦Bradycardia (55 b/min) & blood pressure of 90/70 What lab studies would you like to order?

TSH, Free T4, Total T3 All results normal except ◦T3 - 20 (100-200 ng/dL) - low ◦T4 - 2.8 (4.5-11.2 mcg/dL) - low ◦TSH - 36.5 (0.4-4.0 mIU/L) - high What's the dx? Hypothyroidism Levothyroxine Contraindications: - Hypersensitivity to thyroid hormone or other ingredients - Acute MI, thyrotoxicosis, untx'd adrenal insufficiency - Tx of obesity or infertility Adverse effects: Many - most important: ◦Cardiac - CHF, Acute MI, palpitations, arrhythmias, angina ◦Decreased bone density ◦Weight loss, insomnia, anxiety, flushing, headache

Types of Insulin**

The previous case Basal (long acting) insulins 1.Insulin glargine (Lantus, Trujeo) 2.Insulin detemir (Levemir) 3.Insulin degludec (Tresiba) - up to 40 hrs Should I add a prandial insulin? Well, you can depending on conversation with the patient. Prandial (rapid acting) insulin 1.Insulin lispro (Humalog) 2.Insulin aspart (NovoLog) 3.Insulin glulisine (Apidra)

Steroids: Strong classes (1-4)

Thick skin (palms/soles) Dx: •Lichen Planus, Psoriasis, discoid lupus, chapped feet, lichen simplex chronicus, severe poison ivy, alopecia areata, nummular eczema, and severe atopic dermatitis in adults Limited skin areas to minimize systemic side effects

Steroids: Weak classes (4-7)

Thin skin and occluded areas (eyelids, facial skin, body folds, axillae, groin, genitals, and perineal region) Dx (moderate steroids) •atopic dermatitis, nummular eczema, asteatotic dermatitis, lichen sclerosis, atrophicus of the vulva, scabies (after scabiecide), severe dermatitis Weaker steroid with milder skin irritation

When should opiates be given?

Think about some situations when opiates are appropriate Acute vs Chronic Acute: Fractures, Surgery, Trauma Chronic: Cancer, Trauma Does pt need pain management? Does your practice treat chronic pain?

Great, I'm taking my sliding scale BUT I'm still high after meals!!!

This is usually due to the insulin covering only for the blood sugar and not the food itself. We must then place a "base" of the rapid insulin and add to it... For example: Have the patient take 5 units with EVERY meal. That will cover the food and then whatever the blood sugar is ADD to it. So, my blood glucose is 235. Here is my scale ◦150-200 --> 1 unit ◦200-250 --> 2 units ◦250-300 --> 3 units I would take 5 units (base) + 2 units (scale) = 7 units with my meal

I'm already taking mealtime dosing and it's just not working...

This is when you divide 1600 by their total daily dose For example: Pt takes 80units of Long acting Pt takes 10 units with meals ◦80 units + 10 units*3 (meals) = 110 units daily ◦1600/110 = 14.5 ◦Therefore, pt needs to adjust for every 15 points over target goal BG goal 120-150 --> ◦150-165 --> 1 unit ◦165-180 --> 2 units ◦180-195 --> 3 units ***So then pt would take the adjustment and add to their base of 10 units at meals as we discussed in the previous slide***

What to understand when treating pain

Treat the pain, but also choose your treatment wisely. Don't automatically reach for opiates, but if they need it. Don't be scared to write for it!!! Consider what is causing pain and treat that as well Make patient aware that pain might not go down to a 0/10. The goal is to make their daily living activities more tolerable Does the patient need pain management?

CC: "I have vaginal discharge" x 1 week HPI: 22 y/o female presents with vaginal discharge x 1 week. She describes it as very "liquidy". No pain in abdomen or vagina. She admits to multiple sex partners, no protection. No f/n/v/d •Med Hx: MRSA skin infection X 1 year ago •Meds: none •Allergies: Ciprofloxacin (Rash) •Soc Hx: Smokes 1 pk/day, No alcohol or illicit drugs. VS BP 125/80 HR 64 RR 14 O2sat 96% RA Temp 97.8 Speculum exam reveals: picture below Bimanual No CMT Urine hCG: Negative, U/A: Neg Vag CX:Pending Wet Prep: no clue cells, no budding yeast, POS Trichomonads How do you treat?

Trichomoniasis •Flagyl (metronidazole) 2 g PO single dose •Flagyl (metronidazole) 250mg PO TID x 7 days •Flagyl (metronidazole) 375mg PO BID x 7 days •CDC- Flagyl (metronidazole) 500mg PO BID x 7 days Take with food to minimize upset stomach. •Can also Add Rocephin 250mg IM and 1 gram of Azithro to cover GC/chlamydia •Contraindications - disulfiram rxn

CC "How much Tylenol or Motrin can I give my child?" Mom brings her child with a fever and wants to know how much medication she can give? Dad brings child in for vaccines and want to know what he gave give for pain? Grandma brings child in for pain in the toe and wants to know what can be given? Let's talk Tylenol and Ibuprofen dosing especially in the littles First and Foremost à NO MOTRIN BEFORE 6 months of AGE!!!!!

Tylenol 160mg/5mL OTC infant liquid ◦10-15mg/kg/every 4-6 hours ◦Example: ◦15lb kid ◦15/2.2 = 6.81kg * 10-15 = 68mg-102mg every 4-6 hours Pt would get 2.5mL every 4-6hours 160/5 = 32*2.5 = 80mg per dosage Ibuprofen 50/1.25mL OTC infant liquid ◦5-10mg/kg ever 6-8 hours ◦Example: ◦15lb kid ◦15 lb kiddo = 6.81kg * 5-10mg ◦34.1mg - 34-68mg every 6-8hrs Pt would get 1.25mL every 6-8 hours(50mg)

Alfonso, a 68 y/o HM, presents complaining of "Blurry vision for the past few months that seems to be worsening." He also notes increased fatigue and a decreased energy level that has prevented him from working in the garden which he usually enjoys. PMHx: HTN x 18 yrs, dyslipidemia x 8 yrs, obesity x 25 yrs Meds: Lisinopril 20 mg po once a day Allergies: NKDA SHx: District Salesman, married x 40 yrs, 3 children, no smoking, 1 to 2 glasses of wine with lunch and dinner ROS: occasional polydipsia, polyphagia, fatigue, and weakness. Gets up once during the night to urinate Denies check pain, dyspnea, dizziness, numbness or tingling in extremities, weight changes, nausea, vomiting or diarrhea. No skin lesions or headaches. No other visual complaints. PE: NAD, alert & oriented x 3, central obesity, VSS B/P 124/76mmHg LA sitting, P 80bpm, R 18bpm, T 98.2F Oral, BMI 27.4 kg/m2 HEENT - PERRLA, EOMI's, no retinal hemorrhages or exudates, TM's pearly gray, nose and throat clear without exudates or lesions Neck - supple, no lymphadenopathy, no thyromegaly, no JVD Lungs - CTA bilat, CV - RRR, no murmurs, gallops or rubs Abd - soft, nontender, normal BS, no hepatosplenomegaly EXT - peripheral pulses 2+ throughout, no lesions, edema or lesions Neuro - CN II-XII intact, feet with normal vibratory and light touch sensation and monofilament-normal Na - 141, K - 4.0, Cl - 96, CO2 - 22 BUN - 20, Cr - 1.1, Ca - 9.9, Phos - 3.2 AST - 21, ALT - 15, Alk Phos - 45, T. bili - 0.9 Fasting glucose - 162 (high) HgA1C - 6.8% (high) Fasting lipids - Chol - 280 (high) LDL - 193 (high) HDL - 27 (low) Trig - 302 (high) UA - 1+ glucose, neg ketones, neg protein, neg microalbumin What med besides Metformin?

Type 2 Diabetes, Overweight, Hyperlipidemia w/ low HDL Why not Metformin XR? Why Crestor? Metformin without XR is free at Publix in Florida - Why NOT 90 tablets? I want the patient to return within 1 month so that we can discuss lifestyle changes, review bg log and discuss if changes are needed in dosing of medication Crestor is not very expensive $10 at Wal-Mart for the generic. It comes generic and is very effective controlling lipids -Has a better safety profile. It can be utilized in children as young as 7 for homozygous familial hyperlipidemia, whereas most other statins have an approval of age 10 and older. Pravastatin has an approval for >age 8 Why 90 tablets? - No need to recheck cholesterol for at least 6-8 wks - therefore, 90 tablets is more cost affordable and will increase compliance for the duration until retesting of labs can be completed

Fluid Boluses

Typical dose •Peds: 20 mL/kg •Adults: 1-2 L •Renal pts/edematous: 10 mL/kg or 500 mL Fluids used •NS or LR (in surgery usually): Repeated boluses of NS can lead to hyperchloremic acidosis •No benefit to using colloids Special cases •Sepsis/trauma - may need 60-80 mL/kg to adequately resuscitate Notes: Normal Cl in blood (110)

CC: "It burns when I urinate" X 4 days HPI: 35 y/o F presents complaining of burning with urination, increased frequency and urgency x 4 days. No abdominal pain, no vaginal discharge, no n/v/d. LMP: 4 weeks ago •PMH: Seasonal Allergies •Surg Hx: C section x 2 •Soc Hx: Denies any drugs, wine socially, no tobacco •Meds: LoLoestrin 1tab PO daily •Allergies: Keflex, PCN (Hives) •VS BP 138/76 HR 100 RR 18 O2sat 100% RA Temp 99.6 •All PE findings within normal limits •Pt declines pelvic exam •UA: 2+LE, 2+Nitrites, >50WBCs •Urine hCG: POS How do you treat?

UTI in Preg female MacroBID (nitrofurantoin) 1 tab PO BID x 5days •Brown urine •Monitor for extended use Monurol (fosfomycin) 1 single dose 300mg •UTD states not as effective as Bactrim •$$$$

CC: "I can't stand it anymore! I have blood in my poop and feel very weak" HPI: 26-year-old female comes into the office complaining of hematochezia and an increase in stool frequency. States over the last 2 months had noticed an increase in stools to 5-6x daily and has BPBPR +abdominal pain relieved with defecation. Dx with ulcerative colitis x 2 years ago. Was treated acutely and has been able to control it with probiotics x 2 years. Notes increase in fatigue and weakness since symptoms started. Denies any vomiting, constipation, nausea. ■PMHx: UC for 2 years; HTN for 2 years ■MEDS: Probiotics and HCTZ 25 mg daily, compliant ■LABS: Hgb 10.9 g/dL; ■PE (pertinent positives): orthostatic hypotension What would you give besides prednisone?

Ulcerative Colitis (UC)

A 32 y/o Asian F presents complaining of palpitations and dyspnea with exertion worsening for the past 2 weeks. Symptoms started about a month ago but were intermittent and now are occurring multiple times a day. No chest pain or pressure. Additionally, she reports a 14 lb weight loss over the past 2 months, feeling hot and sweaty all of the time, noticeable hair loss, and irritability. PMHx - no significant medical history Fam Hx - father has HTN, mother has thyroid problems Social Hx - lives with husband and 2 daughters (7 & 5 yrs old), no tobacco use, drinks alcohol occasionally (2-3 drinks a month) Meds - multivitamin Allergies - Sulfa meds (rash) ROS - No visual changes, occasional nausea and diarrhea. Gen: NAD, A&O x 3, slender build VS: BP-130/78mmHg LA sitting, P-120 to 160bpm irregular, RR-20bpm, T-100.6 F Oral, Wt-128 lbs, Ht-5'6" Skin: Hyperpigmented on upper back and lower extremities, warm and moist. Hair is fine and sparse in the frontal area. HEENT: PERRLA, EOMI, + Lid lag, mild proptosis, mild lid retraction NECK: supple, smooth symmetrically enlarged thyroid, + thyroid bruit, prominent pulsations in neck vessels Lungs: CTA bilat, no rhonchi, rales or wheezes CV: Irregularly irregular rhythm, + tachycardia, no murmurs or rubs Abd: soft, nontender, + bowel sounds, No HSM or masses, Aortic pulsations are palpable Rectal: guaiac negative Extremities: pulses 2+ throughout, no calf tenderness, no clubbing or cyanosis, cap refill <2 secs. Nails - see photo Neuro: fine tremor with outstretched hands, hyperreflexia at knees, no muscle weakness Goiter present Lower extremities has pretibial myxedema Nails are brittle Labs: see powerpoint ECG: in Afib

What is the assessment? Hyperthyroidism - Grave's disease New onset Atrial fibrillation What is the plan? Treatment: Radioactive Iodine - tx of choice for Grave's disease Methimazole (tapazole) - 5 to 120 mg a day Beta Blocker - ◦Metoprolol - 25 to 100 mg twice a day ◦Atenolol - 25 to 100 mg once a day What are the adverse effects? Contraindications? The absolute contraindication for radioiodine is pregnancy

A 16 y/o adolescent female is brought to the emergency room by her mother because the daughter seems confused and is behaving strangely. The mother reports the patient has always been healthy and has no significant medical history, but she has lost 10 lb recently without trying and has been complaining of fatigue for 2 or 3 weeks. The patient had attributed the fatigue to sleep disturbance, as recently she has been getting up several times at night to urinate This morning, the mother found the patient in her room, complaining of abdominal pain, and she had vomited. She appeared confused and did not know that today was a school day. The patient is slender, lying on a stretcher with eyes closed, but she is responsive to questions. She is afebrile, and has a HR- 118 bpm, B/P- 125/84 mm Hg with deep and rapid respirations at the rate of 24 breaths per minute. Upon standing, her heart rate rises to 145 bpm. and her blood pressure falls to 110/80 mm Hg. Her funduscopic examination is normal, her oral mucosa is dry, and her neck veins are flat. Her chest is clear to auscultation, and her heart is tachycardic with a regular rhythm and no murmur. Her abdomen is soft with active bowel sounds and mild diffuse tenderness, but no guarding or rebound. Her neurologic examination reveals no focal deficits. Lab studies include serum Na-131 mEq/L. K-5.3 mEq/L, Cl-95 mEq/L, HCO3- 9 mEq/L BUN-35 mg/dL creatinine-1.3 mg/dL Glucose-475 mg/dL Arterial blood gas reveals pH-7.12 with pCO2 24 mm Hg and p02-95 mm Hg Urine drug screen and urine pregnancy test are negative UA shows no hematuria or pyuria, but 3+ glucose and 3+ ketones.

What is the assessment? Pt is in DKA What is the plan? Pt needs to be admitted to the ICU Admit to the ICU Correction of fluid loss with intravenous fluids ◦isotonic sodium chloride solution or lactated Ringer solution - 1 to 3 liters in the first hour Correction of hyperglycemia with insulin ◦continuous IV insulin infusion using an infusion pump of rapid-acting insulin ◦optimal rate of glucose decline is 100 mg/dL/h. Do not allow the blood glucose level to fall below 200 mg/dL during the first 4-5 hours of tx Correction of electrolyte disturbances, particularly potassium loss ◦If the potassium level is greater than 6 mEq/L, do not administer potassium supplement. If the potassium level is 4.5-6 mEq/L, administer 10 mEq/h of potassium chloride. If the potassium level is 3-4.5 mEq/L, administer 20 mEq/h of potassium chloride. Correction of acid-base balance - bicarbonate may be considered Treatment of concurrent infection, if present

CC: "It is hard for me to swallow and my throat hurts." HPI: 34 y/o M present to the ER. Pt. has difficulty talking due to pain and voice is very low. He is drooling, has a muffled voice and states " My throat hurts and it's hard to swallow." He states pain is sharp (10/10) x 4 days and is taking Advil and Cepacol with no relief. Pt. states he is also feels "feverish" but has not taken his temperature. Pt. states he has been unable to eat or drink in the last day due to pain PMH: Herniated Disc Lumbar L5/S1 Surg Hx:None Meds: Advil OTC Allergies: PCN (Rash, Hives) Soc Hx: Smokes 1 PPD for 15 years, drinks occasionally beer, smokes marijuana at times. Vitals: BP 128/90mmHg HR 129bpm RR19bpm O2sat 97%RA Temp 102.8F Oral How to tx?

What is your assessment and plan? - Peritonsillar Abscess: I&D? Needle Aspirate? Remember we are in the hospital setting so how would you order meds for this pt? - IV antibiotics while in the ER - allergic to PCN Clindamycin 600mg every 6 hours IV Vancomycin 15-20mg/kg IV every 8-12 hours -If responds well to parenteral and stable - then release with written Rx and advise to follow up with PCP/ENT

CC: "I have no energy for 2 weeks." HPI: 32-year-old female presents to your office stating she has had no energy for the last 2 weeks. Notes fatigue is worsening, and she doesn't want to get out of bed. Has lost interest in what she used to enjoy. States she knew she had to be seen when she stopped wanting to go to her children's school activities. This all started after an argument with her husband over finances 3 weeks ago. Notes she is feeling guilty about not working and inability to contribute to the household's finances. Admits to history of depression but was never formally seen/diagnosed and has never taken medications for it. Admits to decreased appetite. Denies any issues sleeping. States will sleep 10+ hours at night and sleep on and off during the day. Denies any anxiety, suicidal or homicidal ideation. PMHx: none - healthy Surg Hx: Appendectomy (child 12 y/o) Meds: Multivitamins daily Allergies: NKDA Social Hx: Married for 8 years, 2 children (boy 6 y/o & girl 4 y/o); single family home, safe, no abuse, no tobacco, illegal drugs, started drinking one glass of wine nightly to relax VS BP 132/82mmHg RA sitting HR 98bpm RR 18bpm Temp 98.2F Oral PE: Pt crying on and off during exam ENT - Eyes, conjunctiva pink w/o lesions; Thyroid, non-enlarged, w/o nodules or tenderness CV - S1 & S2 RRR w/o murmurs, rubs, gallops RESP - CTA BL Labs: Electrolytes, CBC, TSH/FT4 - NL

What is your diagnosis? - MDD What? If any, medication would you start the patient on? - Zoloft (sertaline) 50 mg one tablet daily Why? - It is affordable and works well ($9/mo at Wal-Mart w/o coupon and $6.52 at Publix with a coupon). What would you take into consideration when writing medication? - Patient is depressed regarding finances. Cost of medication must be considered (Any other SSRIs, Tryptophan foods, MAOIs, age, drowsiness_ What else would you do for patient? - Referral to behavioral health counselor, psychiatrist or psychologist, Journaling, CBT, get BP under control, exercise

CC: "My stomach hurts" An 18-year-old female presents to your ER complaining of a 1-week history of abdominal pain. She tells you that she and her friends recently returned from spring break in Cozumel. Upon returning, she has had a constant ache (7/10) on her right side. Pt notes mild nausea, no diarrhea/vomiting/constipation. Denies any dysuria, urinary frequency or urgency. Notes she has some discharge, but not "too much" The patient's mother is worried because her daughter has been unable to eat for the past 2 days and she feels she may have become sick from "drinking the water" while on vacation. Mom notes she has given her daughter pepto-bismol with no relief. ■PMHx - None ■Soc Hx - Denies alcohol/tobacco/illicit drug usage. Denies being sexually active. Patient is a student ■Medicines - Daily vitamin ■Allergies - NKDA ■Vital Signs: BP 104/68mmHg RA sitting, RR 10bpm, Pulse 74bpm, Temp 98.8F Oral LMP 2 weeks ago Cardiovascular and pulmonary exams are normal. The abdominal exam is significant for a soft, non-distended, but diffusely tender over the suprapubic area with light and deep palpation. Pain is greater RLQ vs LLQ without guarding or rebound. Bowel sounds are normal x 4 quadrants After asking the mother to step out of the room while you examine the patient, you ask your patient about her sexual history. She tells you she has had 5 partners, and she was with the 5th while on vacation. She is not on any current contraception, occasionally will use condoms, if she can afford/acquire them and has never been pregnant. Admits to not using condoms while on vacation. Her LMP was 2 weeks ago and was heavier than normal. Pt also admits she has had a lot more vaginal discharge that has worsened since returning home. She is wearing a pantyliner and changing it twice daily due to the amount of discharge she is having Pelvic exam reveals a copious amount of foul-smelling discharge within the vault and covering the cervix. Cervix is erythematous. Bimanual exam is positive for cervical motion tenderness and right adnexal tenderness without masses

What is your diagnosis? - PID Which microorganisms are the most likely suspects? - Neisseria gonorrhea, Chlamydia trachomatis, enterics (gram negative rods), Streptococcus spp What tests do you want to order? - U/A C&S, Urine HCG, Chlamydia/Gonorrhea swab, Trich How would you treat this patient? - Azithromycin What will happen if she does not receive treatment? - She could have scarring and infertility

CC - "I have back pain" A 58-year old lawyer presents to the emergency room with a headache, generalized muscle pain and severe, uncontrollable back spasms x 2 days. Pain is a constant tightness 7/10 and spasms are a 10/10. Pain does not radiate anywhere. He is very irritable, easily annoyed, and worried because he has had the back spasms throughout his court case that day and they have become extremely painful. Pt notes he is under a lot of stress and works for a very busy practice. Pt has taken Ibuprofen with no relief and nothing worsens the pain. Upon further discussion pt admits that he injured his right arm 10 days ago. He was punctured by a nail while tearing down a barn on his family's property. Pt notes that his arm is painful and oozing, but he hasn't had time to get it seen due to his schedule. Pt is unaware of his last tetanus. States he is only seen yearly with PCP for bp medication refills. ■PMHx - Hypertension controlled with meds ■Medication - Atenolol 50mg once daily ■Allergies - NKDA ■Vitals: BP 142/82mmHg LA lying flat, HR 96bpm, RR 10bpm, Temp 99F Oral ■The patient is in acute distress while observed in the hospital bed. He is sitting on the edge of the bed grimacing when spasming occurs. On exam, his back appears to have very tight contractions and spasms. Paraspinal muscle spasms are noted. ■The right upper extremity wound is exquisitely tender to palpation with surrounding erythema and edema (2cmx2cm). +purulent discharge noted with light pressure over area

What is your diagnosis? - Tetanus and possible Abscess (MRSA vs MSSA) What is the proper treatment in this case? - Pt to be admitted to hospital - Antitoxin (TIG), antibiotics, wound debridement, muscle relaxers, sedatives Any other tests to be completed? - Culture and Sensitivity of discharge from wound What will happen if he is not treated? - Tetanus can be fatal How could this have been prevented? - Staying up to date on tetanus vaccine or getting the vaccine when he was punctured by the nail. NOT ON QUIZ

How do we get fluids into the pt?

•Dependent on cannula gauge •Gravity - ~120 mL/min (14g) •Pressure bag - ~184 mL/min (14g) •Most standard pumps - 999 mL/hour •Level 1 infuser - up to 500 mL/min!

Long-acting opioids

•Modified release tramadol •Methadone •Modified-release hydromorphone •Modified-release morphine •Modified-release oxycodone •Transdermal buprenorphine •Transdermal fentanyl

CC: "My tongue looks funny" HPI: 61 y/o M patient presents complaining "my tongue looks funny and white.". Pt. states that for the last 2 weeks he has noticed his tongue has a white appearance. Pt. denies any trauma or eating different foods. Pt. states he went 2 weeks ago for his usual chemotherapy treatment for small cell lung cancer. Pt. denies any recent fevers. He states he feels tired a lot. Pt admits to mild discomfort of tongue. Last dental exam was 3 years ago. Denies any mouth bleeding or difficulty swallowing. Notes that he is just concerned. Med Hx: Small Cell Lung CA, DM, HTN, CAD, COPD Meds: Metformin 500mg PO BID, Lisinopril 10 mg PO daily, Lipitor 40mg PO daily, Albuterol MDI prn, Advair Diskus 250/50 one puff BID *Compliant with all meds* Allergies: Ciprofloxacin (Rash) Soc Hx: Smoker quit 2 years when diagnosed but smoke 1 PPD for 40 years. No alcohol or illicit drugs. Vitals BP 109/80mmHg HR 64bpm RR 14bpm O2sat 96% RA Temp 99.4 F Oral PE: Tongue blade is used to scrape off whitish patches which have slight bleeding after examining Special Considerations?

oropharyngeal candidiasis (oral thrush) What are pt's recent labs for DM control? - Last reported 5 months ago and HgbA1c 8.3% and FBG 196 - Pt doesn't check glucose at home. Notes that he "feels fine" Does pt rinse his mouth after using Advair? - Pt notes he tries to remember, but sometimes forgets Pt is on chemo - think mucositis! Contraindications: none at this time Adverse Rxns: none Safety/Monitoring: if no improvement return.. Special Considerations: Due to chemo tx - think mucositis - If continues - will need to possibly discuss with onco in regard to a prophylactic fluconazole- Efficacy still be determined at this time - Will need to monitor to make sure that fungal infection improves and does not become systemic- High risk due to multiple factors: Chemo, DM, Inhaled steroids Counsel pt on DM control - Order updated labs - Alter medications pending results Counsel pt on rinsing mouth after using Advair - Use when brushes teeth in the am and pm Pt to follow up with dentist - he is overdue for his annual cleaning

CC- "I need refills on my psoriasis cream" 48 yo Caucasian man comes into the office. He has hx of psoriasis that has worsened over recent years. Admits to using creams in the past with some relief. Has not had any medication for the last 5 years due to lack of insurance. Patient is here today to restart treatment. Pt remembers that he did take a medication in the past that worked but doesn't remember what the name was. States it came in a "red tube". •PMHx: Psoriasis, HTN •FamHx: Unknown •Medications: Hydrochlorothiazide 25mg once daily (compliant) •Allergies: PCN Counseling? Tx options?

psoriasis •Counsel pt on medication compliance •RTC w/in 2-4 weeks to assess improvement Betamethasone dipropionate 0.05% Apply to affected area twice daily for 30 days #45 grams (forty-five) Refills: 1 (one)

How should a prescription be written in compliance with the new controlled substance prescribing laws (effective July 01, 2018)?

When writing an acute pain prescription for a 3-day supply of a schedule II controlled substance: · Write "acute pain" When writing an acute pain prescription for a 7-day supply of a schedule II controlled substance: · Write "acute pain exemption" All new prescriptions will have two boxes to check on the prescription for "Acute pain" or "non acute pain"

When is a PA allowed to write a prescription for a 7-day supply of a schedule II controlled substance?

1. The prescriber, in his or her professional judgment, believes that more than a 3-day supply of such an opioid is medically necessary to treat the patient's pain as an acute medical condition 2. The prescriber must indicate "ACUTE PAIN EXCEPTION" on the prescription 3. The prescriber adequately documents in the patient's medical records the acute medical condition and lack of alternative treatment options that justify deviation from the 3-day supply limit established in this subsection.

Peds Fluid Calculations •4 kg •54 kg •17 kg •102 kg •1200 g •48 kg •23 kg

4 kg = 16 mL/hr 54 kg = 94 mL/hr 17 kg = 54 mL/hr 102 kg = 142 mL/hr 1200 g = 4.8 mL/hr 48 kg = 88 mL/hr 23 kg = 63 mL/hr

Pain control - MC used is going to be Tylenol and ibuprofen IF NEEDED - as an outpatient is mostly Tylenol with codeine Hydrocodone and Oxycodone are used rarely and are usually for surgical issues Analgesic: ◦Codeine 0.5-1mg/kg/dose by mouth ever 4-6 hours as needed Max 60mg/dose (codeine) Elixir, oral suspension, or oral soln ◦3-6yo 5mL (12mg codeine and 120mg apap) every 6-8 hrs prn ◦7-12yo 10mL (24mg codeine and 240mg apap) every 6-8 hrs prn ◦> 12 yo 15mL (36mg codeine and 360mg apap) every 4 hrs prn

7-12 yr old example

Definition of a Prescription

A prescription is a written, verbal, or electronic order from a practitioner or designated agent to a pharmacist for a particular medication for a specific patient.

A 23-year-old male presents to the Emergency Department with high fever, dilated pupils, hyperreflexia on DTRs, and bilateral Babinski. However, there is no muscle rigidity. Which of the following medications is most likely the cause? A. Sertraline (Zoloft) B. Amitriptyline (Elavil) C. Bupropion (Wellbutrin) D. Haldol (Haloperidol)

A. Sertraline (Zoloft) SSRIs have the greatest association with Serotonin Syndrome

Contents of the Prescription

ALL prescriptions, regardless of prescriber, should have the following essential elements: Date of the order Allows the determination of the life of the prescription to validate refills •Legend drugs - expire 1 year from date issued •Controlled drugs - III-V - expire 6 months from date issued - II - expire 7 days from date issued Pt Name and Date of Birth Name of the drug Strength of the drug Quantity of the drug Directions for use Under Florida State Law, every prescription written by a PA must contain the following info pre-printed on security-type paper: •Prescribing PA's name •PA's prescribing number •Supervising physician's name •Practice address and telephone number

Maximize pt safety

ALWAYS write legibly. ALWAYS space out words and numbers to avoid confusion. ALWAYS complete medication orders. AVOID abbreviations. When in doubt, ask to verify.

Good Prescribing Practice: Keep it simple

AVOID Multiple drugs per prescription -- can add to confusion KEEP IT SIMPLE

Good Prescribing Practice: Avoid the Use of

Abbreviations Many drugs identified with abbreviations •EX: HCTZ for hydrochlorothiazide, MSO4 for morphine sulfate Attempts to standardize abbreviations have been unsuccessful JCAHO (January 1, 2004): "Do Not Use" List: Abbreviations, Acronyms, and Symbols Prescribing and dispensing errors have led to a number of adverse events (N/V, hallucinations, AMS, lethargy, seizures, and death)

CC - "I have pimples" A 17 yo Caucasian male is seen in your office with mom. Pt notes that he has some "pimples" and wants to get rid of them. Mom notes she has purchased multiple OTC facewashes and lotions that include; Neutrogena, Noxzema, Clearasil, Pro-Active and none have been successful in helping patient with skin. Pt notes he doesn't like to wash his face and so the products that have more steps cause him to not complete the regimen. Patient plays football, soccer and rows on the crew team. He will wear sunscreen when outside sometimes, but not all the time. Patient does wear a hat when outdoors. •PMHx: None •FamHx: Mom 50 (HTN), Dad 48 (Neg Hx), MGF 72 (HTN/DM2), PGM 68 (Depression) •Medications: None •Allergies: None Tx besides Erythromycin Gel 2%?

Acne

CC - "I have pimples" A 17 yo Caucasian male is seen in your office with mom. Pt notes that he has some "pimples" and wants to get rid of them. Mom notes she has purchased multiple OTC facewashes and lotions that include; Neutrogena, Noxzema, Clearasil, Pro-Active and none have been successful in helping patient with skin. Pt notes he doesn't like to wash his face and so the products that have more steps cause him to not complete the regimen. Patient plays football, soccer and rows on the crew team. He will wear sunscreen when outside sometimes, but not all the time. Patient does wear a hat when outdoors. •PMHx: None •FamHx: Mom 50 (HTN), Dad 48 (Neg Hx), MGF 72 (HTN/DM2), PGM 68 (Depression) •Medications: None •Allergies: None Would you counsel the pt on anything? Tx options?

Acne -Facewashing -Suncscreen usage -Tallk to pt about realistic goals -No PICKING!! Tx Erythromycin Gel 2% Apply thin layer to face twice daily after washing for acne #60 grams (sixty) Refills: 1 (one) Also Benzoyl peroxoide 5% Apply thin layer to face after washing at bedtime #60 grams (sixty) Refills 1 (one)

CC - "I am having difficulty breathing and a cough x 3 days" A 24 yo African American female, known asthmatic, presents to your office with difficulty breathing x 3 days. States that she uses Pulmicort daily and ProAir HFA as needed. Pt also has hx of allergies and does that Fexofenadine (Allegra) daily. States went to visit her friend over the weekend and her friend had adopted a new kitten. Pt states since the visit with her friend she has had to use her ProAir more recently throughout the day and it is no longer helping. She normally uses her ProAir maybe 1x a week but is now having to use it at least 4x a day. Today she has used it 3x with no relief of tightness in her chest. Denies any hx of hospitalization or intubation for uncontrolled asthma. Pt has been asthmatic since 6 years old. MEDS: Pulmicort 180mcg 2 puffs inhaled 2x daily - compliant, ProAir HFA 90mcg 1-2 puffs every 4-6 hours as needed for asthma control - compliant, Fexofenadine 180mg daily for allergies - compliant. ALLERGIES - seasonal, cats, dogs, pollen, PCN - hives FHx - Mother (48) and Father (50) - Alive and Well, Sister (26) Asthma SHx - No tobacco, vaping, alcohol, illicit drug usage VITALS - normal - Pulse - 98bpm, O2 - 96% RA sitting, RR 15bpm PE: Wheezing auscultated B/L. Albuterol Neb given in office, improvement of wheezing upon repeat auscultation and O2 recheck went to 99% RA sitting, Pulse 110bpm Dx? What to do?

Acute Asthma Exacerbation **For glucocorticoid courses lasting 3 weeks or less, there is no need to taper the dose if pts are also taking inhaled glucocorticoids** When do we start worrying about our asthmatics for home tx and send them to the hospital??? - Breathless at rest, sitting forward - Drowsy, confused, or agitated - Unable to speak in full sentences - Respiratory rate >30 breaths/minute - Heart rate >120 beats/minute - PEF ≤50 percent predicted or personal best or unable to perform PEF - Arterial oxygen saturation (SpO2) <90%

CC: "I have a headache and my face hurts really bad." HPI: 21 y/o M presents complaining of facial pain/pressure and headache for 8 days. Pt admits to "yellowish thick snot" Pt. denies any recent injury or trauma. Pt. states he feels "facial pressure" worsening when he leans forward to tie his shoes. Denies any ear pain, sore throat, neck pain, vomiting or fever. Pt. has taken OTC Alka Seltzer Cold with no relief. PMH: ADHD, Seasonal Allergies Surg Hx: None Soc Hx: College student denies any drugs, beer socially, no tobacco Meds: Adderall 10mg PO BID Allergies:NKDA Vitals BP 138/76mmHg HR 100bpm RR 18bpm O2sat 100% RA Temp 99.6 F Oral PE: •Positive tenderness over frontal sinus region on palpation and percussion •Transillumination reveals difficulty with light shining through and opaque appearance Medication Considerations?

Acute Bacterial Rhinosinusitis Contraindications: none Adverse Rxns: N/V/D? Take with food Safety/Monitoring: Nothing concerning for specific pt

CC: "I have a headache and my face hurts really bad." HPI: 21 y/o M presents complaining of facial pain/pressure and headache for 8 days. Pt admits to "yellowish thick snot" Pt. denies any recent injury or trauma. Pt. states he feels "facial pressure" worsening when he leans forward to tie his shoes. Denies any ear pain, sore throat, neck pain, vomiting or fever. Pt. has taken OTC Alka Seltzer Cold with no relief. PMH: ADHD, Seasonal Allergies Surg Hx: None Soc Hx: College student denies any drugs, beer socially, no tobacco Meds: Adderall 10mg PO BID Allergies:NKDA Vitals BP 138/76mmHg HR 100bpm RR 18bpm O2sat 100% RA Temp 99.6 F Oral PE: •Positive tenderness over frontal sinus region on palpation and percussion •Transillumination reveals difficulty with light shining through and opaque appearance What would you give besides Augmentin?

Acute Bacterial Rhinosinusitis Contraindications: none Adverse Rxns: N/V/D? Take with food Safety/Monitoring: Nothing concerning for specific pt

CC: "I have a headache and my face hurts really bad." HPI: 21 y/o M presents complaining of facial pain/pressure and headache for 8 days. Pt admits to "yellowish thick snot" Pt. denies any recent injury or trauma. Pt. states he feels "facial pressure" worsening when he leans forward to tie his shoes. Denies any ear pain, sore throat, neck pain, vomiting or fever. Pt. has taken OTC Alka Seltzer Cold with no relief. PMH: ADHD, Seasonal Allergies Surg Hx: None Soc Hx: College student denies any drugs, beer socially, no tobacco Meds: Adderall 10mg PO BID Allergies:NKDA Vitals BP 138/76mmHg HR 100bpm RR 18bpm O2sat 100% RA Temp 99.6 F Oral PE: •Positive tenderness over frontal sinus region on palpation and percussion •Transillumination reveals difficulty with light shining through and opaque appearance What would you give besides Fluticasone Propionate?

Acute Bacterial Rhinosinusitis Contraindications: none Adverse Rxns: N/V/D? Take with food Safety/Monitoring: Nothing concerning for specific pt

Chronic pain impacts on the whole person and their community

Ask your pts about any changes that have happened in their lives since their ongoing pain started. For example: •How are you sleeping? •Are you feeling worried or sad? •Are you still visiting friends or relatives? •Are you eating well? Chronic pain can cause many problems. Normal activities may become difficult - the person may feel tired and not interested or motivated to do anything Mood changes (depression, anxiety) - this can make other conditions more difficult to manage as the patient not be motivated to take action or be too focussed on their pain. Less social interaction. Poor sleep - this can also effect mood and the ability to do normal activities. Changed appetite - this may also effect other conditions where healthy lifestyle and diet are important such as diabetes and heart disease

A 5 yo Caucasian male accompanied by mom presents to your clinic due to recent cough x 1 month. Mom states that the patient usually gets a cough in the spring that is improved with allergy medications, but no improvement this time. She is giving him loratadine 5mg at night for allergies. She notes that she hears the patient coughing throughout the night and it is keeping her up. No known sick contacts. Cough is dry, no phlegm. Patient has no runny nose, ear pain, sore throat, head pain, N/V/D. PMHx - seasonal allergies FAMHx - Mom and Dad Alive and Well, no siblings. Grandparents - unknown history MEDS - Loratadine (Claritin) 5mg/5mL daily prn for allergies - compliant ALLERGIES - Pollen VITALS - Normal, O2 99% RA Tx besides Ventolin?

Asthma

CC - "I have a cough" A 5-year-old Caucasian male accompanied by mom presents to your clinic due to recent cough x 1 month. Mom states that the patient usually gets a cough in the spring that is improved with allergy medications, but no improvement this time. She is giving him loratadine 5mg at night for allergies. She notes that she hears the patient coughing throughout the night and it is keeping her up. No known sick contacts. Cough is dry, no phlegm. Patient has no runny nose, ear pain, sore throat, head pain, N/V/D. PMHx - seasonal allergies FAMHx - Mom and Dad Alive and Well, no siblings. Grandparents - unknown history MEDS - Loratadine (Claritin) 5mg/5mL daily prn for allergies - compliant ALLERGIES - Pollen VITALS - Normal, O2 99% RA Tx besides Proventil?

Asthma

CC - "I have a cough" A 5-year-old Caucasian male accompanied by mom presents to your clinic due to recent cough x 1 month. Mom states that the patient usually gets a cough in the spring that is improved with allergy medications, but no improvement this time. She is giving him loratadine 5mg at night for allergies. She notes that she hears the patient coughing throughout the night and it is keeping her up. No known sick contacts. Cough is dry, no phlegm. Patient has no runny nose, ear pain, sore throat, head pain, N/V/D. PMHx - seasonal allergies FAMHx - Mom and Dad Alive and Well, no siblings. Grandparents - unknown history MEDS - Loratadine (Claritin) 5mg/5mL daily prn for allergies - compliant ALLERGIES - Pollen VITALS - Normal, O2 99% RA Dx? What to do?

Asthma What do you want to do? Add an inhaler - beta 2- agonist (SABA inhaler) PFTs for baseline Possible referral pending on pt's rxn with medication and control of asthma Can write prescription for a spacer

CC - "I itch and have a rash" 10 yo African American female seen in office today for itching and rash of popliteal fossa. Dad notes that daughter has had a rash on and off. Rash worsens with scratching and hot showers. Improves with OTC meds. Mom has been using hydrocortisone and Aquaphor at home with some relief. Patient has hx of asthma, but it is well controlled and only uses rescue inhaler in the spring when the pollen count is high. No new soaps/lotions/detergents. Mom notes that she uses fragrance free detergent and Aveeno eczema wash/lotion on her child. • PMHx: Asthma, Seasonal Allergies •FamHx: Mother 35 (None), Father 35 (None), MGF 60 (HTN, DM2), PGM 58 (HTN) •Medications: loratadine 5mg daily as needed for allergies, Albuterol 0.083% with neb 3x daily as needed •Allergies: PCN, Cefdinir Counseling points? Tx options?

Atopic Dermatitis •Continue w/ loradane •Utilize cream as prescribed (keep area hydrated •Keep baths/showers & warm water NOT hot water •Try to keep exposure to allergies at minimum Tx: Topical steroid Eucrisa or Tacrolimus Triamcinolone 0.1% CREAM Apply to affected area three times daily as needed for atopic dermatitis. Repeat as needed for up to 2 weeks #30 gram (thirty) Refills 0 (zero)

A 27-year-old woman notices that her mouth has been drooping in the right corner, and it is difficult for her to drink water without drooling, since 1 day ago. She cannot close her right eye completely, and her right eye is red and irritated. She denies having HA, visual disturbances, N/V. She does not have any history of trauma. Her PMH is unremarkable. She is not taking any medications. Her mother had a stroke when she was 60 years old. The pt states that she is from Michigan and has not been traveling recently. You notice the right corner of her mouth droops, and the right nasolabial fold is absent. The right lower eyelid is sagging, and the patient cannot completely close her right eye. On attempts to close the right eye, the eye rolls upward. The patient also cannot wrinkle her forehead. The other cranial nerves seem to be normal, and the neurologic examination reveals no deficits other than as stated. What is your Dx?

Bell's Palsy How will you manage this condition? Steroids and Antivirals If you suspect herpes simplex virus as a cause, what is your further treatment? Anti-virals as noted prior Also keep eye lubricated to not dry out

CC: "I have a headache and my face hurts really bad." HPI: 21 y/o M presents complaining of facial pain/pressure and headache for 8 days. Pt admits to "yellowish thick snot" Pt. denies any recent injury or trauma. Pt. states he feels "facial pressure" worsening when he leans forward to tie his shoes. Denies any ear pain, sore throat, neck pain, vomiting or fever. Pt. has taken OTC Alka Seltzer Cold with no relief. PMH: ADHD, Seasonal Allergies Surg Hx: None Soc Hx: College student denies any drugs, beer socially, no tobacco Meds: Adderall 10mg PO BID Allergies:NKDA Vitals BP 138/76mmHg HR 100bpm RR 18bpm O2sat 100% RA Temp 99.6 F Oral Positive tenderness over frontal sinus region on palpation and percussion Transillumination reveals difficulty with light shining through and opaque appearance Would you write an Antibiotic Rx and why/why not?

Bacterial RhinoSinusitis Depending on source - needs symptoms for at least 7-10 days prior to antibiotics vs treating symptomatically Pt to take OTC medications to aid with sxs - Saline nasal spray - Cetirizine/Loratadine/Fexofenadine are all OTC- Tylenol/Ibuprofen to be alternated for facial pain/pressure - Increase fluids

CC: "I have a headache and my face hurts really bad." HPI: 21 y/o M presents complaining of facial pain/pressure and headache for 8 days. Pt admits to "yellowish thick snot" Pt. denies any recent injury or trauma. Pt. states he feels "facial pressure" worsening when he leans forward to tie his shoes. Denies any ear pain, sore throat, neck pain, vomiting or fever. Pt. has taken OTC Alka Seltzer Cold with no relief. PMH: ADHD, Seasonal Allergies Surg Hx: None Soc Hx: College student denies any drugs, beer socially, no tobacco Meds: Adderall 10mg PO BID Allergies:NKDA Vitals BP 138/76mmHg HR 100bpm RR 18bpm O2sat 100% RA Temp 99.6 F Oral Positive tenderness over frontal sinus region on palpation and percussion Transillumination reveals difficulty with light shining through and opaque appearance Cost of meds out of pocket (no insurance/high co-pay)

Bacterial RhinoSinusitis Pt's compliance with medication Will patient return for assessment? Should I try OTC meds 1st? Contraindications: none Adverse Reactions: N/V/D? Take w/ food Safety/Monitoring: Nothing concerning for specific pt

CC: "My vagina itches and there is a smelly discharge" x 3days HPI: 28 year old female presents with vaginal pruritus and foul smelling discharge x 3days. She states she has been douching to help get rid of the odor but it is making it worse. No pain, no abd pain, no f/n/v/d. No vag bleeding. Has intercourse with husband, monogamous relationship •PMH: None •Meds: Depo Provera 150mg IM q 13 weeks •Surg Hx: None •Soc Hx: Denies smoking, drinks 2 beers daily, no illicit drugs. •VS BP 120/68 HR 88 RR 17 O2sat 98% Temp 98.6 •Rest of exam unremarkable •U/A: WNL •Urine hCG: Neg •Vag Cx: Pending •Wet prep: POS Clue cells, no budding yeast, no trichomonads How do you treat?

Bacterial Vaginosis •Flagyl (metronidazole) 500mg PO BID x 7days NO ALCOHOL •MetroGel 1 app full 5g containing 37.5mg metronidazole intravag qhs or BID x 5 days •MetroGel 1 app full 5g containing 65mg metronidazole intravag qhs ONCE •Solosec (secnidazole)- 2g oral granules PO mixed in food X1 dose Metronidazole ER 750 oral capsule. Take one capule by mouth once a day x 7days for or clindamycin gelbacterial vaginosis. Take on empty stomach

A 27-year-old woman notices that her mouth has been drooping in the right corner, and it is difficult for her to drink water without drooling, since 1 day ago. She cannot close her right eye completely, and her right eye is red and irritated. She denies having HA, visual disturbances, N/V. She does not have any history of trauma. Her PMH is unremarkable. She is not taking any medications. Her mother had a stroke when she was 60 years old. The pt states that she is from Michigan and has not been traveling recently. You notice the right corner of her mouth droops, and the right nasolabial fold is absent. The right lower eyelid is sagging, and the patient cannot completely close her right eye. On attempts to close the right eye, the eye rolls upward. The patient also cannot wrinkle her forehead. The other cranial nerves seem to be normal, and the neurologic examination reveals no deficits other than as stated. Tx besides prednisone 20 mg tablet?

Bell's Palsy

A 27-year-old woman notices that her mouth has been drooping in the right corner, and it is difficult for her to drink water without drooling, since 1 day ago. She cannot close her right eye completely, and her right eye is red and irritated. She denies having HA, visual disturbances, N/V. She does not have any history of trauma. Her PMH is unremarkable. She is not taking any medications. Her mother had a stroke when she was 60 years old. The pt states that she is from Michigan and has not been traveling recently. You notice the right corner of her mouth droops, and the right nasolabial fold is absent. The right lower eyelid is sagging, and the patient cannot completely close her right eye. On attempts to close the right eye, the eye rolls upward. The patient also cannot wrinkle her forehead. The other cranial nerves seem to be normal, and the neurologic examination reveals no deficits other than as stated. Tx besides prednisone?

Bell's Palsy

American Thyroid Association Guidelines (2016)

Beta-adrenergic blockade is recommended in all pts with symptomatic thyrotoxicosis, especially elderly pts and pts with resting heart rates in excess of 90 beats per minute Pts with overt Graves hyperthyroidism should be treated with any of the following modalities: radioactive iodine therapy, antithyroid drugs, or thyroidectomy If methimazole is chosen as the primary therapy for Graves disease, the medication should be continued for approximately 12-18 months and then discontinued If surgery is chosen as the primary therapy for Graves disease, near-total or total thyroidectomy is the procedure of choice Euthyroidism should be expeditiously achieved and maintained in hyperthyroid patients with Graves ophthalmopathy or risk factors for the development of ophthalmopathy In pts with Graves hyperthyroidism who have mild active ophthalmopathy and no risk factors for deterioration of their eye disease, radioactive iodine therapy, antithyroid drugs, and thyroidectomy should be considered equally acceptable therapeutic options In Graves disease patients with mild Graves ophthalmopathy who are treated with radioactive iodine, steroid coverage is recommended

CC - "my baby has a fever x 3 days" A 6-month-old Caucasian female was brought into the office due to fever x 3 days. Mom notes that baby is feeding well and having wet/dirty diapers like normal. Mom states child is up all-night coughing and is fussier than normal. Mom has given child Tylenol with relief of fever. Notes that the highest fever was 101.1F rectally. Child attends daycare but no one has been sick at home Vitals - Temp 100F Rectally (last Tylenol dosage 2 hours ago) O2 - 96% RA sitting. Otherwise, normal PE - +mild retractions noted upon exam, +wheezing noted b/l +rhinorrhea noted, pt crying during exam. Neb given in office and O2 - 98% after neb and wheezing improved b/l Tx besides Albuterol?

Bronchiolitis: Possible RSV

CC - "my baby has a fever x 3 days" A 6-month-old Caucasian female was brought into the office due to fever x 3 days. Mom notes that baby is feeding well and having wet/dirty diapers like normal. Mom states child is up all-night coughing and is fussier than normal. Mom has given child Tylenol with relief of fever. Notes that the highest fever was 101.1F rectally. Child attends daycare but no one has been sick at home Vitals - Temp 100F Rectally (last Tylenol dosage 2 hours ago) O2 - 96% RA sitting. Otherwise, normal PE - +mild retractions noted upon exam, +wheezing noted b/l +rhinorrhea noted, pt crying during exam. Neb given in office and O2 - 98% after neb and wheezing improved b/l Dx? What to do?

Bronchiolitis: Possible RSV What do you want to do? Albuterol sent home due to improvement in office (supportive tx) What is the next step? Prescriptions and symptomatic care at home Any counseling for mom? - Mom to monitor breathing - if worsens, to take child to ER immediately for assessment, mom can watch for retractions - Continue with Tylenol for fever - If fever increases over 102.2F to go to ER immediately - Return to clinic in 3 days for reassessment

A 28 yo woman presents with a 2-month hx of dominant R hand pain & paresthesia of the thumb, index, & middle fingers for the past 3 months. She works in a retail sales office & spends much of the day typing on a keyboard. PE reveals a + Tinel's sign at the R wrist. She would like a prescription strength medication to relieve her pain since OTC ibuprofen failed to help. Her only meds are BCPs. NKDA.

Carpal Tunnel Syndrome Any Rx NSAIDs, but COX-2 agents have least SAE than non-selective NSAIDS- CELECOXIB (Celebrex).

A 28 yo woman presents with a 2-month hx of dominant R hand pain & paresthesia of the thumb, index, & middle fingers for the past 3 months. She works in a retail sales office & spends much of the day typing on a keyboard. PE reveals a + Tinel's sign at the R wrist. She would like a prescription strength medication to relieve her pain since OTC ibuprofen failed to help. Her only meds are BCPs. NKDA. Meds besides Naproxen?

Carpal Tunnel Syndrome Any Rx NSAIDs, but COX-2 agents have least SAE than non-selective NSAIDS- CELECOXIB (Celebrex).

CC: "Lump on my right eyelid" HPI: A 34 y/o male presents with complaints of a painless lump to the right eyelid for 3 days. Denies any trauma or injury. Denies any fever or insect bites. Notes that since he noticed the area it hasn't gotten larger. Doesn't know what to do. PMH: Hyperlipidemia Meds: Fish Oil, 1000 mg PO BID Allergies: NKDA Soc Hx: Married, accountant, no alcohol, tobacco, or illicit drugs. VS: BP 124/76mmHg RA sitting, HR76bpm, RR16bpm, Temp 98.4F Oral PE: Visual Acuity OD 20/30, OS 20/20, OU 20/30; Pupils PERRLA EOMI +painless, firm rubbery nodule noted on upper right eyelid How to Tx?

Chalazion No antibiotic needed Counsel pt on warm compresses (if stye abxs) If no improvement or visual impairment occurs, pt to see ophthalmologist

A 35-year-old woman presents for her Pap smear. She tells you that she and her husband are now ready to start a family, but they are worried about the hypertension the patient has been suffering from for a year. She was seen in the hospital for chest pain 2 months ago and was prescribed Lisinopril 10mg daily. Notes she doesn't check her bp at home. Denies any chest pain, shortness of breath, pedal edema, palpitations, dizziness, headaches. ALLERGIES: NKDA MEDICATONS: Lisinopril 10mg daily (compliant) PMHx: HTN FamHx: HTN Maternal Grandmother (83), Paternal Grandmother (81), Mom (59), Dad (62) and Sister (37). Vital signs:130/80mmHg RA sitting, P 80 regular and RR 12 unlabored. What do we want to do with her medication?

Change it! Lisinopril is not baby friendly AT ALL What would you like to change it to? Methyldopa written - it is very common and well known to be safe for pregnancy. However, is only mildly effective and therefore might not achieve BP control on this medication Labetalol and Nifedipine are other options that are pregnancy safe for pt She would also need to start Pre-natal vitamins with Folic Acid and DHEA to aid with proper fetal growth and decrease chance of spina bifida Pt to return to office in 2 weeks for bp monitoring and then return if needed for OB and MFM referral once she is pregnant for follow ups

A 32 y/o Asian F presents complaining of palpitations and dyspnea with exertion worsening for the past 2 weeks. Symptoms started about a month ago but were intermittent and now are occurring multiple times a day. No chest pain or pressure. Additionally, she reports a 14 lb weight loss over the past 2 months, feeling hot and sweaty all of the time, noticeable hair loss, and irritability. PMHx - no significant medical history Fam Hx - father has HTN, mother has thyroid problems Social Hx - lives with husband and 2 daughters (7 & 5 yrs old), no tobacco use, drinks alcohol occasionally (2-3 drinks a month) Meds - multivitamin Allergies - Sulfa meds (rash) ROS - No visual changes, occasional nausea and diarrhea. Gen: NAD, A&O x 3, slender build VS: BP-130/78mmHg LA sitting, P-120 to 160bpm irregular, RR-20bpm, T-100.6 F Oral, Wt-128 lbs, Ht-5'6" Skin: Hyperpigmented on upper back and lower extremities, warm and moist. Hair is fine and sparse in the frontal area. HEENT: PERRLA, EOMI, + Lid lag, mild proptosis, mild lid retraction NECK: supple, smooth symmetrically enlarged thyroid, + thyroid bruit, prominent pulsations in neck vessels Lungs: CTA bilat, no rhonchi, rales or wheezes CV: Irregularly irregular rhythm, + tachycardia, no murmurs or rubs Abd: soft, nontender, + bowel sounds, No HSM or masses, Aortic pulsations are palpable Rectal: guaiac negative Extremities: pulses 2+ throughout, no calf tenderness, no clubbing or cyanosis, cap refill <2 secs. Nails - see photo Neuro: fine tremor with outstretched hands, hyperreflexia at knees, no muscle weakness Goiter present Lower extremities has pretibial myxedema Nails are brittle Labs: see powerpoint ECG: in Afib BB med?

Close follow-up visits with monitoring of examination findings, thyroid hormone levels, and thyrotropin levels are required. Continue anticoagulation for A-fib if it is persistent (we will discuss this in the cardio lecture) If the patient is on antithyroid medication, liver function tests and CBC counts with differentials should be monitored based on the clinical situation.

Short-acting opioids

Codeine Tramadol Hydromorphone Immediate-release morphine Immediate-release oxycodone Immediate-release tramadol

24-year-old Caucasian female here today with complaints of Headaches. Note that she has had headaches for the previous 12-years. Pt missed school in the past due to the headaches. Notes they happen 1-2x per month and will be worse if it correlates with her menses. It is a constant pressure pain over her right eye. Notes that flashes of lights and zigzag lines precede the headache. +photophobia +N/V. Sitting in a dark room improves the pain (3/10) and drinking red wine can worsen her pain (8/10). Denies taking any medication for pain. Notes that headaches will last 4-6 hours and feels tired/listless for the next 24 hours. She wants to know what else can be done PE: Her general and neurological examinations are normal Vitals all WNL Tx other than NSAID?

Common Migraine

CC: "My stomach hurts" HPI - 52-year-old female presents to your office for abdominal pain. States her last BM was 7 days ago. Notes that she is having decreased appetite over the last few days. State pain is a constant, bloating pain (3/10). Denies anything making the pain worse or better. Has tried Magcitrate and Miralax x1 with no relief. Pt has hx of hypothyroid, does not exercise. States she eats vegetables daily and drinks around 32oz water daily. ■PMHx: hypothyroidism, IDA, ■MEDS: Levothyroxine 50 mg QD, Ferrous gluconate 324 mg TID - compliant ■ALLERGIES - None ■VITALS: 120/78mmHg LA sitting, HR 78bpm, RR 14bpm, Temp 98.6F Oral ■LABS: TSH is high, Free T4 is low, Ferritin is high What med could you give besides colace?

Constipation

CC - "I have a cough x 3 weeks" A 37 yo Caucasian female comes into the office for a cough. Notes that cough has been consistent over the last week. Pt admits to having bronchitis 3 wks ago and all symptoms have improved, but her cough stopped improving and has remained the same over the last week. Denies any phlegm, ear pain, sore throat, headache, coughing at night. Denies any history of allergies, asthma, eczema. Denies any sick contacts. Pt notes that cough will just occur throughout the day. Has tried OTC Robutussin, cough drops, Delsym with minimal relief. Pt wants to know what else can be done. States she took Azithromycin for bronchitis. ALLERGIES: None MEDICATIONS: None FHX - Mother and Father both alive and well, No siblings. MGM - 87 Deceased HTN PGM - 82 Deceased DM2 SHx - No alcohol, tobacco, vaping, illicit drug usage Vitals - all within normal O2 - 99%RA Tx besides Promethazine/Dextromethorphan?

Cough

CC - "I have a cough x 3 weeks" A 37 yo Caucasian female comes into the office for a cough. Notes that cough has been consistent over the last week. Pt admits to having bronchitis 3 wks ago and all symptoms have improved, but her cough stopped improving and has remained the same over the last week. Denies any phlegm, ear pain, sore throat, headache, coughing at night. Denies any history of allergies, asthma, eczema. Denies any sick contacts. Pt notes that cough will just occur throughout the day. Has tried OTC Robutussin, cough drops, Delsym with minimal relief. Pt wants to know what else can be done. States she took Azithromycin for bronchitis. ALLERGIES: None MEDICATIONS: None FHX - Mother and Father both alive and well, No siblings. MGM - 87 Deceased HTN PGM - 82 Deceased DM2 SHx - No alcohol, tobacco, vaping, illicit drug usage Vitals - all within normal O2 - 99%RA Tx besides Tessalon Perles?

Cough

CC - "I have a cough x 3 weeks" A 37 yo Caucasian female comes into the office for a cough. Notes that cough has been consistent over the last week. Pt admits to having bronchitis 3 wks ago and all symptoms have improved, but her cough stopped improving and has remained the same over the last week. Denies any phlegm, ear pain, sore throat, headache, coughing at night. Denies any history of allergies, asthma, eczema. Denies any sick contacts. Pt notes that cough will just occur throughout the day. Has tried OTC Robutussin, cough drops, Delsym with minimal relief. Pt wants to know what else can be done. States she took Azithromycin for bronchitis. ALLERGIES: None MEDICATIONS: None FHX - Mother and Father both alive and well, No siblings. MGM - 87 Deceased HTN PGM - 82 Deceased DM2 SHx - No alcohol, tobacco, vaping, illicit drug usage Vitals - all within normal O2 - 99%RA What would you do?

Cough What would you do? - Write for a cough medication and if no improvement - Xray to be completed - Possible referral to Pulm for PFTs and Spirometry Any tests you would order? - 3-view CXR - PFTs? Spirometry?

CC: "My eye is red" HPI: 7 y/o M present to the clinic with mom. Mom states that the school nurse called her to pick up her son because his Left eye is "red". Pt states it will sometimes itch, but denies any current discomfort. Patient is scratching at his eye throughout exam. Mom states that patient has history of allergies and she gives him Claritin daily. She states recently he has been coughing/sneezing and complaining of some ear pain. Denies any photophobia. Admits that he feels like he has something stuck in his Left eye. PMH: None Surg Hx: None Meds: Claritin 5mg daily Allergies: Pollen, NKDA Vitals: BP 96/68mmHg HR 109bpm RR19bpm O2sat 99%RA Temp 98.8F Oral Exam Findings: Left eye looks as seen in photo No purulent discharge noted Mom and patient deny any matting of eye What do you do?

Counsel pt on viral conjunctivitis Most likely viral due to unilateral Allergic is most often bilateral Return if worsens Handwashing - as always cold compresses, self-limiting, cromolyn sodium Allergic Vs Viral Allergic also has cobble stoning as well as redness bilaterally

HPI - A 38-year-old male presents to the office complaining of diarrhea. Denies any melena or hematochezia at this times. Notes sometimes will have bloody diarrhea. Notes that he has been having diarrhea that has worsened over the last 4 months. Pt admits to having watery stool 4-5x a day. States he is feeling fatigued. Denies any decrease in appetite. Admits to crampy (7/10) abdominal pain that improves with defecation. Denies any fevers, constipation, nausea or vomiting. Has been seeing PCP and taking lansoprazole (Prevacid) with relief for GERD. Referred to GI due to length of time he has had this diarrhea. PMHx: GERD/seasonal allergies MEDS: Fluticasone (Flonase), lansoprazole (Prevacid) SocHx - no tobacco, alcohol or illicit drug usage LABS: Hgb 11 g/dL; stool studies are negative Vitals: BP 130/72mmHg RA sitting, RR 14bpm, HR 110bpm, Temp 98.8F Oral PE positives: Pallor, tachycardia Colonoscopy: consistent with Crohn's Dx involving terminal ileum, ascending and transverse colon. What would you give besides predinisone?

Crohn's Dz

CC: "I have diarrhea x 4 months" HPI - A 38-year-old male presents to the office complaining of diarrhea. Denies any melena or hematochezia at this times. Notes sometimes will have bloody diarrhea. Notes that he has been having diarrhea that has worsened over the last 4 months. Pt admits to having watery stool 4-5x a day. States he is feeling fatigued. Denies any decrease in appetite. Admits to crampy (7/10) abdominal pain that improves with defecation. Denies any fevers, constipation, nausea or vomiting. Has been seeing PCP and taking lansoprazole (Prevacid) with relief for GERD. Referred to GI due to length of time he has had this diarrhea. PMHx: GERD/seasonal allergies MEDS: Fluticasone (Flonase), lansoprazole (Prevacid) SocHx - no tobacco, alcohol or illicit drug usage LABS: Hgb 11 g/dL; stool studies are negative Vitals: BP 130/72mmHg RA sitting, RR 14bpm, HR 110bpm, Temp 98.8F Oral PE positives: Pallor, tachycardia Colonoscopy: consistent with Crohn's Dx involving terminal ileum, ascending and transverse colon.

Crohn's Dz Select appropriate medication - Prednisone vs Budesonide Select non-pharmacologic therapy/interventions -Discuss with patient regarding probiotics and eating well to decrease inflammation. What are the contraindications or side effects of this medication? - Due to the area affected by chrons in this patient, steroids should be used only as a bridge to a longer-term medication pending if/when symptoms return and pending on how disease improves and if it fails steroid tx Notes: Steroid: Prednisone vs Budensonide Antidiarrheal? Long term: priobiotics, diet, high fiber diet, fruits,

CC "my child has had cough and wheezing since yesterday" A mother brings in her 2 yo Hispanic son for cough x 24 hours that is worsening. Mom notes that patient is gasping for air and has a very "barky" cough. Patient attends daycare and there have been multiple kids out sick recently with Upper Respiratory symptoms. Mom notes that patient had a little bit of a runny nose within the last few days, but the cough didn't start until yesterday. Mom states the child has had a mild temp at home up to 100.8F Tympanic at home. Mom has not given patient any medications. Patient has no significant medical history. ◦Allergies: NKDA ◦Medications: None ◦Vitals: Ht 32" WT 28lbs RR 32bpm Pulse 115bpm Temp 101F Tympanic O2 97% RA ◦PHYSICAL EXAM: Patient has a "barky" cough with stridor noted when you first start examining him. When mom is able to calm the patient, he doesn't cough, and no stridor heard on Respiratory exam ◦GENERAL: No cyanosis noted ◦CARDIAC: RRR S1S2 present ◦RESPIRATORY: CTA B/L Dx? Tx?

Croup How do you determine if mild/moderate/severe? ◦Westley Croup Severity Score ß this child is rated mild or a 2 What is your treatment plan? ◦0.15-0.6 mg/kg single dosage (Max 20mg/dosage) ◦28lbs child ◦28/2.2 = 12.7kg ◦12.7*0.6 = 7.6mg ◦One dosage of 7.5mL of the 1mg/1mL Dexamethasone given in office IF child improves - child can be sent home with counseling of - if breathing worsens, retractions occur, difficulty eating/drinking - go to ER IF child does NOT improve - Child would then be referred to the ER because the next step is Racemic Epinephrine which would be given in the ER setting and child would need be observed for 4-6 hours after administration. ◦Racemic Epi - 2.25% soln (0.05 to 0.1 mL/kg (maximum dose: 0.5 mL) diluted in 2 to 3 mL NS) May repeat every 20 minutes IF multiple rounds of RE needed - then child needs to be admitted to the hospital.

A 56-year-old males presents to the office for pain in his feet. Describes the pain not as a pain, but more of a "discomfort" States it feels as though there are "ants crawling" all over his feet. Has not tried anything OTC for the sensation. The sensation is constant. Nothing makes it worse or better. Sensation started a few years ago in his toes and has gradually now spread over his foot. Denies any history of trauma to his feet/back. Denies any other concerns today. PMHx : DM2 - HgbA1c - 10.6% (7/1/2020) Medications: Metformin 500mg daily (compliant) follows with endocrinology Vitals: WNL Dx?

Diabetic Peripheral Neuropathy Would you want to prescribe medication and if so, what kind? - Pregabalin (Lyrica) - Gabapentin (Neurontin) - Amitriptyline (Elavil) - Duloxetine (Cymbalta)

A 65-year-old Caucasian hypertensive obese male patient presents to your Urgent Care with complaints of intermittent palpitations for 2 days. During the conversation, the patient indicates that he is under a lot of stress at his job. His blood pressure is 175/102 mmHg RA sitting, pulse is 118 irregular. Physical Exam reveals clear lung fields with a normal S1, S2 without murmurs, rubs or gallops. There is no pedal edema. ECG is ordered and on the next slide. PMHx: HTN MEDS: None - Hasn't seen a provider in years ALLERGIES: NKDA SOC HX: No alcohol, tobacco or illicit drug usage CHADS2 score= 2 EKG shows Afib What would be the next step as a PCP/UC PA-C?

Due to new onset a fib <48 hours pt should be seen for further work-up in the ER. Prior to sending patient to the ER it needs to be determined is patient - Stable vs unstable - Symptomatic vs Asymptomatic Symptomatic = due to palpitations Stable = pt does not have any signs from hx and presentation to be deemed hemodynamically unstable. Recommend pt to be seen in the ER Pt's daughter drives him across the street to the ER

A 56 yo male with Type II DM presents to the ER after waking up in the morning to find that he had a swollen, red, painful big toe of his left foot. He had recently been on a cruise to the Bahamas and spent much of the time eating and drinking beer more than usual. An X-ray of his left foot shows no acute fracture. Vital signs are within normal limits. Blood work shows an increase in uric acid, but all other labs are otherwise unremarkable. The patient states he cannot tolerate NSAIDs due to a history of GI bleeding. NKDA.

Gout Colchicine (Colcrys) Prescribe correct dose of colchicine 0.6 mg tabs ( 2 tabs at first sign of flare, then one tab 1 hr later, not to exceed 3 tabs). SE: Diarrhea

ALL PAs must identify themselves to patients in 1 of the 5 following manners:

(1) The wearing of a name tag which identifies the licensee as a PA (2) The wearing of an article of clothing on the upper body which identifies the licensee as a PA (3) By orally disclosing to the patient, upon the licensee's initial in-person contact with the patient, that the licensee is a PA (4) By providing, upon the licensee's initial in-person contact with the patient, a business card or similar document which identifies the licensee as a PA (5) By placing notification in the lobby or waiting area of the location where the licensee practices, which contains a photo of the licensee and which identifies the licensee as a PA

Prescribing Errors: Misreading

A hypertensive pt accidentally received Vantin 200 mg instead of Vasotec 20 mg when a pharmacist misread this prescription

Breast Ca: Chemotherapy Agents

Doxorubicin (Adriamycin) Anastrozole (Arimidex) Exemestane (Aromasin) Cyclophosphamide (Cytoxan) Letrozole (Femara) Fluorouracil Methotrexate Nolvadex (Tamoxifen citrate) Paclitaxel (Taxol)

A 23-year-old female presents to the Family Practice Clinic with the complaint that she can not form any new memories. Her long-term memory is still intact. Which of the following medications is she most likely taking? A. Benzodiazepine B. Antidepressant C. Atypical antipsychotic D. Typical antipsychotic

A. Benzodiazepine

A 19-year-old male is on Lithium for Bipolar disorder. Which of the following is most likely a side effect to the medication? A. Polyuria B. Leukopenia C. Increased creatinine clearance D. P wave abnormalities

A. Polyuria

CC: "I have a headache and my face hurts really bad." HPI: 21 y/o M presents complaining of facial pain/pressure and headache for 8 days. Pt admits to "yellowish thick snot" Pt. denies any recent injury or trauma. Pt. states he feels "facial pressure" worsening when he leans forward to tie his shoes. Denies any ear pain, sore throat, neck pain, vomiting or fever. Pt. has taken OTC Alka Seltzer Cold with no relief. PMH: ADHD, Seasonal Allergies Surg Hx: None Soc Hx: College student denies any drugs, beer socially, no tobacco Meds: Adderall 10mg PO BID Allergies:NKDA Vitals BP 138/76mmHg HR 100bpm RR 18bpm O2sat 100% RA Temp 99.6 F Oral PE: •Positive tenderness over frontal sinus region on palpation and percussion •Transillumination reveals difficulty with light shining through and opaque appearance Would you write an Antibiotic Rx and why/why not?

Acute Bacterial Rhinosinusitis Depending on source - needs sxs for at least 7-10 days prior to abxs vs treating symptomatically Pt to take OTC medications to aid with sxs - Saline nasal spray - Cetirizine/Loratadine/Fexofenadine are all OTC - Tylenol/Ibuprofen to be alternated for facial pain/pressure - Increase fluids Contraindications: none Adverse Rxns: N/V/D? Take with food Safety/Monitoring: Nothing concerning for specific pt

Types of Pain

Acute pain can be important for the body to tell the brain that there is something wrong and help to avoid harm. A complex unpleasant experience that occurs in response to body trauma. It can affect the body and the mind. The cause is usually clear. It is usually easy to 'see' the pain, such as an injury or infection It lasts few days or weeks until healing has occurred - Examples of acute pain A broken leg: the initial pain is lessened by treatments such as splinting or putting on a cast. Pain then gradually gets less as the leg heals. A tooth infection: there may be a lot of pain which is helped by medicines to treat the infection, or by having the tooth removed. Pain goes once the infection (or the tooth) is gone. •Chronic pain is pain that goes on for a long time (>3 months). It needs regular assessment and a different approach to treatment. Is pain that lasts for more than three months, or beyond normal healing time. It is a persistent pain that can disrupt sleep, mood and normal living. The cause is not always clear. Chronic pain may start with an injury or infection, or there may be an ongoing cause of pain (e.g. arthritis). However, some people suffer chronic pain in the absence of any past injury or evidence of disease. Chronic pain is like any other chronic condition (like diabetes) it can have a big effect on people's lives and often needs long-term management. - Example of chronic pain: A person is in a car accident and damages their neck. Six months later, after the injured area had been treated and had time to heal, their pain is ongoing but there is no damage found on examination.

Chemo Drugs

Alkylating agents - cyclophosphamide (Cytoxan), chlorambucil, bendamustine (Treanda), Ifosfamide (causes hemorrhagic cystitis, prevent this by giving with Mesna), melphalan, dacarbazine Platinum drugs - cisplatin, carboplatin, oxaliplatin Purine analogs - fludarabine (Fludara), pentostatin (Nipent), cladribine (Leustatin) Anti-metabolites - cytarabine (ara-C), gemcitabine (Gemzar), methotrexate, pralatrexate (Folotyn), 5-fluorouracil (5-FU), 6-mercaptopurine (6-MP), capecitabine (Xeloda), pemetrexed (Alimta), Hydroxyurea Anthracyclines - doxorubicin (Adriamycin), daunorubicin, idarubicin Other anti-tumor antibiotics - mitoxantrone, Mitomycin-C, bleomycin (lung damage, need PFT's) Topoisomerase inhibitors - topotecan, irinotecan, etoposide, mitoxantrone Mitotic inhibitors - vincristine (oncovin), paclitaxel (Taxol), docetaxel (Taxotere), vinblastine (Velban), vinorelbine (Navelbine), ixabepilone (Ixempra) Corticosteroids - Prednisone, Methylprednisolone (Solu-medrol), Dexamethasone (Decadron) (prevent N/V causes by chemo and prevent allergic reactions)

A 75-year-old man who weighs 80 kg presents to the emergency department with symptoms consistent with a stroke. A severe headache develops, followed by confusion and hemiparesis on his left side. He has a long-standing history of hypertension treated with an angiotensin-converting-enzyme (ACE) inhibitor. On examination, he appears euvolemic, with an elevated blood pressure of 150/100 mm Hg and normal serum electrolyte levels. What type and rate of maintenance intravenous fluid would you administer while the patient is receiving nothing by mouth before evaluation and management of a possible stroke?

An appropriate fluid would be 5% dextrose in a solution of 0.9% saline at a rate of 100 ml per hour. This patient appears to have an acute intracranial process suggestive of a stroke. He is at risk for increased intracranial pressure and cerebral edema. Patients with CNS processes are at high risk for hyponatremia due to the syndrome of inappropriate antidiuresis or cerebral salt wasting. In addition, hyponatremia can have extremely serious consequences in patients with CNS disease, since a small decrease in the serum sodium concentration could result in cytotoxic cerebral edema, thus aggravating vasogenic cerebral edema, which probably is already present. Administration of 0.9% saline would help to prevent the development of hyponatremia, but the patient's electrolyte levels would need to be monitored closely, since the plasma sodium concentration can decrease in patients with CNS disease who receive 0.9% saline. The addition of 3% saline may be necessary to maintain a serum sodium concentration above 140 mmol per liter.

An 87-year-old man who weighs 70 kg and who has a left-lower-lobe pneumonia is admitted to the hospital from a nursing home. He has a history of myocardial infarctions and well-controlled congestive heart failure managed with an ACE inhibitor, a beta-blocker, a thiazide diuretic, and a sodium-restricted diet. He received 500 ml of 0.9% saline during transport to the hospital. His vital signs are stable, and there is no evidence of pulmonary congestion or peripheral edema on examination. Electrolyte levels are remarkable for a serum sodium concentration of 134 mmol per liter, potassium 3.2 mmol per liter, total carbon dioxide 31 mmol per liter, and creatinine 1.2 mg per deciliter (106 μmol per liter). Intravenous antibiotics are initiated. He is weak and has tachypnea and clinically significant coughing. There is a concern that he is too ill to be able to drink sufficient fluids. What type and rate of maintenance intravenous fluid would you administer?

An appropriate fluid would be 5% dextrose in a solution of 0.9% saline plus 20 mmol potassium chloride per liter at a rate of 60 ml per hour. This patient is at risk for both fluid overload and hyponatremia due to congestive heart failure. Therefore, the rate of intravenous fluids should be restricted. A rate of 60 ml per hour or approximately 1.5 liters per day is probably adequate to maintain good peripheral perfusion without causing fluid overload. This patient has a low serum sodium concentration at 134 mmol per liter, so hypotonic fluids should not be administered. Recent data suggest that sodium supplementation may be beneficial in patients with heart failure. Potassium should be added to the intravenous fluids, since the patient has hypokalemia, probably from the thiazide diuretic and poor nutrition. In this patient, the volume of intravenous medications needs to be accounted for, since it could contribute to both fluid overload and hyponatremia.

A 3 yo Caucasian female is brought to the office by her grandmother. Grandma notes that patient has been daytime potty trained for the last 6months, but recently has started having urinary accidents again. Grandma states that the child's urine has a pungent odor in her diapers when she first awakens in the morning. Denies any concerns of sexual abuse. Patient spends all the time with grandma and 5-year-old sister at home. Patient takes bubble baths nightly and has wanted to be more independent recently with toileting by herself during the day. ◦Allergies: NKDA ◦Medications: None ◦Vitals HT - 37", WT - 32lbs, BP 90/60mmHg, Temp 100F ◦PHYSICAL EXAM (pertinent positives): Abd - Neg CVA tenderness, Mild Suprapubic pain with deep palpation. Normoactive bowel sounds, GENITAL: No lesions noted Labs? Meds?

Any labs you would want to order in office? ◦U/A Dipstick as well as U/A culture and sensitivity RESULTS: ◦LEUKS 2+ ◦NITRATES + ◦All other readings on urine - neg What medication would you want to give? ◦Bactrim (>2 years old) 200mg/40mg per 5 mL ◦8mg/kg/day TMP every 12 hours for 3-10 days Max 320mg/kg/day ◦32lbs ◦32lbs/2.2kg = 14.54kg ◦14.54kg x 8mg = 116.36mg/day /2 (12 hours) = 58.18mg each dosage ◦7.2mL per dosage Any other counseling for grandma? ◦Showers instead of bath ◦Wipe front to back

A 27-year-old female presents to the office with history of migraines. Notes that she currently is taking butalbital-caffeine-apap (Fioricet) for migraines. States that the medication is not working as well for the migraines (8/10 pressure pain) and she is having to take the medication 3-4x per week to give her relief (3/10). +photophobia +phonophobia. Cool compresses on her neck improve the pain. Last CT scan 2 weeks ago when patient was seen in the ER and was negative for any masses, lesions, bleeding. Medications - Fioricet - unknown dosage Allergies - DNKA Vitals - WNL What would you want to do with the medication ?

Any labs? Change medication? If so, would you want to do something daily? What are our daily options? -TCA's: Amitriptyline, Nortryptiline - Anticonvulsants: Topamax - Antihypertensive: Propranolol What if the patient also has CAD or HTN? - Maybe a beta-blocker is a better option: Propranolol (Inderal), Metoprolol Tartrate (Toprol XL) -Can treat both HTN and Migraine - Stay away from "triptans" (Imitrex, Maxalt, Relpax) in pts with CAD as it is contraindicated What if the patient also has depression? - An antidepressant might be a better option - Treat for both depression and migraine: Amitriptyline (Elavil)

PA Practice Laws: supervising physician

Anytime a PA practices in any work setting as a physician assistant he/she must have a supervising physician, either a primary or an alternate supervising physician who will be responsible for their actions legally. •This applies to any employment setting, volunteer or humanitarian setting. 100 hours of CME is required for each renewal cycle of licensure (every 2 years), 10 of which must be in the specialty for which the PA is issued prescribing privileges

Can PAs in Florida obtain a DEA number?

As of January 02, 2017 the DEA accepts Florida applications for controlled substance prescribing. If you are approved, a DEA number will be provided. DEA licensure is required in order to prescribe a controlled substance. The cost of DEA licensure is $731.00 for a three (3)-year license

A 33-year-old male presents to the Emergency Department with severe muscle rigidity, mental status changes, elevated CBC, CPK & LFTs. Which of the following medications is most likely the cause of his symptoms? A. Sertraline (Zoloft) B. Haldol (Haloperidol) C. Bupropion (Wellbutrin) D. Amitriptyline (Elavil)

B. Haldol (Haloperidol) Neuroleptic Malignant Syndrome (NMS) is caused more often by typical and atypical antipsychotic medications

A 26-year-old female is taking Adderall for her ADHD. She comes to you to get treated for her GERD. What is the drug interactions between Adderall and Prilosec? A. Decreased serum levels of Adderall B. Increased serum levels of Adderall C. Decreased serum levels of Prilosec D. Increased serum levels of Prilosec

B. Increased serum levels of Adderall

Good Prescribing Practice: Strength of the drug

Be familiar with drugs and their various dosing strengths and dosage forms •When in doubt, use references or call pharmacy for help Weight - based dosing •Always convert patient weight to correct units (kg) Liquid medications •One product may be available in several concentrations •Indicate BOTH concentration and dose of medication •Example: Cephalexin suspension 125 mg/ 5 ml 1 teaspoon (5 ml) every 6 h Dispense # 200 ml

CC: " I woke up this morning feeling very dizzy" HPI: 42 y/o F presents to your clinic with severe dizziness. Pt. is a server states she worked a long shift yesterday went to bed and when awoke this am and sat up to get out of bed "the room was spinning. I felt like I was drunk but I didn't even drink." Pt. denies any recent trauma or injury. Pt. states worsens with position and turning of her head. Pt. denies any headache or ear pain/fullness. Pt admits to history of seasonal allergies. Has been using OTC Claritin and Nasonex as recommended. Denies any allergy symptoms currently. Pt. states dizziness comes and goes. PMH: PCOS, Seasonal Allergies Surg Hx: umbilical hernia repair Meds: Metformin 500mg PO BID, Loratadine 10 mg PO daily prn, Mometasone Furoate 50mcg/spray intranasal daily prn Allergies: Bactrim/Sulfa (Rash) Soc Hx: Single, denies drinking alcohol, smoking, or illicit drugs. Vitals BP 122/88mmHg HR 87bpm RR18bpm O2sat 99% RA Temp 98.4F Oral PE: - Pupils PERRLA, EOMI - Visual Acuity 20/20 OD,OS,OU with Rosenbaum card @3ft - CN II-XII grossly intact - Cerebellar function test intact - Dix-Hallpike reveals mild rotational nystagmus - MMSE Normal

BPPV Considerations: - Meclizine may cause tiredness in pt - Maneuvers & time is the best tx for BPPV - May need neuro referral pending on severity of vertigo - Benzos/Antiemetics not chosen due to side effects of sedation & pt is not c/o vomiting Can also give: Scopolamine (patch), 20 tablets Contraindications: none Adverse Rxns: Long-term adaptation/vestibular loss take for the least amt of time possible Safety/Monitoring: none

A 63 y/o man presents with a 3-month history of dizziness. His dizziness comes and goes, but usually lasts for about 10 to 15 sec. He notices that his dizziness is worse when he rolls over in bed or when he gets out of bed. At one time, he became very dizzy while trying to reach for an object on a high shelf. He does not have any N/V associated with it. When it occurs, it is severe, and he has tried to avoid sleeping on his left side. He does not have any hearing loss or tinnitus. He denies aural pressure and HAs. His PMH is otherwise unremarkable. He is not on any medications On PE, he is a healthy appearing 63 y/o man. His temperature is 98.8°F; pulse, 64 beats/min; and BP, 124/74 mmHg. There are no lesions or masses on his face or head. His voice is normal, and his speech is fluent. His facial nerve function is normal. His ear canals and tympanic membranes are normal appearing. His remaining head and neck examination is normal. The cranial nerve examination is normal. The remaining PE is normal. Dx?

BPPV Does this patient require pharmacotherapy? No, If symptoms persist and vestibular therapy fails, pharmacotherapy can be added If indicated, what medications could you give for sxs? -Meclizine, diphenhydramine (antihistamines), scpolamine - Benzodiazepines if antihistamines don't work - Odansetron (Zofran), Promethazine if symptoms include nausea/vomiting **more sedating**

CC: "My left eye is red and was stuck together this morning" HPI: 47-year-old female presents with left eye redness and itching. States woke up this am with her eye "matted shut" and she had to use warm compresses to remove crusts prior to leaving for work. Pt feels as though there is sand in her left eye and now her right eye is starting to "look red". Denies any sick contacts. Notes that she doesn't wear contacts. Pt states she had something similar like this in college. PMH: Migraines Meds: Fioricet PRN Allergies: NKDA Soc Hx: Married with two kids. Teacher. Pt denies any alcohol, tobacco, or drugs. VS BP 132/78mmHg RA sitting, HR 64bpm, RR16bpm, Temp 98.4F Oral PE: Visual Acuity OD 20/30 OS 20/40 OU 20/40 How do you treat?

Bacterial Conjunctivitis Tobramycin is the most cost effective and moxifloxacin is the most convenient - so, what would your patient prefer? What does their insurance cover?

CC: "I have a headache and my face hurts really bad." HPI: 21 y/o M presents complaining of facial pain/pressure and headache for 8 days. Pt admits to "yellowish thick snot" Pt. denies any recent injury or trauma. Pt. states he feels "facial pressure" worsening when he leans forward to tie his shoes. Denies any ear pain, sore throat, neck pain, vomiting or fever. Pt. has taken OTC Alka Seltzer Cold with no relief. PMH: ADHD, Seasonal Allergies Surg Hx: None Soc Hx: College student denies any drugs, beer socially, no tobacco Meds: Adderall 10mg PO BID Allergies:NKDA Vitals BP 138/76mmHg HR 100bpm RR 18bpm O2sat 100% RA Temp 99.6 F Oral Positive tenderness over frontal sinus region on palpation and percussion Transillumination reveals difficulty with light shining through and opaque appearance Would you give for allergies?

Bacterial RhinoSinusitis

CC: "I have a headache and my face hurts really bad." HPI: 21 y/o M presents complaining of facial pain/pressure and headache for 8 days. Pt admits to "yellowish thick snot" Pt. denies any recent injury or trauma. Pt. states he feels "facial pressure" worsening when he leans forward to tie his shoes. Denies any ear pain, sore throat, neck pain, vomiting or fever. Pt. has taken OTC Alka Seltzer Cold with no relief. PMH: ADHD, Seasonal Allergies Surg Hx: None Soc Hx: College student denies any drugs, beer socially, no tobacco Meds: Adderall 10mg PO BID Allergies:NKDA Vitals BP 138/76mmHg HR 100bpm RR 18bpm O2sat 100% RA Temp 99.6 F Oral Positive tenderness over frontal sinus region on palpation and percussion Transillumination reveals difficulty with light shining through and opaque appearance Would you give for congestion?

Bacterial RhinoSinusitis

An otherwise healthy 19-year-old man is brought to the ED by his roommate who states that he has "not been acting right" for the past 24 hrs. Per the roommate, the pt had complained of a HA 2 days prior to arrival and has been progressively somnolent and confused since then. The pt has no PMH and does not take any medications. His roommate states that the pt is a college student who does not use any illegal drugs and occasionally drinks alcohol. ROS is positive for HA and AMS as stated above as well as a tactile fever for the past 2 days. Additional ROS is unobtainable as the pt is unable to answer any questions. On PE the pt is noted to be febrile to 101°F orally, with a HR of 120 beats/min, BP of 114/69mmHg left arm sitting, and a RR of 20 breaths/min. His oxygen sat is 98% on RA. The head and neck examination are significant for dry mucous membranes and nuchal rigidity. His cardiopulmonary examination is within normal limits apart from tachycardia. The abdomen is soft and non-tender. His skin is noted to be warm and well perfused without any rash. The neurologic examination is significant for an AMS with a GCS of 10 (eyes open to voice [3], patient moans to painful stimuli [2], and localizes painful stimuli [5]). The motor examination is symmetric, and the patient appears to be sensate in all extremities. His reflexes are 2+ bilaterally throughout the upper and lower extremities with down going toes. Laboratory studies reveal a leukocytosis of 24,000/mm3 with a left shift and are otherwise unremarkable. A CT scan is completed which shows no mass, shift, bleed, or edema. What is the most likely diagnosis?

Bacterial meningitis What diagnostic test is the study of choice? - Lumbar puncture & CSF culture What is the most appropriate tx of this condition? - Initial therapy should include a 3rd generation cephalosporin in a sufficient dose to achieve adequate CSF concentration (ROCEPHIN 2g IV q12hr for 7-14 days) - Ceftriaxone or cefotaxime at a dose of 2 g is typically recommended in the US - Increasing worldwide prevalence of drug resistant Streptococcus pneumoniae, most authorities now recommend a dose of vancomycin along with the 3rd-generation cephalosporin until a resistance profile can be obtained Ceftriaxone - 2 g intravenously (IV) every 12 hrs PLUS vancomycin - 15 to 20 mg/kg IV every 8 to 12 hrs PLUS Dexamethasone - IV: 0.15 mg/kg/dose every 6 hrs for 4 days Additional Txs? Additional tx: Adjunctive dexamethasone (0.15mg/kg IV every 6h) either before or with the 1st does of antibiotics. Family members and close contacts -, Current CDC guidelines recommend antibiotic prophylaxis (typically with a fluoroquinolone or rifampin) for close contacts of patient with meningitis due to Neisseria meningitides Begin antibiotics as soon as possible if bacterial meningitis is suspected - Time to antibiotic administration is correlated with outcome Control fever with Tylenol If viral meningitis is suspected, give acyclovir 10 mg/kg/day IV divided TID x 10 days Consider steroids with first dose prior to starting antibiotics: Decadron 0.4 mg/kg q 12hrs for 4 doses Bacterial Meningitis Empiric Therapy

An 18-year-old female is on valproic acid for bipolar disorder. Which of the following must be documented in the patient's chart because it is a Black Box Warning? A. Can cause thrombocytopenia B. Increases suicide rate in patients under 24-years-old C. Can cause fetal neural tube defects D. Can cause skin discoloration and pruritus

C. Can cause fetal neural tube defects

A 28-year-old female presents to the Emergency Department with hypertensive crisis after the Disney Wine Feast. She was drinking wine, eating several different types of aged cheese, and liverwurst. Which of the following medications was she probably taking? A. SSRI B. SNRI C. MAOI D. TCA

C. MAOI MAOI plus Tyramine rich foods will cause a hypertensive crisis

A 65-year-old Caucasian hypertensive obese male patient presents to your Urgent Care with complaints of intermittent palpitations for 2 days. During the conversation, the patient indicates that he is under a lot of stress at his job. His blood pressure is 175/102 mmHg RA sitting, pulse is 118 irregular. Physical Exam reveals clear lung fields with a normal S1, S2 without murmurs, rubs or gallops. There is no pedal edema. ECG is ordered and on the next slide. PMHx: HTN MEDS: None - Hasn't seen a provider in years ALLERGIES: NKDA SOC HX: No alcohol, tobacco or illicit drug usage CHADS2 score= 2 EKG shows Afib Pt presents to the ER for complaints noted above. Daughter advises you that he was seen in the UC this am and was told to come over to be seen. Pt notes he is still having palpitations and bp in the ER is 182/108mmHg LA sitting and Pulse 116bpm irregular. Labs and Echo ordered Labs including cardiac enzymes are normal. Echo reveals mild LVH, no LAE and no mural thrombus or valvular abnormalities. ECG repeated in the ER and there is no change from the ECG that was sent by the UC You receive a consultation regarding current patient. After reviewing the chart, labs, echo and medication you visit the patient. You know diltiazem is poor at bp control, so you want to add a beta-blocker to patient's medications. You add by mouth: - Metoprolol Tartrate 25mg twice daily You admit the patient to the telemedicine floor and continue to monitor him throughout his hospital stay During his stay, you know that pt is at risk for blood clots, so you add an anticoagulant to his medication **see next slide** After 48 hrs, pt is back to NSR and his rate is controlled. You write him prescriptions and discharge him from the hospital. Patient is advised to follow up with his PCP within 2 weeks of discharge and will be following with your office in 3-6 months. What would be the next step as a Cardio PA-C?

CHA2DS2-VASc is calculated and determined to be a risk of 2 A score of 2 equals needing anticoagulation It is recommended to start on a NOAC - New Oral Anti-Coagulant Pt is 65 and has Medicare Rivaroxaban (Xarelto) prescribed

CC - "I'm short of breath x 1 week" A 72 yo Hispanic male presents to your office with worsening of his SOB over the past week. Notes he has a history of COPD and has been taking inhalers for the last 5 years. States that his breathing is usually under control ad he will have to take antibiotics maybe 1x a year, but this is his 3rd time within the last 12 months he has had these symptoms. States he is taking his inhalers daily as prescribed and over the last few weeks his inhalers are not helping with his dyspnea. Pt states he is now coughing up green phlegm all day long vs his normal clear/white phlegm just in the AM. Denies any sick contacts. PMHx - COPD x 5 years MEDS - Spiriva 18mcg - inhale 1 capsule daily - compliant SHx - Quit smoking 5 years ago (30/pack year), no alcohol, vaping or illicit drugs ALLERGIES - None VITALS - Normal - O2 94% RA Sitting Tx besides Prednisone?

COPD with acute exacerbation

56-year-old Caucasian male with hx of hypertension comes to your office for shortness of breath. He notes that he used to become short of breath with walking around his apt complex, so he stopped exercising. He is now getting short of breath walking from his parking spot into the grocery store. Notes that he will get short of breath at work sometimes and so he just has the young guys do the work for him. Shortness of breath improves with rest. Denies any chest pain, palpitations, dizziness or any shortness of breath with just sitting. Admits to some swelling in his feet. Allergies: None Medications: Lisinopril 10mg daily (compliant) PMHx: None - last seen by PCP 1 years ago. Colonoscopy 6 years ago, labs 1 year ago SHx: +tobacco use (20 pack years), will drink an occ beer on the weekend. Denies any illicit drug usage or vaping Vitals: BP 138/90mmHg RA sitting, HR 82, RR 14, O2 98% RA, Temp 98.6F Oral Physical Exam: 2+ pedal edema b/l - otherwise, unremarkable CXR: Mild pulmonary congestion noted due to pleural effusion, but nothing significant EKG: NSR - No ST elevation or ST depression noted Labs: CBC, CMP, Lipid, HgbA1c, BNP, TSH - CBC - WNL - CMP - GFR 65mg/mmol - otherwise, WNL - Lipid - TC 247, LDL 163, HDL 28, Trigs 400 - HgbA1c - 5.7% (WNL) - BNP - 350pg/mL - TSH - 3.2 (WNL) - Echocardiogram - EF 55 Tx besides Lisinopril?

CHF Bring pt back in to check lab - specifically potassium in 1 week due to daily Lasix Pt to eat bananas/tomatoes and other potassium rich food to help prevent hypokalemia Referral to cardio to determine if a beta blocker is necessary **avoid usage in acute phase due to increased mortality** Increase Lisinopril to 20mg daily due to BP being elevated on the 10mg - Due to normal EF - max out ACEI before starting on beta blocker Referral to nephro due to low GFR and to aid with kidney protection during treatment of CHF

56-year-old Caucasian male with hx of hypertension comes to your office for shortness of breath. He notes that he used to become short of breath with walking around his apt complex, so he stopped exercising. He is now getting short of breath walking from his parking spot into the grocery store. Notes that he will get short of breath at work sometimes and so he just has the young guys do the work for him. Shortness of breath improves with rest. Denies any chest pain, palpitations, dizziness or any shortness of breath with just sitting. Admits to some swelling in his feet. Allergies: None Medications: Lisinopril 10mg daily (compliant) PMHx: None - last seen by PCP 1 years ago. Colonoscopy 6 years ago, labs 1 year ago SHx: +tobacco use (20 pack years), will drink an occ beer on the weekend. Denies any illicit drug usage or vaping Vitals: BP 138/90mmHg RA sitting, HR 82, RR 14, O2 98% RA, Temp 98.6F Oral Physical Exam: 2+ pedal edema b/l - otherwise, unremarkable CXR: Mild pulmonary congestion noted due to pleural effusion, but nothing significant EKG: NSR - No ST elevation or ST depression noted Labs: CBC, CMP, Lipid, HgbA1c, BNP, TSH - CBC - WNL - CMP - GFR 65mg/mmol - otherwise, WNL - Lipid - TC 247, LDL 163, HDL 28, Trigs 400 - HgbA1c - 5.7% (WNL) - BNP - 350pg/mL - TSH - 3.2 (WNL) - Echocardiogram - EF 55 Dx? Tx?

CHF What would you like to start patient on/if anything? Diuretic - due to edema - Usually a loop diuretic - Monitor K+!!!! ACE/ARB - already taking, check dosage - Lisinopril - Affordable and kidney protective Beta Blocker (possibly)

CC "I am short of breath and have a cough" A 65 yo Asian male comes to the office stating he has SOB and cough. Pt notes that this has been going on for many months. Notes he is now having difficulty breathing while sitting and watching tv. States about 3 yrs ago was diagnosed with asthma and given an albuterol inhaler. Notes it worked for his shortness of breath in the beginning, but now he must utilize it daily, but it doesn't seem to help. Currently smokes 1 pack daily. Pt notes he has no plan on quitting soon. PMHx - HTN, ASTHMA SOCHx - No alcohol or Illicit drug usage, 47 pack year smoking hx. MEDS - Hydrochlorothiazide 25mg daily (compliant), Ventolin HFA 90mcg 2 puffs every 4-6 hours as needed for shortness of breath VITALS - BP 114/68mmHg LA Sitting, HR 74bpm, RR 12 bpm, O2 94% RA Cardiac - S1/S2 heard, RRR, No murmurs, rubs or gallops Respiratory - Increased AP diameter of chest note upon inspection, Increased resonance to percussion, Prolonged expiration noted, no wheezing, rhonchi or rales Musculoskeletal - normal, no clubbing noted upon inspection of digits REFERRALS Pt referral to get Spirometry completed - awaiting results Referral to pulmonologist - pending spirometry results Dx? Tx?

COPD What would you want to start him on? An inhaler specific for COPD (in the morning) Spiriva? Combivent?

CC - "I'm short of breath x 1 week" A 72 yo Hispanic male presents to your office with worsening of his SOB over the past week. Notes he has a history of COPD and has been taking inhalers for the last 5 years. States that his breathing is usually under control ad he will have to take antibiotics maybe 1x a year, but this is his 3rd time within the last 12 months he has had these symptoms. States he is taking his inhalers daily as prescribed and over the last few weeks his inhalers are not helping with his dyspnea. Pt states he is now coughing up green phlegm all day long vs his normal clear/white phlegm just in the AM. Denies any sick contacts. PMHx - COPD x 5 years MEDS - Spiriva 18mcg - inhale 1 capsule daily - compliant SHx - Quit smoking 5 years ago (30/pack year), no alcohol, vaping or illicit drugs ALLERGIES - None VITALS - Normal - O2 94% RA Sitting Tx besides Prednisone or Azithromycin 250 mg?

COPD with acute exacerbation

CC - "I'm short of breath x 1 week" A 72 yo Hispanic male presents to your office with worsening of his SOB over the past week. Notes he has a history of COPD and has been taking inhalers for the last 5 years. States that his breathing is usually under control ad he will have to take antibiotics maybe 1x a year, but this is his 3rd time within the last 12 months he has had these symptoms. States he is taking his inhalers daily as prescribed and over the last few weeks his inhalers are not helping with his dyspnea. Pt states he is now coughing up green phlegm all day long vs his normal clear/white phlegm just in the AM. Denies any sick contacts. PMHx - COPD x 5 years MEDS - Spiriva 18mcg - inhale 1 capsule daily - compliant SHx - Quit smoking 5 years ago (30/pack year), no alcohol, vaping or illicit drugs ALLERGIES - None VITALS - Normal - O2 94% RA Sitting Dx? What to do?

COPD with acute exacerbation What do you want to prescribe, if anything for pt? Steroids and Antibiotics due to sxs presented With COPD - what 3 sxs should be noted for pt to be written for antibiotics? increased dyspnea, increased sputum volume, or increased sputum purulence Anything else you would want to do? - Does pt have a pulmonologist? - Possibly refer for updated PFT/Spirometry if unable to complete in your office

24-year-old Caucasian female here today with complaints of Headaches. Note that she has had headaches for the previous 12-years. Pt missed school in the past due to the headaches. Notes they happen 1-2x per month and will be worse if it correlates with her menses. It is a constant pressure pain over her right eye. Notes that flashes of lights and zigzag lines precede the headache. +photophobia +N/V. Sitting in a dark room improves the pain (3/10) and drinking red wine can worsen her pain (8/10). Denies taking any medication for pain. Notes that headaches will last 4-6 hours and feels tired/listless for the next 24 hours. She wants to know what else can be done PE: Her general and neurological examinations are normal Vitals all WNL What is her most likely dx?

Common Migraine What is the first line therapy? Due to patient only having a headache 1-2x per month. Will counsel with patient about trying naproxen as a first-line treatment other tx: Oral: Sumatriptan 85 mg/naproxen 500 mg What are some pt education you should share? Discuss with patient lifestyle measures that can help with migraine. Such as good sleep hygiene, routine meal schedules, regular exercise, and managing migraine triggers, no caffeine, sleep, take NSAID before onset of menses What drugs can be given as prophylactic treatment? ●Beta-Blockers (Metoprolol, propranolol and timolol): Immediate release (metoprolol tartrate) : Oral: Initial: 25 mg twice daily; ●Antidepressants (Amitriptyline and Venlafaxine) ●Anticonvulsants (Valproate and Topiramate) ●CGRP Antagonist (Erenumab and Fremaanezumab)

CC - "I have a fever and cough x 2 weeks" A 42 yo Asian female presents with fever and cough x 2 weeks. Notes that she checks her temp at home, and it will range from 101-103F. States that the cough started 2 weeks ago and that the fever has only been for the last 2-3 days. Pt has been using ibuprofen and Tylenol to keep the fever down. States that she is having difficulty breathing and it is worsening over the last few days. States is now noticing shortness of breath when she climbs her stairs at home. Many co-workers at work have been sick and have been missing work. Pt has green phlegm, no sore throat, ear pain, headaches. Admits to some chest pain with coughing but denies any chest pain otherwise. MEDS - Tylenol/Ibuprofen OTC - as needed for fever ALLERGIES - PCN (angioedema) FHX - Unknown SHx - no tobacco, vaping, alcohol, illicit drugs Vitals - O2 96% RA in office, Temp 100.4F Oral (Last Tylenol dosage 1 hour ago), Pulse 96bpm, Other vitals normal PE - Rhonchi noted in LLL 3 view CXR stat completed - LLL consolidation noted Dx? Referrals?

Community Acquired Pneumonia (Bacterial PNA - CAP) Referrals? Education? - No referrals needed at this time - Pt to go to ER/call 911 if breathing worsens - Currently stable and can be sent home on antibiotics - NO PCN or CEPHALOSPORIN due to angioedema caused by PCN

A 23 y/o graduate student was studying late at night for an examination. He recalls studying, but his next memory is being on the floor and aching throughout his body. He was incontinent of urine but not stool and felt slightly confused. No one was with him, and he did not know what to do. He called his mother, who recommended that he go to the local ER. The student was too busy and decided not to go to the hospital. He called the student health center the next day, and their physicians examined him. PE His VS were normal Neurological exam, including motor and sensory evaluation, reflexes, and cranial nerve function were normal. His physical exam was normal. He wants to know what to do Dx?

Complex Partial Seizure In Primary Care - what would you do? - Refer to neurology for EEG and further assessment and possible medication - Order CT/MRI of brain to assess if concerned about an anatomical cause - Recommended to start an anti-seizure medication if >2 unprovoked seizures (low blood sugars can also cause seizures) If in Neuro or if you chose to start patient on medication, what is your first line therapy in this diagnosis? - Lamotrigine can be utilized for focal or generalized seizures - 30 tabs are $4.50 at Wal-mart (affordable) If the pt has a recurrence while on medications, what's the first question you need to ask? - Is patient compliant with medication? - Check medication levels in blood What if patient was a female? Which medication is NOT preferred in women of childbearing age? Valproic Acid (Depakene) Why?? - It can cause neural tube defects - If female pt is taking this med - please make sure she is aware and that birth control is being utilized

CC: "My stomach hurts" HPI - 52-year-old female presents to your office for abdominal pain. States her last BM was 7 days ago. Notes that she is having decreased appetite over the last few days. State pain is a constant, bloating pain (3/10). Denies anything making the pain worse or better. Has tried Magcitrate and Miralax x1 with no relief. Pt has hx of hypothyroid, does not exercise. States she eats vegetables daily and drinks around 32oz water daily. ■PMHx: hypothyroidism, IDA, ■MEDS: Levothyroxine 50 mg QD, Ferrous gluconate 324 mg TID - compliant ■ALLERGIES - None ■VITALS: 120/78mmHg LA sitting, HR 78bpm, RR 14bpm, Temp 98.6F Oral ■LABS: TSH is high, Free T4 is low, Ferritin is high

Constipation Select appropriate medication -Colace written for patient to take Select non-pharmacologic therapy/interventions -Colace is non-pharmacologic but can be written RX. Patient can also try a fleet enema. Counsel about increasing exercise, water intake and vegetables. Discuss with patient that iron can cause constipation. Pt can get iron from food intake - adjust Thyroid medication as needed What are the contraindications or side effects of this medication? -Laxatives can sometimes cause stomach cramping Describe the cost of the medication - Medication is affordable and comes generic

Pt returns 2 weeks later. BP has improved again, and patient has lost 5lbs. He is feeling better overall and continues to monitor his bp. Although not perfect at home he has noticed an improvement. Vitals: BP 138/78mmHg LA sitting What should we do?

Cont to monitor? Add/adjust medications? I would monitor at this point. With weight loss and continued improvement, give patient another 2 weeks to continue to monitor at home. If no improvement or worsening of bp - we would add another medication. What would be a good option, if needed? If patient needed another medication, which would be a good choice? CCB

CC: "My eye hurts" HPI: 27-year-old male presents to clinic with eye pain. Notes that it feels as though something is in his eye. Pain is a constant achy pain (6/10). Light will worsen the pain. Hx of wearing contacts. States went to a party 2 nights ago and slept in his contacts. Since then he has had the redness and feeling of sand in his eye. PMH: Depression Meds: Zoloft 50mg PO Daily Allergies: PCN (Rash) Soc Hx: College Student. Single. Admits to drinking alcohol socially (beer), uses E-cig, but denies any illicit drug use. VS BP112/76mmHg LA sitting HR 71bpm RR14bpm Temp 98.6F Oral PE: - Visual Acuity OD 20/50 OS20/30 OU 20/40 - Pupils PERRLA, EOMI - Fluorescein Stain (seen Above) - IOP 20mmHg - Lids everted no FB or lesions noted

Contact Conjunctivitis Why did I write as "use as directed? Because these are the instructions and it would be difficult to fit on the tiny box - 2 drops in affected eye x 15 min for 6 hours, then 2 drops every 30 min x 1 days, then 2 drops every 1-hour x 1 day, then 2 drops every 4 hours x 14 days

CC: "My left eye is red and was stuck together this morning" HPI: 47-year-old female presents with left eye redness and itching. States woke up this am with her eye "matted shut" and she had to use warm compresses to remove crusts prior to leaving for work. Pt feels as though there is sand in her left eye and now her right eye is starting to "look red". Denies any sick contacts. Notes that she doesn't wear contacts. Pt states she had something similar like this in college. PMH: Migraines Meds: Fioricet PRN Allergies: NKDA Soc Hx: Married with two kids. Teacher. Pt denies any alcohol, tobacco, or drugs. VS BP 132/78mmHg RA sitting, HR 64bpm, RR16bpm, Temp 98.4F Oral PE: Visual Acuity OD 20/30 OS 20/40 OU 20/40 Besides Tobramycin ?

Contraindications: None Adverse Reactions: If no improvement - follow up with pcp/optometrist Safety/Monitoring: nothing specific

CC: "My left eye is red and was stuck together this morning" HPI: 47-year-old female presents with left eye redness and itching. States woke up this am with her eye "matted shut" and she had to use warm compresses to remove crusts prior to leaving for work. Pt feels as though there is sand in her left eye and now her right eye is starting to "look red". Denies any sick contacts. Notes that she doesn't wear contacts. Pt states she had something similar like this in college. PMH: Migraines Meds: Fioricet PRN Allergies: NKDA Soc Hx: Married with two kids. Teacher. Pt denies any alcohol, tobacco, or drugs. VS BP 132/78mmHg RA sitting, HR 64bpm, RR16bpm, Temp 98.4F Oral PE: Visual Acuity OD 20/30 OS 20/40 OU 20/40 Tobramycin ?

Contraindications: none Adverse Reactions: if no improvement- f/u with pcp/optometrist Safety/Monitoring: nothing specific

Controlled Substances: Schedules

Controlled drugs are divided into Schedules according to abuse potential •Schedule I - Highest abuse risk. No safe medical use in U.S. Examples: heroin, marijuana, LSD, PCP, and crack cocaine. •Schedule II - High abuse risk but have safe and accepted medical use. Examples: morphine, hydromorphone, methylphenidate, dextroamphetamine. •Acetaminophen/Codeine (Tylenol #3), acetaminophen/hydrocodone (Vicodin), acetaminophen/oxycodone (Percocet). •Schedule III - Abuse risk less than II and safe and accepted medical use. Examples: Androgel, Suboxone, Ketamine •Schedule IV - Abuse risk less than III and safe and accepted medical use. Examples: diazepam (Valium), alprazolam (Xanax) , Tramadol (Ultram) •Schedule V - Abuse risk less than IV and safe and accepted medical use. Mainly consist of preparations containing limited quantities of certain stimulant and narcotic drugs for antitussive and antidiarrheal purposes. Examples: Cheratussin, Hycotuss, Lomotil

A prescription drug whose use and distribution is tightly controlled because of its abuse potential or risk

Controlled or scheduled drug Controlled drugs are classified into schedules •Schedules I, II, III, IV, and V Prescriptions for controlled substances have additional requirements by law

Encourage and Educate

Many times people "stop moving" when they have pain and that can "hurt" them more Educate patients that there is a difference between 'hurt' and 'harm' when you have chronic pain. Encourage patients to get moving even if slowly •Stretching •Swimming •Walking - just 1 extra step each day

Controlled Substances

Definition - a prescription drug whose use and distribution is tightly controlled because of its abuse potential or risk Regulation is more strict Federal law and State law regulate the storage, use, and disposal of controlled substances

A 28-year-old woman who weighs 65 kg is admitted through the emergency department with a diagnosis of acute appendicitis. Given a history of poor oral intake, she receives a 1-liter bolus of 0.9% saline in the emergency department. Vital signs and serum electrolyte levels are otherwise normal at presentation. What type and rate of maintenance intravenous fluid would you administer while the patient is receiving nothing by mouth both before surgery and postoperatively? •D5NS 125 mL/hr IV Postoperatively: doesn't change

D5NS 125 mL/hr IV Postoperatively: doesn't change An appropriate fluid would be 5% dextrose in a solution of 0.9% saline at a rate of 100 ml per hour. Perioperatively, patients have numerous stimuli for AVP production, including pain, stress, nausea, narcotics administered for pain management, and hypovolemia. In addition, women in their reproductive years are at increased risk for hyponatremic encephalopathy. Administration of 0.9% saline is suitable to maintain the extracellular volume and will reduce the risk of hospital-acquired hyponatremia.

Other Uses for Fluids

DKA - two bag method •D10NS w/ 20 mEq KPhos and 20 mEq KCl •NS w/ 20 mEq KPhos and 20 mEq KCl •Substitute KCl for K acetate if pH < 6.9 •Bags run according to trend in blood sugar: Dropping too fast can lead to cerebral edema Acidosis •D5W w/ 150 mEq sodium bicarbonate: May need sodium acetate during shortage 3% NaCl - 513 mEq/L •Used to draw water out of extracellular space •Cerebral edema •Corrects hyponatremia Albumin •Used to increase oncotic pressure in patients with diminished protein stores •5% - dosed the same as a fluid bolus (20 mL/kg) •25% - used in fluid restricted pts - 0.5-1 g/kg: Renal pts commonly

A 67 y/o woman is brought into your office by her daughter. The daughter states her memory has been getting worse over the last 3 years. Initially, she had problems remembering recent events and people's names and tended to dwell in the past. She got lost several times while driving, most recently in a familiar neighborhood. She has stopped cooking because she can no longer work her electric oven. Sometimes her words don't make sense. Her social graces have remained preserved, however, and she is still quite pleasant to be around, although she tends to interact socially less and less. She still walks around the block every day, and her basic gait and coordination seem quite normal. PE - Pt was mildly tachycardic. She was inattentive and had difficulty keeping on task. Her speech revealed numerous paraphasic errors but was otherwise fluent. Her neurologic exam was otherwise unremarkable. - MMSE in office - 15/30 What is your most likely diagnosis?

Dementia/Alzheimer's (early onset?) What drugs would you prescribe for this patient? - Acetylcholinerase inhibitors or NMDA (memetaine) - Namenda, Aricept - Exelon patch - if patient has difficulty swallowing/remembering tablet What are some major side effects of the above listed medications? Labs needed to be followed? - Somnolence, confusion, N/V/D - Monitor LFTs with Aricept Counseling - Speak with daughter (family member) regarding what care the patient has at home. Does the patient live alone? Does she have access to a vehicle? Discuss safety at home, locks on doors, driving privileges. - Refer pt to neuropsychiatrist if available. Give daughter info about resources in the community for pts with dementia and family members of these pts. - Discuss realistic goals for pt and daughter. Long term care options as well.

CC: " I'm throwing up and have diarrhea" HPI: A 62-year-old female comes into your office. She states that for the last 3 days has been having watery diarrhea 8x daily at least. Denies any melena or hematochezia. Notes that she has crampy abdominal pain that will worsen prior to bowel movement (7/10) and improve with defecation (2/10). Notes she has vomited 3x since she woke up this am. Last time she had diarrhea and vomiting was this morning. She is drinking flat ginger ale and eating chicken soup. Pt states it will immediately come back up. States her grandson was visiting this weekend and had some vomiting. States is feeling slightly better today, but overall still feels horrible. ■PMHx: HTN, hyperlipidemia ■MEDS: HCTZ 25 mg daily, Atorvastatin 10 mg at night ■ALLERGIES: None ■VITALS: BP 112/78mmHg LA lying flat, HR 103bpm, RR 17bpm, Oral 99.4F Oral ■LABS: WBCs 12,000 otherwise normal ■PE: orthostatic hypotension, tachycardia and mildly febrile

Diarrhea Select appropriate medication -Promethazine written for nausea Select non-pharmacologic therapy/interventions - Pt should continue with bland food. Add Pedialyte instead of ginger ale. Pt can use OTC Imodium as needed to aid with diarrhea. Counsel patient on symptomatic treatment. What are the contraindications or side effects of this medication? - Can cause fatigue Describe the cost of the medication. - Affordable - 12 tablets is $6 at publix and wal-mart

If you prescribe opiates...

Document! Document! Document! Document OPPQRST, ChLORIDEPP with EVERY visit Document the change in pain once patient is taking medication - especially with chronic pain Make patients aware of side effects/interactions Pain medication + alcohol or benzodiazepines Drowsiness and Constipation Have a pain contract with the patient Urine Drug screening can be completed To make sure patient isn't taking anything else BUT Also to make sure pt is taking prescribed medication

A 65-year-old Caucasian hypertensive obese male patient presents to your Urgent Care with complaints of intermittent palpitations for 2 days. During the conversation, the patient indicates that he is under a lot of stress at his job. His blood pressure is 175/102 mmHg RA sitting, pulse is 118 irregular. Physical Exam reveals clear lung fields with a normal S1, S2 without murmurs, rubs or gallops. There is no pedal edema. ECG is ordered and on the next slide. PMHx: HTN MEDS: None - Hasn't seen a provider in years ALLERGIES: NKDA SOC HX: No alcohol, tobacco or illicit drug usage CHADS2 score= 2 EKG shows Afib Pt presents to the ER for complaints noted above. Daughter advises you that he was seen in the UC this am and was told to come over to be seen. Pt notes he is still having palpitations and bp in the ER is 182/108mmHg LA sitting and Pulse 116bpm irregular. Labs and Echo ordered Labs including cardiac enzymes are normal. Echo reveals mild LVH, no LAE and no mural thrombus or valvular abnormalities. ECG repeated in the ER and there is no change from the ECG that was sent by the UC What would be the next step as a ER PA-C?

Due to stability of patient, no cardioversion needed at this time BUT...we need to get his BP down and his HR controlled: Order IV diltiazem for the a fib - 10mg bolus + 5mg/hr gtt, titrated to goal of HR 60-90 Add an order for blood pressure - IV Hydralazine 5mg as needed SBP>160 and SBP >100 Call cardiology to make a consult on this case and admit pt for at least 24 hour observation

CC: " I have horrible period cramps" x 24 hrs HPI: 21 year old female presents to the ER with severe menstrual cramps x 24 hours. This has happened in the past but has become more severe. No dysuria. +N/V, no diarrhea, no fever. Vomited x 2 today, hasn't drank or eaten all day. No radiation of pain, nothing makes it better or worse. Tried Tylenol, usually works but not this time. LMP 4 weeks ago. Not sexually active. •PMH: Seasonal Allergies •Surg Hx: umbilical hernia repair •Meds: cetirizine 5mg PO daily, Tylenol 1 gram PO 2 hours ago •Allergies: Bactrim/Sulfa (Rash) •Soc Hx: Single, denies drinking alcohol, smoking, or illicit drugs. •VS BP 96/64 HR 110 RR18 O2sat 99% RA Temp 98.4 •Abdomen: Nondistended, NABS, Mild suprapubic tenderness, no LLQ/RLQ tenderness, no rebound, no guarding •Pelvic Exam: Active bleeding from cervix with small clots (< 1cm diameter) in vaginal canal, no CMT, adnexal tenderness bilaterally •U/A: Trace LE, Neg Nitrite, 5-10 WBC, 5-10 Epith Cells •Urine hCG: Neg •Wet Prep: Negative •US Pelvis: Thin, echogenic endometrium, consistent with menstruation. No ovarian masses or abnormalities. How do you treat?

Dysmenorrhea In ER: toradol, fluids NSAIDS Ibuprofen 600mg q 6 hours prn menstrual cramps or ibuprefon 800 mg table. Take one tablet every 8 hours by mouth for dymenorhee as needed (15 tablets) Birth Control •Lo Loestrin 1 tab po daily, take as directed #3, 2 refills or no refills CI'd in Women over 35 who smoke or have hx of complex migraines with prolonged aura, due to increase risk of DVTs and stroke

Controlled Substances - Good Prescribing Practice

For III - V prescription drugs Quantity must be written out numerically and as a word •Example: Dispense #20 (twenty) NO REFILLS

CC: " I have heartburn x 4 months" HPI - A 58-year-old AA male returns to the clinic with complaints of having substernal chest pain after earing. Notes that prior would only happen when he ate spicy food (1-2x per month) but is now having heartburn with most foods and occurring 4-5x per week. Has been worsening over the last 4 months. Patient is eating whatever he wants, drinks 1 beer in the evenings and doesn't try to limit his intake. Has tried Tums with minimal relief and has tried OTC ranitidine (Zantac) with some relief but has stopped taking it due to recent news articles stating ranitidine (Zantac) can cause cancer. PMH: HTN 12 yrs/CKD 2 yrs/ Type 2 DM 5 years Meds: Amlodipine 5mg qd, Glyburide 5 mg bid, Aspirin 81 mg QD, Ibuprofen 400 mg prn pain Allergies: None Vitals: BP 118/72mmHg LA sitting, RR 16bpm, HR 86bpm, TEMP 98.2F Oral PE: ■CARDIO - RRR, no gallops, rubs, murmurs noted ■RESP - CTA B/L ■GI - Epigastric tenderness with deep palpation. NO guarding, NO rebound, normoactive bowel sounds. Meds?

GERD DOC: PPIs Select non-pharmacologic therapy/interventions. - Pt needs to cut back/out greasy/fried, acidic and spicy foods. Patient need to eat smaller meals, increase water consumption and decrease caffeine. Pt also needs to make sure that he is taking his ibuprofen with food and to use it sparingly What are the contraindications or side effects of this medication? - Long term usage (years) is currently being investigated regarding kidney issues. Notes: Can put him on Mobic (COX2 inhibitor) instead of ibuprofen Pharmacopeia states which dosage is OTC 'forms' section for each drug

CC: " I have stomach pain and cramping" 30-year-old female presents to your office with epigastric pain and tenderness. Notes that pain started after purchasing food from a local food-truck 2 weeks ago. Notes that she initially had some vomiting but that has now improved. Denies any n/v/d. Denies any constipation. Patient notes epigastric tenderness that is constant cramping pain (4/10). Worsens with eating/drinking (7/10). Admits to early satiety. Denies any melena or hematochezia. Pt has been taking famotidine (Pepcid) and Tums with minimal relief. ■PMHx: none ■MEDS: Famotidine 10 mg as needed ■ALLERGIES: None ■PE (Pertinent positives): +epigastric pain with light palpation, +guarding. No rebound, +hyperactive bowel sounds ■LABS: H pylori stool positive CC: " I have stomach pain and cramping" 30-year-old female presents to your office with epigastric pain and tenderness. Notes that pain started after purchasing food from a local food-truck 2 weeks ago. Notes that she initially had some vomiting but that has now improved. Denies any n/v/d. Denies any constipation. Patient notes epigastric tenderness that is constant cramping pain (4/10). Worsens with eating/drinking (7/10). Admits to early satiety. Denies any melena or hematochezia. Pt has been taking famotidine (Pepcid) and Tums with minimal relief. ■PMHx: none ■MEDS: Famotidine 10 mg as needed ■ALLERGIES: None ■PE (Pertinent positives): +epigastric pain with light palpation, +guarding. No rebound, +hyperactive bowel sounds ■LABS: H pylori stool positive What would you give with Clarithromycin?

H.Pylori

CC: " I have stomach pain and cramping" 30-year-old female presents to your office with epigastric pain and tenderness. Notes that pain started after purchasing food from a local food-truck 2 weeks ago. Notes that she initially had some vomiting but that has now improved. Denies any n/v/d. Denies any constipation. Patient notes epigastric tenderness that is constant cramping pain (4/10). Worsens with eating/drinking (7/10). Admits to early satiety. Denies any melena or hematochezia. Pt has been taking famotidine (Pepcid) and Tums with minimal relief. ■PMHx: none ■MEDS: Famotidine 10 mg as needed ■ALLERGIES: None ■PE (Pertinent positives): +epigastric pain with light palpation, +guarding. No rebound, +hyperactive bowel sounds ■LABS: H pylori stool positive

H.Pylori Select appropriate medication -Although the treatment of metronidazole, amoxicillin and a PPI is more cost effective, it only eradicates around 80% cases in the US. Bismuth quadruple therapy has an eradication rate around 91% However, verify that tetracycline is an option, otherwise, substitute with doxycycline. Select non-pharmacologic therapy/interventions -Eat foods that are kinder to the stomach - low fat/greasy/acidic. Small meals, more water What are the contraindications or side effects of this medication? -Make patient aware that Bismuth (pepto-bismol) can cause dark black stools. Pt has no contraindications for said medication because pt doesn't have any allergies. Describe the cost of the medication -Tetracycline is around $95, so without any insurance, doxycycline is a much better option

CC: "My husband has been confused and disoriented for the past 2 days" HPI: 70-year-old male with a history of cirrhosis is brought to the office by his wife. She notes that he is disoriented and confused for the last 2 days. Patient has been sleeping more throughout the day and will fall asleep mid-conversation. Recently released from the hospital where he was admitted for LE edema, worsening renal function and difficulty breathing. While in the hospital he had 3L fluid removed from his abdomen due to ascites. Was discharged to follow up with GI but that isn't until next week. Wife wants to know what can be done. ■PMHx: Cirrhosis 1 yr, CKD Stage IV, Osteomyelitis ■MEDS: Doxycycline, Torsemide, HCTZ, Oxycodone, Lactulose ■ALLERGIES - None ■VITALS - BP 120/68mmHg LA sitting, RR 14bpm, HR 68bpm, Temp 98.8F Oral ■LABS: Hgb 10.4 g/dL, Ammonia 176 mcg/dL

Hepatic encephalopathy Select appropriate medication - Rifaximin written - but note costs Select non-pharmacologic therapy/interventions - I would call patient's GI to see if he can get a sooner appt. If not, patient to go to hospital for evaluation due to increase in ammonia + symptoms. Rifaximin added at this time due to patient being on lactulose and continuing to have symptoms. What are the contraindications or side effects of this medication? - Side effects can be dizziness, fatigue, unfortunately the same symptoms as encephalopathy Describe the cost of the medication - Medication is $2500 without insurance. Therefore, unless patient's insurance will cover it, it will be very difficult to afford. Pt to see if there are any pharmaceutical aid with medications. Notes: Also consider Neomycin as a 2nd line tx

CC: "Rash on my ear that hurts" HPI: 50 y/o M presents with painful rash on his right ear x 1 week and radiates to his face. States he has had a burning pain for the last 10 days, but the rash showed up within the last 3-5 days. Pt. denies any new soaps, washes, or exposure to any new detergents. Rash has been spreading along ear. Pt. states the pain is a constant burning sensation (10/10). Touching the area makes it worse. Has tried OTC ibuprofen without any relief. Pt. denies any recent travel or fever. PMH: Hyperlipidemia, GERD, HTN Meds: Amlodipine 5mg PO daily, Protonix 40mg PO daily, Crestor 10 mg PO daily Surg Hx: Right Rotator cuff repair Soc Hx: Denies smoking, drinks 2 beers daily, no illicit drugs. Bus driver occupation. Vitals BP 140/94mmHg HR 99bpm RR 17bpm O2sat 98%RA Temp 98.6F Oral How to tx besides Valacyclovir & Prednisone?

Herpes Zoster Oticus

CC: "Rash on my ear that hurts" HPI: 50 y/o M presents with painful rash on his right ear x 1 week and radiates to his face. States he has had a burning pain for the last 10 days, but the rash showed up within the last 3-5 days. Pt. denies any new soaps, washes, or exposure to any new detergents. Rash has been spreading along ear. Pt. states the pain is a constant burning sensation (10/10). Touching the area makes it worse. Has tried OTC ibuprofen without any relief. Pt. denies any recent travel or fever. PMH: Hyperlipidemia, GERD, HTN Meds: Amlodipine 5mg PO daily, Protonix 40mg PO daily, Crestor 10 mg PO daily Surg Hx: Right Rotator cuff repair Soc Hx: Denies smoking, drinks 2 beers daily, no illicit drugs. Bus driver occupation. Vitals BP 140/94mmHg HR 99bpm RR 17bpm O2sat 98%RA Temp 98.6F Oral How to tx besides Valacyclovir?

Herpes Zoster Oticus

CC: "Rash on my ear that hurts" HPI: 50 y/o M presents with painful rash on his right ear x 1 week and radiates to his face. States he has had a burning pain for the last 10 days, but the rash showed up within the last 3-5 days. Pt. denies any new soaps, washes, or exposure to any new detergents. Rash has been spreading along ear. Pt. states the pain is a constant burning sensation (10/10). Touching the area makes it worse. Has tried OTC ibuprofen without any relief. Pt. denies any recent travel or fever. PMH: Hyperlipidemia, GERD, HTN Meds: Amlodipine 5mg PO daily, Protonix 40mg PO daily, Crestor 10 mg PO daily Surg Hx: Right Rotator cuff repair Soc Hx: Denies smoking, drinks 2 beers daily, no illicit drugs. Bus driver occupation. Vitals BP 140/94mmHg HR 99bpm RR 17bpm O2sat 98%RA Temp 98.6F Oral How to tx?

Herpes Zoster Oticus Any extra questions/referrals?: Pt admits to having chicken pox as a child Pt has not used any creams over the area No lesions noted on tip of nose - No emergent Ophthalmologist referral needed at this time - Counsel pt if lesions appear around eye or on tip of nose specifically to be seen by Ophthalmologist ASAP!

CC: "Rash on my ear that hurts" HPI: 50 y/o M presents with painful rash on his right ear x 1 week and radiates to his face. States he has had a burning pain for the last 10 days, but the rash showed up within the last 3-5 days. Pt. denies any new soaps, washes, or exposure to any new detergents. Rash has been spreading along ear. Pt. states the pain is a constant burning sensation (10/10). Touching the area makes it worse. Has tried OTC ibuprofen without any relief. Pt. denies any recent travel or fever. PMH: Hyperlipidemia, GERD, HTN Meds: Amlodipine 5mg PO daily, Protonix 40mg PO daily, Crestor 10 mg PO daily Surg Hx: Right Rotator cuff repair Soc Hx: Denies smoking, drinks 2 beers daily, no illicit drugs. Bus driver occupation. Vitals BP 140/94mmHg HR 99bpm RR 17bpm O2sat 98%RA Temp 98.6F Oral Valacyclovir vs Acyclovir?

Herpes Zoster Oticus Contraindications: None Adverse Rxns: N/V/D? Take with food. Counsel on steroids causing jittery-ness Safety/Monitoring: None

CC: "Rash on my ear that hurts" HPI: 50 y/o M presents with painful rash on his right ear x 1 week and radiates to his face. States he has had a burning pain for the last 10 days, but the rash showed up within the last 3-5 days. Pt. denies any new soaps, washes, or exposure to any new detergents. Rash has been spreading along ear. Pt. states the pain is a constant burning sensation (10/10). Touching the area makes it worse. Has tried OTC ibuprofen without any relief. Pt. denies any recent travel or fever. PMH: Hyperlipidemia, GERD, HTN Meds: Amlodipine 5mg PO daily, Protonix 40mg PO daily, Crestor 10 mg PO daily Surg Hx: Right Rotator cuff repair Soc Hx: Denies smoking, drinks 2 beers daily, no illicit drugs. Bus driver occupation. Vitals BP 140/94mmHg HR 99bpm RR 17bpm O2sat 98%RA Temp 98.6F Oral What other meds besides Prednisone?

Herpes Zoster Oticus Contraindications: None Adverse Rxns: N/V/D? Take with food. Counsel on steroids causing jittery-ness Safety/Monitoring: None

CC: "Rash on my ear that hurts" HPI: 50 y/o M presents with painful rash on his right ear x 1 week and radiates to his face. States he has had a burning pain for the last 10 days, but the rash showed up within the last 3-5 days. Pt. denies any new soaps, washes, or exposure to any new detergents. Rash has been spreading along ear. Pt. states the pain is a constant burning sensation (10/10). Touching the area makes it worse. Has tried OTC ibuprofen without any relief. Pt. denies any recent travel or fever. PMH: Hyperlipidemia, GERD, HTN Meds: Amlodipine 5mg PO daily, Protonix 40mg PO daily, Crestor 10 mg PO daily Surg Hx: Right Rotator cuff repair Soc Hx: Denies smoking, drinks 2 beers daily, no illicit drugs. Bus driver occupation. Vitals BP 140/94mmHg HR 99bpm RR 17bpm O2sat 98%RA Temp 98.6F Oral Med costs?

Herpes Zoster Oticus Special Considerations: - Why Valacyclovir vs Acyclovir - Let's look at cost.... Acyclovir is more cost efficient BUT... - 5x daily dosing vs 3x daily - Think compliance!!! Prednisone and gabapentin for pain control Patient to return to be assessed within 1 week Contraindications: none Adverse Reactions: N/V/D? Take with food. Counsel on steroids causing jittery-ness Safety/Monitoring: none

A 28-year-old female comes into your office complaining of fatigue. States it worsens when she has her menses. Her menses are heavy, and she uses 4-5 overnight pads per day. Admits they are fully saturated with blood when she changes them. Denies excessive clotting or cramping. Denies any melena or hematochezia. LMP 1 week ago. Denies any shortness of breath or palpitations. PMHx: None SocHx: Drinks 1 glass of wine per night. Denies any tobacco, illicit drug usage Vitals: All within normal limits. Allergies: NKDA Medications: None vAny labs you would like to order? Tx?

Hgb/Hct with retic count and peripheral smear - You can do an Hgb in office to check immediately if concerned of needing to send to ER for transfusion (Hgb<7g/dL) Ferritin TIBC Serum Iron What would you want to Rx? - Oral iron replacement •Ferrous sulfate 325 mg (65 mg elemental iron) PO tid •SE include dark stools, N/V and constipation •Requires 6-8 weeks of treatment to correct anemia Vit C helps w/ absorption of iron SE of oral iron: constipation (can do milk of magnesium w/ it) but eat with fibers (leafy greens)

CC "I was bit by a dog" A 12-year-old female with no past medical history is brought to your clinic by her parents for a dog bite that occurred 2 hours ago. The patient reports that she walking home from school on a different route than usual and encountered a "big, mean-looking dog" near an alley. She crossed the street to try to avoid it, but it ran after her and bit her leg through her jeans. Pt notes that an adult came out and grabbed the dog. States that she went home right away and washed the wound with soap and water. Notes that pain is throbbing and a 7/10. Standing makes the pain worse and sitting makes the pain better. ■PMHx - None ■Immunizations are up to date. ■Allergies - None ■Vitals - BP 102/64mmHg RA sitting RR 15bpm, HR 98bpm, Temp 98.7F Oral, Wt 45kg, Ht 60inches BMI 68% Cardiovascular and pulmonary examinations are normal. Examination of her skin and extremities reveals a 2 cm linear, shallow laceration over the right anterior thigh surrounded by 5 puncture wounds. Minimal active bleeding from these sites. She has full AROM of the RLE without pain

How do you want to treat this pt? - Antibiotics and clean area if needed When choosing an antibiotic for an animal bite, which microorganisms do you need to cover? - Pasteurella Canis What would you do for her wounds? - Clean area. No suturing due to puncture wounds What could happen if she does not receive any tx? - The area could become infected (5%) and of course possibly rabies, so due to hx - dog would need to go for a 10 days observation period and prophylaxis would be completed if dog developed symptoms (pending if dog was vaccinated) If unable to find owner - then recommendation is to contact health officials.

CC: " I can't stop vomiting" X 1 month HPI: 32 y/o F presents with vomiting for 4 weeks. Pt states she is 8 weeks pregnant. She has had multiple ER visits for excessive vomiting and was given "something in the IV that worked great- it started with an R?". She has been taking pyridoxine 25mg PO q 6 hours and that hasn't worked. She mentions she tried OTC Unisom because her friend said it worked, but it didn't help her. She has had no prenatal care and has only come to the ER for problems. They wrote her the RX for the pyridoxine 1 week ago. Pt denies fevers, diarrhea, abdominal pain and vaginal bleeding. This is her first pregnancy. •PMH: None, G1P0000, ER records reviewed from last weeks visit- US: 7week1day IUP, HR 140 •Surg Hx: None •Meds: Prenatal vitamins, Pyridoxine 25mg PO Daily •Allergies: NKDA •VS BP 124/80 HR 120 RR 18 O2sat 99% RA Temp 99.2 oral •General: Well developed, well nourished, alert and oriented •HEENT: dry mucous membranes •Abdomen: soft, nontender How do you tx?

Hyperemesis Gravidarum Reglan (Metoclopramide) 10mg PO q 6 hrs, Cat B (30 tablets, no refills) • Black box Tardive Dyskinesia most likely in elderly women, and for extended use Phenergan 25 mg PO/PR q 4-6hr, Cat C •CI if less than 2 years old- resp depression, caution in older, IV use is restricted due to tissue gangrene (can dilute 12.5mg in 50cc of saline), recommend 25mg IM deep. Also suppository 12.5mg, 25mg, 50mg Zofran 5mg ODT PO q 8 hours, Cat ??? (questionable in all 3 trimesters) Diclegis (Doxylamine-Unisom + pyridoxine-Vit B6)-2 tabs PO at bedtime or max 4 tabs (1 in am, 1 in pm, 2 at bedtime) Safety montioring: IV NA (1 L bolus, repeat in 30 mins-1 hr)

Hazel C., a 30-year-old female, demonstrates a subtle onset of the following sxs over the last 4 months: ◦Dull facial expression with droopy eyelids ◦Puffiness of the face and periorbital swelling ◦Sparse, dry hair with dry, scaly skin ◦Evidence of intellectual impairment ◦Lethargy, constipation, and hypothermia ◦Change of personality ◦Bradycardia (55 b/min) & blood pressure of 90/70 What if you want brand name for the pt?

Hypothyroidism Levothyroxine (Synthroid) Average full replacement dose: 1.7 mcg/kg/day (e.g., 100 to 125 mcg/day for a 70 kg adult) orally Mild to moderate hypothyroidism - starting dosage of 75-100 mcg/day is appropriate Older pts may require less than 1 mcg/kg/day (starting dose of 50 mcg/day) Clinical benefits begin in 3-5 days and level off after 4-6 weeks. Achieving a TSH level within the reference range may take several months because of delayed re-adaptation of the hypothalamic-pituitary axis. Dosing changes should be made every 6-8 weeks until the patient's TSH is in target range

Keys to Rational Prescribing

Make a specific diagnosis Select a specific therapeutic objective Select a drug of choice Determine the appropriate dosing regimen Devise a plan for monitoring the drug's action and determine an end point for therapy Plan pt education RECHECK pt's allergies

Monitoring Maintenance Fluids

Monitoring •Physical exam •Vital signs - HR, BP •Serum chemistries (BMP, osmolality, urinalysis, etc.) •Urine output - Adults - 0.5-1 mL/kg/hr - Peds - 1 mL/kg/hr Many people stay on IV fluids longer than necessary •Discontinue when oral intake is adequate •Avoid prolonged IV access when possible

CC: "My knee hurts" for 1 week. HPI: 75 y/o M pt presents with c/o "my knee hurts.". Pt. states that for the last week, he has noticed his left knee is swollen. Pt. denies any trauma or recent travel. He went 2 weeks ago for his usual chemotherapy treatment for small cell lung cancer. He states he feels tired a lot and occasionally feverish. Med Hx: Small Cell Lung CA, DM, HTN, CAD, COPD, OA. Meds: Metformin 500mg PO BID, Lisinopril 10 mg PO daily, Lipitor 40mg PO daily, Albuterol MDI prn, Advair Diskus 250/50 one puff BID, Diclofenac 50 mg PO BID. Allergies: PCN (Rash) Soc Hx: Smoker quit 2 yrs when diagnosed, but smoke 1 PPD for 40 years. No alcohol or illicit drugs. VS BP 109/80mmHg RA sitting HR 64bpm RR 14bpm O2sat 96% RA Temp 101.4F Oral WT

Monoarthritis knee Septic arthritis must be ruled out- treat empirically with BSAs..e.g VANCO/GENTAMICIN Use a Broad spectrum antibiotic: Vancomycin or Gentamicin (admit to hospital - both IV meds) Vancomycin •15-20mg/kg every 8-12 hours IV Gentamicin 1-1.7mg/kg every 8 hours IV Note: check the synovial fluid pic from powerpoint

A 24 y/o graduate student was studying late at night for an examination. As he looked at his textbook, he realized that his left arm and left leg were numb. He dismissed the complaint, recalling that 6 or 7 months ago he had similar symptoms. He rose from his desk and noticed that he had poor balance. He queried whether his vision was blurred and remembered that he had some blurred vision approximately 1 to 2 years earlier, but that this resolved. He had not seen a physician for any of these previous symptoms. He went to bed and decided that he would seek medical consultation the next day. Dx?

Multiple Sclerosis What diagnostic testing should be done MRI brain Lumbar puncture What is your next step in therapy? Steroids to aid with flare-up Referral for Neuro for medication if needed chronically What are some therapies this pt can do in between flare ups? Many therapies are specific from patient to patient -Physical therapy -Mental Health -Muscle relaxors (spasticity) -Medication for neuropathic pain if needed Oral glucocorticoids are recommended although IV glucocorticoids are utilized highly. Oral 625mg-1250mg daily x 3-7 days IV - admit to hospital for 1000mg daily x 7 days **IV recommended in patients with optic neuritis due to oral steroids worsening neuritis**

CC "My right ear hurts!" HPI: 40 y/o F presents to your office with right ear pain x 2 days. Pt. states she is a flight attendant and has been experiencing this sharp ear pain (8/10) for the last 2 days. She states the pressure makes it worse (10/10). She admits to "feeling hot" but denies checking her temperature. She denies any ear discharge. Ibuprofen helps the pain briefly but only for a short time ~2 hours. PMH: Uterine Fibroids Surg Hx: C-section (10 years ago) Soc Hx: Smokes ½ PPD x10 years Meds: Zolpidem 5mg PRN Allergies: PCN (Hives) Vitals BP 118/88mmHg HR 84bpm RR 16bpm 02sat 98% RA Temp 99.2 F Oral Pharmacist calls and says patient notes that it wasn't just hives when she took the PCN, she also had anaphylaxis and that patient is now saying she also had an unknown reaction to Keflex... What do you do?!

OM Contraindications: PCN allergy --> hives = IgE mediated rxn Adverse Reactions: N/V/D? Take with food, Anaphylactic sxs? STOP med/take benadryl and go to ER Safety/Monitoring: Macrolide written due to hx allergy Other meds to consider & why they weren't chosen in this scenario: Azithromycin & Clarithromycin - High rate of resistance to S. pneumoniae Trimethoprim-sulfamethoxazole - High rate of resistance to H. influenzae and S. pneumoniae Clindamycin - No activity against H. influenzae

CC "My right ear hurts!" HPI: 40 y/o F presents to your office with right ear pain x 2 days. Pt. states she is a flight attendant and has been experiencing this sharp ear pain (8/10) for the last 2 days. She states the pressure makes it worse (10/10). She admits to "feeling hot" but denies checking her temperature. She denies any ear discharge. Ibuprofen helps the pain briefly but only for a short time ~2 hours. PMH: Uterine Fibroids Surg Hx: C-section (10 years ago) Soc Hx: Smokes ½ PPD x10 years Meds: Zolpidem 5mg PRN Allergies: PCN (Hives) Vitals BP 118/88mmHg HR 84bpm RR 16bpm 02sat 98% RA Temp 99.2 F Oral Other Medications to consider other than Doxy and Omnicef?

OM (2dry Eustachian Tube dysfunction?) Azithromycin & Clarithromycin - High rate of resistance to S. pneumoniae Trimethoprim-sulfamethoxazole - High rate of resistance to H. influenzae and S. pneumoniae Clindamycin - No activity against H. influenzae

CC "My right ear hurts!" HPI: 40 y/o F presents to your office with right ear pain x 2 days. Pt. states she is a flight attendant and has been experiencing this sharp ear pain (8/10) for the last 2 days. She states the pressure makes it worse (10/10). She admits to "feeling hot" but denies checking her temperature. She denies any ear discharge. Ibuprofen helps the pain briefly but only for a short time ~2 hours. PMH: Uterine Fibroids Surg Hx: C-section (10 years ago) Soc Hx: Smokes ½ PPD x10 years Meds: Zolpidem 5mg PRN Allergies: PCN (Hives) Vitals BP 118/88mmHg HR 84bpm RR 16bpm 02sat 98% RA Temp 99.2 F Oral Pharmacist calls and says pt notes that it wasn't just hives when she took the PCN, she also had anaphylaxis and that patient is now saying she also had an unknown reaction to Cefalexin (Keflex)...

OM (2dry Eustachian Tube dysfunction?) Contraindications: PCN allergy --> hives = IgE mediated rxn Adverse Rxns: N/V/D? Take with food, Anaphylactic symptoms? STOP med/take benadryl and go to ER Safety/Monitoring: Macrolide written due to hx allergy

CC "My right ear hurts!" HPI: 40 y/o F presents to your office with right ear pain x 2 days. Pt. states she is a flight attendant and has been experiencing this sharp ear pain (8/10) for the last 2 days. She states the pressure makes it worse (10/10). She admits to "feeling hot" but denies checking her temperature. She denies any ear discharge. Ibuprofen helps the pain briefly but only for a short time ~2 hours. PMH: Uterine Fibroids Surg Hx: C-section (10 years ago) Soc Hx: Smokes ½ PPD x10 years Meds: Zolpidem 5mg PRN Allergies: PCN (Hives) Vitals BP 118/88mmHg HR 84bpm RR 16bpm 02sat 98% RA Temp 99.2 F Oral

OM (2dry Eustachian Tube dysfunction?) Contraindications: PCN allergy --> hives = IgE mediated rxn Adverse Rxns: N/V/D? Take with food, Anaphylactic symptoms? STOP med/take benadryl and go to ER Safety/Monitoring: Macrolide written due to hx allergy Can also use: Cefdinir, Azithro, Doxy

What Fluids are Available? Additives

Routine additives Potassium chloride - 20-40 mEq/L •Replace ongoing losses K+ bolus can be irritating and lethal (development of arrhythmias) Sodium bicarbonate - 50-150 mEq/L •Used for alkalinization •Acetate also converts to bicarbonate: Given as sodium/potassium acetate (ex: someone getting TPN) Other additives •Amino acids (4 grams) •Other electrolytes (Calcium, magnesium, phosphorous) •Lipids

CC: "I have excruciating pain in my left ear." HPI: 17 y/o F was swimming at the pool and the beach all last week while on vacation in the Florida keys. She states since last night she has severe ear pain 10/10 and she can't even put her head on the pillow because it hurts so bad. She states she "feels warm" but has not checked for a fever. She also feels some "gooey" stuff is coming from her ear. She has taken Goody's powder with no relief. PMH: Asthma Surg Hx: None Meds: Albuterol 90mcg/spray MDI PRN Allergies: Azithromycin (Hives) Soc Hx: Denies alcohol, drugs, or smoking Vitals BP 112/78mmHg HR 72bpm RR16bpm O2sat 99% RA Temp 98.9F Oral PE: Positive Tragal and Helix tenderness. Otoscopic exam limited - unable to assess TM Medication Considerations?

Otitis Externa Contraindications: none Adverse Rxns: Skin irritation - return to office Safety/Monitoring: none

CC: "My tongue looks funny" HPI: 61 y/o M patient presents complaining "my tongue looks funny and white.". Pt. states that for the last 2 weeks he has noticed his tongue has a white appearance. Pt. denies any trauma or eating different foods. Pt. states he went 2 weeks ago for his usual chemotherapy treatment for small cell lung cancer. Pt. denies any recent fevers. He states he feels tired a lot. Pt admits to mild discomfort of tongue. Last dental exam was 3 years ago. Denies any mouth bleeding or difficulty swallowing. Notes that he is just concerned. Med Hx: Small Cell Lung CA, DM, HTN, CAD, COPD Meds: Metformin 500mg PO BID, Lisinopril 10 mg PO daily, Lipitor 40mg PO daily, Albuterol MDI prn, Advair Diskus 250/50 one puff BID *Compliant with all meds* Allergies: Ciprofloxacin (Rash) Soc Hx: Smoker quit 2 years when diagnosed but smoke 1 PPD for 40 years. No alcohol or illicit drugs. Vitals BP 109/80mmHg HR 64bpm RR 14bpm O2sat 96% RA Temp 99.4 F Oral Tongue blade is used to scrape off whitish patches which have slight bleeding after examining Special Considerations for Tx?

Oral Candidiasis Due to chemo tx - think mucositis - If continues - will need to possibly discuss with onco in regard to a prophylactic fluconazole- Efficacy still be determined at this time - Will need to monitor to make sure that fungal infection improves and does not become systemic- High risk due to multiple factors: Chemo, DM, Inhaled steroids Counsel pt on DM control - Order updated labs - Alter medications pending results Counsel pt on rinsing mouth after using Advair - Use when brushes teeth in the am and pm Pt to follow up with dentist - he is overdue for his annual cleaning

CC: "My tongue looks funny" HPI: 61 y/o M patient presents complaining "my tongue looks funny and white.". Pt. states that for the last 2 weeks he has noticed his tongue has a white appearance. Pt. denies any trauma or eating different foods. Pt. states he went 2 weeks ago for his usual chemotherapy treatment for small cell lung cancer. Pt. denies any recent fevers. He states he feels tired a lot. Pt admits to mild discomfort of tongue. Last dental exam was 3 years ago. Denies any mouth bleeding or difficulty swallowing. Notes that he is just concerned. Med Hx: Small Cell Lung CA, DM, HTN, CAD, COPD Meds: Metformin 500mg PO BID, Lisinopril 10 mg PO daily, Lipitor 40mg PO daily, Albuterol MDI prn, Advair Diskus 250/50 one puff BID *Compliant with all meds* Allergies: Ciprofloxacin (Rash) Soc Hx: Smoker quit 2 years when diagnosed but smoke 1 PPD for 40 years. No alcohol or illicit drugs. Vitals BP 109/80mmHg HR 64bpm RR 14bpm O2sat 96% RA Temp 99.4 F Oral Tongue blade is used to scrape off whitish patches which have slight bleeding after examining How to tx?

Oral Candidiasis Special Considerations: What are pt's recent labs for DM control? - Last reported 5 months ago & HgbA1c 8.3% & FBG 196 - Pt doesn't check glucose at home. Notes that he "feels fine" Does patient rinse his mouth after using Advair? - Pt notes he tries to remember, but sometimes forgets Pt is on chemo - think mucositis! Contraindications: none at this time Adverse Reactions: none Safety/Monitoring: if no improvement return

A 45-year-old African-American man presents with blood pressure issues. He states that he attended a work-sponsored health fair; he had his blood pressure taken, and the health fair representative strongly urged him to make an appointment to be seen by his primary care provider. The fair occurred 3 months ago. He has randomly checked his blood pressure at different times since the health fair in local grocery stores, and although he does not remember the numbers, he knows that "they were above normal". The patient also believes that both his mother and father are taking blood pressure medication, but he is not 100% sure. Last seen by provider 15 years ago. Vitals: BP 175/95mmHg LA Sitting, 172/99mmHg RA Sitting HR 88bpm, RR 16 bpm, BMI of 30 kg/m2 and a waist circumference of 41 inches. Medications: None Allergies: NKDA PMHx: None PE: Lungs and heart exam are normal. There is no edema present. Next step?

Order labs including: Kidney function (BMP or CMP) Lipids Urinalysis ECG in office - NSR no ST elevation/depression noted JNC-8 guidelines Goal - <140/90mmHg African-American - start with CCB, thiazide or both Amlodipine 5mg daily and return in 2 weeks to reassess Counsel pt regarding lifestyle changes: -Smoking Cessation -Labs to check on glucose and lipid control -Dietary changes (DASH diet, Eating healthy) -Moderate alcohol consumption -Reduce sodium consumption to less then 2400mg/day -Increase fiber -Increase water -Increase physical activity -Moderate-to-vigorous activity 3-4 days a week averaging 40 min per session.

CC "my child is cranky with a fever x 3 days" Mom brings in her 1 yo Caucasian female daughter for fever x 3 days. Mom notes that patient has had fever ranging from 100.2F - 102.1F Tympanic over the last 3 days. Tylenol was given with improvement of fever. Pt is pulling at her ears, not sleeping well, crying more often then normal but is eating and drinking normally with normal amount of wet/dirty diapers daily. Pt attends daycare, no longer drinks from bottles and there are no smokers in the home. (Pt not dehydrated) Allergies: NKDA Medications: None Vitals: Ht:27", Wt 20lbs, HR 110 bpm, RR 25 bpm HC 46cm TEMP 100.2F (Tylenol 5 hours ago) O2 99%RA PHYSICAL EXAM: HEENT: H: normocephalic, E: Sclera/conjunctiva normal, non-injected E: Right TM red bulging, no effusion noted. Left TM Pearly grey no bulging or effusion, N - Rhinorrhea noted T - noninjected no erythema noted. ◦CARDIO - S1S2 noted, no murmurs, rubs or gallops ◦RESPIRATORY - CTA B/L Ok, but...what if mom stated that kiddo had a PCN allergy (hives) - NO ANAPHYLAXIS? What would you want to give?

Otitis Media Cefdinir ◦20lbs ◦14mg/kg/day (OM acute) ◦20lbs/2.2kg = 9.09kg ◦14mg*9.09kg = 127.27mg/day / 2 (12 hours)= 63.6mg per dosage ◦125mg/5mL = 25 mg/mL = (63.6/25) = 2.5mL ◦Pt to take 2.5mL twice daily x 10 days ◦5mL daily x 10 = 50mL

CC "my child is cranky with a fever x 3 days" Mom brings in her 1 yo Caucasian female daughter for fever x 3 days. Mom notes that patient has had fever ranging from 100.2F - 102.1F Tympanic over the last 3 days. Tylenol was given with improvement of fever. Pt is pulling at her ears, not sleeping well, crying more often then normal but is eating and drinking normally with normal amount of wet/dirty diapers daily. Pt attends daycare, no longer drinks from bottles and there are no smokers in the home. (Pt not dehydrated) Allergies: NKDA Medications: None Vitals: Ht:27", Wt 20lbs, HR 110 bpm, RR 25 bpm HC 46cm TEMP 100.2F (Tylenol 5 hours ago) O2 99%RA PHYSICAL EXAM: HEENT: H: normocephalic, E: Sclera/conjunctiva normal, non-injected E: Right TM red bulging, no effusion noted. Left TM Pearly grey no bulging or effusion, N - Rhinorrhea noted T - noninjected no erythema noted. ◦CARDIO - S1S2 noted, no murmurs, rubs or gallops ◦RESPIRATORY - CTA B/L Dx? Tx?

Otitis Media What do you want to prescribe? Amoxicillin Now let's talk dosing! ◦20lbs ◦80-90mg/kg/day (OM acute) ◦20lbs/2.2kg = 9.09kg ◦80mg*9.09kg = 727.27mg/day / 2 (12 hours)= 363mg per dosage ◦400mg/5mL = 80 mg/mL = (363/80) = 4.54mL Pt to take 4.5mL twice daily x 10 days ◦9mL daily x 10 = 90mL

PA Prescribing Laws in Florida: Unauthorized meds

PAs ARE NOT AUTHORIZED TO PRESCRIBE THE FOLLOWING MEDICINAL DRUGS, IN PURE FORM OR COMBINATION: •(a) General, spinal or epidural anesthetics •(b) Radiographic contrast materials

PA Prescribing Laws in Florida: Orders

PAs may sign a written order in the hospital for any drug, including controlled medicinals, used in the scope of their supervising physicians' practice in a hospital setting. This is not considered prescribing but an order to be carried out by nursing staff Inpatient orders must be cosigned by a physician within the required time frame as per the hospital's bylaws. This has been affirmed by a legal opinion by the Florida Attorney General's office on two occasions, but this continues to be an issue that is challenged on occasion by hospitals.

55 y/o man in good health until about 6 months ago. At that time he noticed the development of a tremor in his right arm. States that the tremor is all the time and worsens when he is not using his arm. He has no other complaints and his family history is unknown as he is adopted. There is a tremor in the right arm at rest and while he walks, he has a sustained tremor in both arms, and to some degree during finger-nose-finger maneuvers (fairly fine and without an obvious rhythm). He has a poker face and a slow, deliberate gait. Tone is increased in the right arm and leg. The PE is otherwise unremarkable. He and his wife deny his use of alcohol or any other medications. Dx?

Parkinson's Disease What is a common side effect specifically to Parkinson's medication, Amantadine? Amantadine: **Can cause livedo reticularis or a red/blue "fishnet" mottling**

What are the regulations to controlled substance prescribing for PAs in Florida?

PAs must be delegated the authority to prescribe controlled substances by their supervising physicians. PAs are prohibited from prescribing controlled substances in registered pain management clinics. PAs may not prescribe psychiatric mental health controlled substances for children younger than 18 years of age As of July 1, 2018 there are additional regulations regarding the prescribing of controlled substances by PAs: · For the treatment of acute pain, a prescription for an opioid drug listed as a Schedule II controlled substance may not exceed a 3-day supply · Under special circumstances (ie: acute pain exemption with appropriate documentation), a 7-day supply of a schedule II controlled substance may be prescribed. This is described in more detail below. · An emergency opioid antagonist must be prescribed in addition to the schedule II controlled substance for certain traumatic injuries as described below. · Prescribers are required to check the prescription drug monitoring program (PDMP) before prescribing or dispensing a controlled substance for a patient age 16 or older. The PDMP must be checked each time a prescription for a controlled substance is written. · Each person registered with the United States Drug Enforcement Administration and authorized to prescribe controlled substances pursuant to complete a board-approved two-hour continuing education course on prescribing controlled substances as part of biennial license renewal. The course must be completed by January 31, 2019, and at each subsequent renewal.

CC "I have green stuff leaking out of my vagina and my stomach hurts" x 4 days HPI: 22 y/o F presents to the ER complaining of green vaginal discharge and abdominal pain. Started 4 days ago with discharge and the pain started this morning. She has felt feverish, but never took her temperature. Pain in lower abdomen, does not radiate. No n/v/d. No URI symptoms. No vaginal bleeding. LMP normal 2 weeks ago. •PMH: None •Surg Hx: Appendectomy age 10 •Soc Hx: multiple sexual partners, unprotected intercourse •Meds: None •Allergies: None •VS BP 118/88 HR 110 RR 16 02sat 98% RA Temp 100.6 oral Speculum exam reveals: Bimanual: +CMT US: no tubo-ovarian abscess How do you treat?

PID Rocephin 250mg IM once AND Doxycycline 100mg PO BID x 14 days •Photosensitivity •No antacids within 2 hours •MUST TAKE WITH FLUIDS to avoid esophageal irritation •Taking barbituates, carbamazepine, rifampin, and phenytoin will decrease doxy levels

55 y/o man in good health until about 6 months ago. At that time he noticed the development of a tremor in his right arm. States that the tremor is all the time and worsens when he is not using his arm. He has no other complaints and his family history is unknown as he is adopted. There is a tremor in the right arm at rest and while he walks, he has a sustained tremor in both arms, and to some degree during finger-nose-finger maneuvers (fairly fine and without an obvious rhythm). He has a poker face and a slow, deliberate gait. Tone is increased in the right arm and leg. The PE is otherwise unremarkable. He and his wife deny his use of alcohol or any other medications. Med besides Carbidopa/levodopa?

Parkinson's Disease

55 y/o man in good health until about 6 months ago. At that time he noticed the development of a tremor in his right arm. States that the tremor is all the time and worsens when he is not using his arm. He has no other complaints and his family history is unknown as he is adopted. There is a tremor in the right arm at rest and while he walks, he has a sustained tremor in both arms, and to some degree during finger-nose-finger maneuvers (fairly fine and without an obvious rhythm). He has a poker face and a slow, deliberate gait. Tone is increased in the right arm and leg. The PE is otherwise unremarkable. He and his wife deny his use of alcohol or any other medications. Med besides Pramipexole?

Parkinson's Disease

CC: "F/U for Bipolar I disorder" HPI: A 22-year-old male presents with his mother for follow up on bipolar 1 disorder. Patient notes that he was seeing a different PCP and was given Lithium. Mother notes that patient has not been taking his medication and his mood swings are starting to affect his daily life. Pt notes he isn't sleeping, is up playing xbox at all hours of the night. Notes that he is having racing thoughts and mom notes that he has spent all the money she has given him since he has returned home. Patient notes that he stopped the medication due to side effects. Patient was seeing a counselor while at college but hasn't seen once since returning home. He states he will try to find another counselor here as it has helped him in the past. Pt has hx of suicide attempt x 3 over the last 2 years and was Baker acted x1. Denies any suicidal ideation or homicidal ideation currently. PMHx: Bipolar I disorder; suicide attempts x3 over the last 2 years Surg Hx: tonsillectomy (child 8 y/o) Meds: Lithium 300mg 3x daily Allergies: NKDA VS BP 132/82mmHg RA sitting, HR 92bpm, RR 18bpm, Temp 98.4F Oral What is the next best treatment?

Patient is describing a manic episode and has known bipolar 1 disorder. Patient needs to follow up with a new counselor and possible psychiatry. Referrals can be written while in the office. Lithium can be changed to Valproic Acid (Depakote). Labs should be drawn today to get baseline LFTs and have patient return in 2-4 weeks to reassess mood as well as monitor valproic acid levels in the blood. Repeat LFTs in 6 months

CC: "It is hard for me to swallow and my throat hurts." HPI: 34 y/o M present to the ER. Pt. has difficulty talking due to pain and voice is very low. He is drooling, has a muffled voice and states " My throat hurts and it's hard to swallow." He states pain is sharp (10/10) x 4 days and is taking Advil and Cepacol with no relief. Pt. states he is also feels "feverish" but has not taken his temperature. Pt. states he has been unable to eat or drink in the last day due to pain PMH: Herniated Disc Lumbar L5/S1 Surg Hx:None Meds: Advil OTC Allergies: PCN (Rash, Hives) Soc Hx: Smokes 1 PPD for 15 years, drinks occasionally beer, smokes marijuana at times. Vitals: BP 128/90mmHg HR 129bpm RR19bpm O2sat 97%RA Temp 102.8F Oral How do you Tx?

Peritonsillar abscess - I&D? Needle Aspirate? Remember we are in the hospital setting so how would you order meds for this pt? IV abxs while in the ER - allergic to PCN - Clindamycin 600mg q 6 hrs IV - Vancomycin 15-20mg/kg IV q 8-12 hrs If responds well to parenteral & stable - then release with written Rx and advise to f/u with PCP/ENT

Which drugs can a prescribing PA write prescriptions for at this time?

Prescribing physician assistants are able to prescribe any medication except those specifically prohibited by the Formulary Committee of the PA council and the Boards of Medicine. The medications that physician assistants are excluded from prescribing include general anesthetics and radiographic contrast materials. While physician assistants can prescribe controlled substances, there are regulations as outlined below. Of note, the regulations were most recently changed on July 1, 2018. Florida statute requires ALL prescribing PAs to take a three-hour continuing education course on the Safe and Effective Prescribing of Controlled Substances prior to EVERY Florida license renewal. Only a PA with a current license with approved prescribing privileges can write prescriptions in Florida. Prescribing PAs can prescribe any medication except those specifically excluded by law under the provisions of our Exclusionary formulary. Also, the only medications you should write for are those medications authorized by your supervising physician within the scope of his/her practice since you are also limited to practice within the scope of your supervising physician's practice. In a hospital setting the law is different. You may write orders for any medications, including controlled substances. It is considered a verbal order from your supervising physician when you write it if the duty has been delegated to you. You cannot give the patient a written prescription upon discharge for any of those medications excluded by law. If it is written in the discharge orders, the nurse may call it in to a pharmacy for the patient unless it is a medication requiring a written prescription.

Classification of Drugs

Prescription drug Controlled or scheduled drug

Drug that requires a prescription because it is considered potentially harmful if not used under the supervision of a licensed health care practitioner

Prescription drug Known synonymously as a legend drug because the label of the drug bears the legend, "Caution: Federal Law Prohibits Dispensing without a Prescription" or "Rx only."

Hx of prescriptions

Prescriptions have been in use since ancient times •Latin adopted as standard language •"Rx" = prescription •"Sig." = directions Federal Food, Drug, and Cosmetic Act (1938) •Non - prescription •Prescription (Legend drugs, Controlled drugs)

Review GPFPAL

Prior pregnancies: G PFPAL ›Gravida - total number of pregnancies ›Parity - outcome of all pregnancies (>20 wks) ›Full term - 37 weeks or greater ›Premature births - 20 - 36 weeks ›Abortions/Miscarriages <20 weeks ›Living children - total number of living children

CC: "Rash on my ear that hurts" HPI: 50 y/o M presents with painful rash on his right ear x 1 week and radiates to his face. States he has had a burning pain for the last 10 days, but the rash showed up within the last 3-5 days. Pt. denies any new soaps, washes, or exposure to any new detergents. Rash has been spreading along ear. Pt. states the pain is a constant burning sensation (10/10). Touching the area makes it worse. Has tried OTC ibuprofen without any relief. Pt. denies any recent travel or fever. PMH: Hyperlipidemia, GERD, HTN Meds: Amlodipine 5mg PO daily, Protonix 40mg PO daily, Crestor 10 mg PO daily Surg Hx: Right Rotator cuff repair Soc Hx: Denies smoking, drinks 2 beers daily, no illicit drugs. Bus driver occupation. Vitals BP 140/94mmHg HR 99bpm RR 17bpm O2sat 98%RA Temp 98.6F Oral Any extra question/referrals?

Pt admits to having chicken pox as a child Pt has not used any creams over the area No lesions noted on tip of nose - No emergent Ophthalmologist referral needed at this time - Counsel pt if lesions appear around eye or on tip of nose specifically to be seen by Ophthalmologist ASAP! Herpes Zoster Oticus Contraindications: None Adverse Rxns: N/V/D? Take with food. Counsel on steroids causing jittery-ness Safety/Monitoring: None

A 56-year-old male with Type II DM presents to the ER after waking up in the morning to find that he had a swollen, red, painful big toe of his left foot. He had recently been on a cruise to the Bahamas and spent much of the time eating and drinking beer more than usual. An X-ray of his left foot shows no acute fracture. Vital signs are within normal limits. Blood work shows an increase in uric acid, but all other labs are otherwise unremarkable. The patient states he cannot tolerate NSAIDs due to a history of GI bleeding. NKDA. What is the best tx medication once acute attack is controlled?

Pt is now following up with PCP Uric acid labs completed, & levels are at 8 mg/dL What to do next?: Write for allopurinol and continue to recheck levels every 1-2 wks until level below 6mg/dL. Then maintenance dosage will be written

63-year-old Asian Male is seen in your office for a blood pressure follow up. He has had hypertension for 20 years, but recently has had difficulty controlling it. He takes Olmesartan 40mg daily and is compliant. Denies any headaches, chest pain, palpitations, shortness of breath, dizziness. He checks his bp at home 3x a week. Denies any increase in stress recently. Allergies: NKDA Medications: Olmesartan 40mg daily, Metformin 500mg twice daily, Pravastatin 20mg daily, Flomax 0.4mg daily (compliant with all medications) PMHx: DM2, HTN, Hyperlipidemia, BPH SocHx: No tobacco, alcohol, illicit drug usage Vitals: BP 172/101mmHg LA sitting, Pulse 86bpm, RR 15bpm, O2 99% RA, 98.8F Oral, FBG 112, HgbA1c - 7.1% So what would you want to do?

Pt is on max dosage of Olmesartan, so we would want to add a diuretic To aid with compliance, many a combination medication would be a great option Olmesartan/HCTZ 40/12.5mg would be a great possibility Take it in the AM though so he's not up all night urinating. He already has BPH! Labs ordered

You are treating a 52-year-old male, who works as a mechanical engineer, for his hyperlipidemia. When you initially saw him, the man's total cholesterol was 380 mg/dl HDL 15 mg/dl, LDL 250 mg/dl and triglycerides were 280 mg/dl. He is 168 cm (5 ft 4 in) tall and weighs 100 kg (220 lbs.) Your initial approach was to change his lifestyle by setting up an exercise routine and putting him on a low-fat diet. Three months later, his total cholesterol had gone down to 300 mg/dl, HDL 20 mg/dl, LDL 180 mg/dl, Trigs 250, and he weighed 88 kg. Six months later, he is still exercising and observing his diet, but his labs have not changed significantly. His weight decreased another 16 kg, and he now weighs 72 kg. What is the next step in the management of this patient's cholesterol?

Pt needs high intensity statin control. The statins with the least side effects are Rosuvastatin (Crestor) and Pravastatin (Pravachol) Start at 20mg and recheck in 6 months - If still elevated at 6 months - increase to 40mg daily Continue with lifestyle changes High intensity statin treatment. Best statins with least SAE are pravastatin and rosuvastatin.

CC - "It hurts when I swallow x 2 days" A 6 yo Asian female is accompanied by her mother today for pain with swallowing x 2 days. Mom notes that patient has had fever of up to 103F Tympanic. Mom has given ibuprofen and Tylenol with improvement of fever. Mom notes that recently was here 1 week ago with older brother and he was positive for a throat infection. Wants to get patient checked prior to sending her back to school. ◦Allergies - NKDA ◦Medications - None ◦Vitals: Ht 44" Wt 45lbs Temp 99F Tympanic (Tylenol 1 hour ago) All other vitals normal ◦PHYSICAL EXAM ◦HEENT: H: normocephalic, E: Sclera/conjunctiva normal, non-injected E: Right/Left TM Pearly grey no bulging or effusion, N - Rhinorrhea noted T - +posterior oropharynx injected +tonsillar exudate b/l +ant cervical lymphadenopathy ◦CARDIO - S1S2 noted, no murmurs, rubs or gallops ◦RESPIRATORY - CTA B/L test to order? Meds?

Rapid Strep and throat culture What medication would you want to write? Amoxicillin 50mg/kg every 24hours x 10 days (strep) <-- Different dosing then for OM can also do PCN, Azithro 45lbs 50mg/kg/day (Max 1000mg/day) 45lbs/2.2kg = 20.45kg 50mg*20.45kg = 1020.583mg/day > max of 1000mg/day 400mg/5mL = 80 mg/mL = (1000/80) 12.5mL daily --> that is 2.5tsp daily 12.5mLx10 = 125mL IF able - pt can take tablets instead vs 2.5tsp ◦**don't forget they are crushable**

Good Prescribing Practice: Refills

Refills are not required To avoid interrupting maintenance therapy, practitioners can authorize refills on a written prescription Non-controlled substances - No limit to number of refills allowed. Use Discretion. Refills authorized are valid only for life of the prescription - 1 year

65-year-old pt returns to you for hospital follow up. He thanks you for sending him to the hospital and is feeling much better. At this time denies any palpitations, chest pain, shortness of breath since being released from the hospital. Patient has been taking his medications daily. BP in office today is 126/72mmHg LA sitting, HR 68bpm regular Medications: Rivaroxaban (Xarelto) 20mg daily, Metoprolol Succinate (Toprol-XL) 100mg daily Tx as a PCP/UC PA-C?

Refills given - #90 with 1 refill for mail in pharmacy due to Medicare Referral for cardio written Follow up appt created

65-year-old pt returns to you for hospital follow up. He thanks you for sending him to the hospital and is feeling much better. At this time denies any palpitations, chest pain, shortness of breath since being released from the hospital. Patient has been taking his medications daily. BP in office today is 126/72mmHg LA sitting, HR 68bpm regular Medications: Rivaroxaban (Xarelto) 20mg daily, Metoprolol Succinate (Toprol-XL) 100mg daily Tx as a PCP/UC PA-C besides Metoproplol?

Refills given - #90 with 1 refill for mail in pharmacy due to Medicare Referral for cardio written Follow up appt created Can cardiovert as new onset afib < 48 hrs without LAE or mural clot. Medication: Warfarin candidate or NOACs are also being used more now.

What if I need to send a patient to pain mgmt.?

Remember... Pain Management is NOT the same as Addiction Medicine! Be straightforward with your patients about their pain and what they should expect Make pain "tolerable" Medications won't "erase" the pain Know your state-laws when it comes to prescribing opiate medications

Maintenance Fluids

Replace or maintain fluid balance Rates Healthy adult: 30-40 mL/kg/day (~2-3 L/day) •Most people start at 125 mL/hr (3 L/day) and adjust (e.g., 0.5, 1.5, 2x maintenance) Peds: 4:2:1 rule •First 10 kg: 4 mL/kg/hr •Second 10 kg: 2 mL/kg/hr •Subsequent 1 kg : 1 mL/kg/hr What fluids to use? NS - typically 0.45-0.9% NaCl •Iatrogenic hyponatremia with 0.45% NaCl •Remember ADH! (also decrease Na+ levels) Dextrose - typically 5% •Provides calories to prevent catabolism (how many calories/L?) •D10W used in patients at risk for hypoglycemia (neonates) Potassium - typically 20-40 mEq/L •Avoid if hyperkalemic Avoid LR (unless acidotic) •Leads to alkalemia

CC - "My hands itch at night" 32 yo Caucasian male comes to the office with itching and a rash x 5 days. Notes that the itching is worse at night. States that he recently noticed a rash on his hands and wrists. Has been using Vaseline on the area with no relief. States that he was recently visiting with his family and his nephews also had a rash and were given a cream by their pediatrician. Patent unsure of the name of the cream. •PMHx: None •FamHx: Unknown •Medications: None Allergies: None Pe: multiple small, erythematous papules with excoriations noted Counseling? Tx options

Scabies Make pt aware that might have continued itching atfter th tx. This will improve & go away on it's own. It does not automatically mena that the tx didn't work Permethrin 5% cram Apply from the neck down & leave on for 8 hours, rinse & repear in 7 days if needed #30 grams (thirty) Refills: 0 (zero)

CC - "my eyes are itching and red x 1 month" Mother brings in her 7-year-old Asian son due to itchy eyes and cough. States that over the last few years he has been complaining of itching eyes as well as runny notes that worsens in the spring. Mom given the patient Benadryl if the symptoms are severe with relief. Patient has not been on a daily medication. Family history of allergies and asthma. Patient denies any personal history of allergies or asthma. Mom states that she tries to keep house clean and doesn't allow the pets in patient's bedroom ◦Allergies: NKDA ◦Medications: Children's Benadryl OTC as needed for symptoms ◦PHYSICAL EXAM: ◦HEENT: H: normocephalic, E: Conjunctiva injected b/l with cobblestone like changes noted of interior of lower lid b/l. E: Right/Left TM Pearly grey no bulging or effusion, N - Rhinorrhea +pale blue boggy turbinates noted b/l T - +postnasal drip noted. No tonsillar exudate or lymphadenopathy noted ◦CARDIO - S1S2 noted, no murmurs, rubs or gallops ◦RESPIRATORY - CTA B/L Dx? Meds? Counseling?

Seasonal Allergies What do you want to prescribe? Allergy medication ◦Cetirizine (Zyrtec), Loratadine (Claritin), Fexofenadine (Allegra) Nasal spray if needed Any counseling for mom? ◦Try to keep child away from irritants ◦Pt can rinse nose with nasal saline when he comes in from playing outside ◦Pt to keep all animals out of his room ◦Continue with vacuuming and keeping house clean ◦Use medication as needed ◦If no improvement, possible allergist referral Can also do: Flonase, cromolyn

Breast Ca. Hormone Therapy

Selective estrogen receptor modulators (SERM) - raloxifene (Evista), tamoxifen, toremifene (Fareston), lasofoxifene -Block the effect of estrogen in the breast tissue - Can be used to tx women before and after menopause SE: fatigue, hot flashes, night sweats, vaginal d/c, mood swings, blood clots, stroke, endometrial cancer, do not take while pregnant, can improve bone density Aromatase inhibitor - Aromasin, Arimidex and Femara - Best type of hormonal therapy to start with for postmenopausal women - Stop production of estrogen, block aromatase enzyme that turns androgen into small amounts of estrogen in the body

CC - "I have spots on my back" 19 yo Hispanic male comes to your office due to white "spots" noted on his chest and upper back. States has some itching but not all the time. Nothing makes the area worse and nothing makes it better States that he has tried over the counter fungal cream with no relief. Denies any changes in shampoo, lotion or soap. Denies any sick contacts. •PMHx: Acne •FamHx: Unknown •Medications: None •Allergies: Azithromycin Counseling? Tx options?

Tinea versicolor •Wear sunscreen •It may take months for pigmentation to return to normal. This does NOT mean that traet has failed Tx: Selenium sulfide Anitfungal Ciclopirox Celsonblue Ketoconazole 2% shampoo Selenium sulfide 2.5% lotion Apply to affected area, lather, wait 10 mins and rinse. Repeat daily x 10 days #120 milliliters (one hundred twenty) Refills 0 (zero)

Steroids - the weak and the strong!

To reduce tachyphylaxis - Rx for a week on and then a week off Some recommend -Rx for 3 days on and then 4 days off Long-term use of topical steroids can lead to secondary infection with fungus or bacteria (tinea incognito), skin atrophy, telangiectasia, skin bruising and fragility 7 classes of steroids in the USA •1 is Strong •7 is Weak (OTC)

Opiate Considerations

Tolerance •The need for a higher dose to receive same effect Dependence •Experience withdrawal symptoms with removal Addiction •Active drug seeking behavior with continued use despite potential harm Pseudo-addiction •Continuously returns for care; undertreatment

CC: "I can't stand it anymore! I have blood in my poop and feel very weak" HPI: 26-year-old female comes into the office complaining of hematochezia and an increase in stool frequency. States over the last 2 months had noticed an increase in stools to 5-6x daily and has BPBPR +abdominal pain relieved with defecation. Dx with ulcerative colitis x 2 years ago. Was treated acutely and has been able to control it with probiotics x 2 years. Notes increase in fatigue and weakness since symptoms started. Denies any vomiting, constipation, nausea. ■PMHx: UC for 2 years; HTN for 2 years ■MEDS: Probiotics and HCTZ 25 mg daily, compliant ■LABS: Hgb 10.9 g/dL; ■PE (pertinent positives): orthostatic hypotension

Ulcerative Colitis (UC) Select appropriate medication -Prednisone written to improve symptoms and place UC in remission. Pending reaction to medication might need another treatment such as anti-TNF therapy Select non-pharmacologic therapy/interventions -Discuss with patient regarding probiotics and eating well to decrease inflammation. What are the contraindications or side effects of this medication? -Follow up with patient within 2-4 weeks to determine if steroids need to be continued for 8 weeks. Use steroids for shortest amt of time possible due to side effects. IF no improvement - referral to GI to be assessed for anti-TNF or biosimilar treatment Notes: More dehydration since Hgb is good

A 27 yo African American male comes into your clinic. He notes he has a hx of allergies and asthma. He has been using his pump daily for the last 2 months and more recently has been having to use it 2x a week at night. He states that prior to recently he would only need his albuterol maybe 1x a week unless his allergies were acting up. He also uses Cetirizine (Zyrtec) 10mg daily as needed for allergies and has been taking it daily for the last 2 months. Pt was diagnosed with asthma at the age of 12. States he only has ever used albuterol. PMHx: Asthma and Allergies FHx: Mother - 56 HTN, DM Father - 56 DM MGM - 82 HTN MGP - 84 HTN, DM FGM - DM FGP - HTN SHx - Denies tobacco, vaping, alcohol and illicit drug usage MEDS: Cetirizine (Zyrtec) 10mg daily for allergies, Ventolin HFA 90mcg 2 puffs every 4-6 hours as needed for shortness of breath (compliant) ALLERGIES - Pollen Vitals - All within normal O2 98% RA Tx besides Advair?

Uncontrolled Asthma What do you need to counsel the pt on? Rinsing out their mouth after using the medication (increased risk for thrush) Use prior to brushing their teeth - then they will be rinsing out their mouth anyway Use the inhaler daily even if they are feeling better (that means it is working!) Continue to use albuterol - monitor albuterol usage: Goal is < 2x weekly

A 27 yo African American male comes into your clinic. He notes he has a hx of allergies and asthma. He has been using his pump daily for the last 2 months and more recently has been having to use it 2x a week at night. He states that prior to recently he would only need his albuterol maybe 1x a week unless his allergies were acting up. He also uses Cetirizine (Zyrtec) 10mg daily as needed for allergies and has been taking it daily for the last 2 months. Pt was diagnosed with asthma at the age of 12. States he only has ever used albuterol. PMHx: Asthma and Allergies FHx: Mother - 56 HTN, DM Father - 56 DM MGM - 82 HTN MGP - 84 HTN, DM FGM - DM FGP - HTN SHx - Denies tobacco, vaping, alcohol and illicit drug usage MEDS: Cetirizine (Zyrtec) 10mg daily for allergies, Ventolin HFA 90mcg 2 puffs every 4-6 hours as needed for shortness of breath (compliant) ALLERGIES - Pollen Vitals - All within normal O2 98% RA Dx? What to do?

Uncontrolled Asthma What do you want to do? Add a daily inhaled steroid (low dose ICS - singulair) Add daily Leukotriene Inhibitor PFTs pending when they were last completed Possible referral pending on pt's rxn with medication and control of asthma

Sarah, a 45 y/o Caucasian female, presents requesting to have her blood sugar tested - "it must be running low because I don't feel well." PMHx: T2DM x 6 months, HTN x 12 yrs, dyslipidemia x 12 yrs, morbid obesity x 20 yrs Meds: Metformin 1000 mg po BID with food, Lisinopril 20 mg po once a day, pravastatin 40 mg po once a day All: Penicillin - hives SHx: married, 2 teenagers, sales associate, quit smoking 10 yrs ago, moderate alcohol - 3 to 4 beer or wine a week ROS: Daily nocturia, polyuria, polydipsia. No nausea, vomiting or diarrhea, no signs or symptoms of hypoglycemia, no paresthesia's, weakness or tingling. No chest pain or dyspnea. She admits to a 10 lb weight gain over the past 6 months. PE: NAD, alert & oriented x 3, marked obesity, VSS BP 154/90mmHg LA sitting, P 90bpm, R 18bpm, T 98.0F Oral, Wt 265 lbs, BMI 36kg/m2 HEENT - PERRLA, EOMI, no retinal hemorrhages or exudates, nose and throat clear without exudates or lesions Neck - supple, no lymphadenopathy, no thyromegaly Lungs - CTA bilat CV - RRR, no murmurs, gallops or rubs ABD - soft, obese, nontender, normal BS EXT - pulses 2+ throughout, no edema Neuro - CN II-XII intact, DTR's 2+ throughout, feet with normal vibratory and light touch sensation (monofilament normal) Labs: Na - 138, K - 3.7, Cl - 103, CO2 - 31 BUN - 16, Creat - 0.9, CA - 9.4, Phos - 3.3 AST - 16, ALT - 19, Alk Phos - 62, T. Bili - 0.4 Random glucose - 243 (high) HgA1C - 10.0% (high); Previous HgA1C - 8.9 (6 months ago) Fasting lipids - Chol - 244 (high) LDL - 141 (high) HDL - 58 Trig - 225 (high) UA - 2+ glucose, neg ketones, 1+ protein Meds?

Uncontrolled Type 2 diabetes mellitus Currently patient is almost at the max dosage of metformin. Due to worsening glucose control - I would add a medication vs increasing her metformin Uncontrolled hypertension Pt is not at max dosage of lisinopril. Patient can increase current dosage, but combo medication is most likely a better choice **Adding medication, but not adding amount of tablets** Hyperlipidemia not at goal Pt is not at max dosage of pravastatin, however a more efficient medication might be necessary Morbid obesity As always, we can have lifestyle changes that will aid with glucose control. Wt loss surgery can be discussed as well Meds considered: Metformin 1000 mg twice daily Lisinopril 20 mg once daily pravastatin 40 mg once daily

Antidiuretic Hormone

Upregulates aquaporins in the kidneys Pure water reabsorption not Na+ (that would be Aldosterone) Cause thirst response Pain/stress can cause upregulation of ADH Inflammation, pulmonary disease also upregulate (ADH) SIADH (also meds can cause this)

A 24-year-old male comes to the clinic for back pain. Notes that he is a construction worker and last week was lifting heavy material. Has a constant dull pain (6/10) in his lower back that will "shoot" down his right leg (9/10) with twisting and bending. Patient has had this before and was given medication with relief. He has been taking OTC Tylenol "sometimes 2, sometimes 4 at a time" (unknown dose) and his friend's Percocet with mild relief. Lying flat improves the pain and any movement worsens the pain PMHx - none Vitals - WNL A few things worth mentioning with this case... What should we counsel patient about?

Using friend's medication - don't do it OTC Tylenol - is it extra strength? Is it regular? PLUS - Percocet - Oxycodone + Tylenol - He could be Overdosing on Tylenol daily What is the max daily dosage of tylenol for adults? 4 grams/24 hour Do we need to treat his pain? Of course! Does he need opioids first? No What is a better option? - NSAIDs (Work on peripheral tissues and CNS) - Muscle Relaxers: Carisoprodol (Soma) is an opiate - all others are not - Steroids if inflammation is a concern - Physical Therapy - Rest/Stretching - Xray --> MRI pending results Ok, but what if my patient was a 65-year-old patient with chronic dull/achy low back pain (5/10) that improves with rest and worsens with movement. Hx of DM2. - Ketorolac (Toradol) injection in office It would help due to mechanism of action What would I stay away from? -NSAIDs (Work on peripheral tissues and CNS) -Muscle Relaxers -Steroids -Physical Therapy -Rest/Stretching -Xray --> MRI pending results

CC: "My vagina itches" x 1 week HPI: 17 y/o F complains of itchy vaginal area x 1 week. Mild white discharge, no blood. Pt states she was at a gymnastics tournament prior to the symptoms. No abdominal or vaginal pain. Admits to being sexually active with one partner, uses condoms. •PMH: None •Surg Hx: None •Meds: None •Allergies: Azithromycin (Hives) •Soc Hx: Denies alcohol, drugs, or smoking •VS BP 110/74 HR 70 RR16 O2sat 99% RA Temp 98.6 •Speculum Exam Reveals: Picture to the right, cottage cheese like discharge •Bimanual: No CMT •Rest of the exam WNL •Labs •Wet prep: no clue cells, +budding yeast •Urine hCG: Neg •U/A: WNL How do you treat?

Vaginal Candidiasis Diflucan 150mg PO x 1 •SE: Headache, nausea, abdominal pain Terazol Vaginal Cream 0.4% (terazol 7) 1 app full intravaginally qhs x 7 days Terazol Vaginal Cream 0.8% (terazol 3) 1 app full intravaginally qhs x 3 days Miconazole 2% 1 app full intravaginally qhs x 7 days Miconazole 3% 1 app full intravaginally qhs x 3 days Miconazole 100mg suppository intravag qhs x 7 days Miconazole 400mg supp intravag qhs x 3 days Miconazole 1200mg sup qhs x 1 day •Contraindications - taking other azole •Adverse Reactions: HA •Safety/Monitoring: periodic liver function tests

CC: " I woke up this morning feeling very dizzy" HPI: 42 y/o F presents to your clinic with severe dizziness. Pt. is a server states she worked a long shift yesterday went to bed and when awoke this am and sat up to get out of bed "the room was spinning. I felt like I was drunk but I didn't even drink." Pt. denies any recent trauma or injury. Pt. states worsens with position and turning of her head. Pt. denies any headache or ear pain/fullness. Pt admits to history of seasonal allergies. Has been using OTC Claritin and Nasonex as recommended. Denies any allergy symptoms currently. Pt. states dizziness comes and goes. PMH: PCOS, Seasonal Allergies Surg Hx: umbilical hernia repair Meds: Metformin 500mg PO BID, Loratadine 10 mg PO daily prn, Mometasone Furoate 50mcg/spray intranasal daily prn Allergies: Bactrim/Sulfa (Rash) Soc Hx: Single, denies drinking alcohol, smoking, or illicit drugs. Vitals BP 122/88mmHg HR 87bpm RR18bpm O2sat 99% RA Temp 98.4F Oral PE reveals: - Pupils PERRLA, EOMI - Visual Acuity 20/20 OD,OS,OU with Rosenbaum card @3ft - CN II-XII grossly intact - Cerebellar function test intact - Dix-Hallpike reveals mild rotational nystagmus - MMSE Normal How to tx?

Vertigo (BPPV) Considerations: - Meclizine may cause tiredness in patient - Maneuvers and time is the best tx for BPPV - May need neuro referral pending on severity of vertigo - Benzos/Antiemetics not chosen due to side effects of sedation and pt is not c/o vomiting Contraindications: none Adverse Reactions: Long-term adaptation/vestibular loss take for the least amt of time possible Safety/Monitoring: none

A 47-year-old female comes to the office as a new patient to be assessed for her physical. Notes that she has fatigue sometimes, but overall is feeling well. Tries to eat vegetables and fruits, but states that she has a hard time with her diet. Hx of having gastric bypass 5 years ago. Has lost almost 200lbs post-surgery. Admits to hair loss. Takes a multivitamin, B12, Folate, Vit C, Calcium and Zinc daily. Denies any other concerns. Hx of BP, Hyperlipidemia and DM2 but states all her numbers have been normal since she lost weight. PMHx: DM2, HTN, Hyperlipidemia SocHx: Denies any alcohol, illicit drug, tobacco usage Vitals: BP 120/78mmHg, HR 78bpm, RR 15bpm, Temp 98.7F Oral, O2 99% RA Allergies: NKDA Medications: OTC Vitamins listed above (compliant) CBC - Hgb - LOW, MCV - High CMP LIPID TSH VIT B12 - LOW FOLATE TIBC SERUM FERRITIN What do you suggest?

Vit B12 injections - Why? - Due to absorption in the small intestines most likely of loss of the intrinsic factor from her gastric bypass IM B12 therapy - 1000 mcg IM daily for 7 days (or until levels return to normal), then resume maintenance dosing of monthly injections: Therefore repeat labs in 1 week, 1 month and if continued to be normal - does not need to have monthly labs. - Eat leafy greens

CC: "I want to quit smoking" HPI: A 25-year-old male presents to the family practice clinic because he wants to stop smoking. States he has smoked for 7 years, 2 packs per day, and has decided he wants to quit. Denies any issues with anxiety/depression currently. Notes that he has a hx of depression as a child and will still have some bad days but at this time denies any concerns. Denies any suicidal or homicidal ideation. Denies any hx of seizures. Hasn't tried anything otc to quit smoking. PE and labs are all normal. PMHx: Mild Depression as a child - no current treatment Social Hx: Tobacco use (14 pack years) no alcohol or illicit drug usage Surg Hx: tonsillectomy (child 7 y/o) Meds: Multivitamin daily Allergies: NKDA VS BP 120/78mmHg LA sitting HR 80bpm RR 14bpm Temp 98.6 F Oral What are the options?

Wellbutrin, Chantix, Nicotine Patches/Gum/Lozenges What should we take into consideration as well Due to hx of depression Wellbutrin might be a great option to the patient. It would not only aid with depression but would also aid with quitting smoking. Why written that way? So there are a few ways you can write it. I would recommend 90 with a refill due to cost and distribution of medication Secondarily- it allows the patient to return to the clinic within 4-6 weeks for reassessment on how it is working. Would we need to try something else? Do they truly need 12-week treatment? Sometimes patient only needs 7 weeks At visit, medication can then be adjusted - or patient already has their refill

CC: "My stomach hurts" An 18-year-old female presents to your ER complaining of a 1-week history of abdominal pain. She tells you that she and her friends recently returned from spring break in Cozumel. Upon returning, she has had a constant ache (7/10) on her right side. Pt notes mild nausea, no diarrhea/vomiting/constipation. Denies any dysuria, urinary frequency or urgency. Notes she has some discharge, but not "too much" The patient's mother is worried because her daughter has been unable to eat for the past 2 days and she feels she may have become sick from "drinking the water" while on vacation. Mom notes she has given her daughter pepto-bismol with no relief. ■PMHx - None ■Soc Hx - Denies alcohol/tobacco/illicit drug usage. Denies being sexually active. Patient is a student ■Medicines - Daily vitamin ■Allergies - NKDA ■Vital Signs: BP 104/68mmHg RA sitting, RR 10bpm, Pulse 74bpm, Temp 98.8F Oral LMP 2 weeks ago Cardiovascular and pulmonary exams are normal. The abdominal exam is significant for a soft, non-distended, but diffusely tender over the suprapubic area with light and deep palpation. Pain is greater RLQ vs LLQ without guarding or rebound. Bowel sounds are normal x 4 quadrants

What are some of your differentials? - Colitis, Appendicitis, PID, UTI, Ovarian Cyst What tests do you want to order? - CT scan? Ultrasound? Pelvic Exam? U/A C&S, Chlamydia/Gonorrhea, Trich? Any concerns? - Let's do a pelvic exam and get mom to leave the room for more questioning

Alfonso, a 68 y/o HM, presents complaining of "Blurry vision for the past few months that seems to be worsening." He also notes increased fatigue and a decreased energy level that has prevented him from working in the garden which he usually enjoys. PMHx: HTN x 18 yrs, dyslipidemia x 8 yrs, obesity x 25 yrs Meds: Lisinopril 20 mg po once a day Allergies: NKDA SHx: District Salesman, married x 40 yrs, 3 children, no smoking, 1 to 2 glasses of wine with lunch and dinner ROS: occasional polydipsia, polyphagia, fatigue, and weakness. Gets up once during the night to urinate Denies check pain, dyspnea, dizziness, numbness or tingling in extremities, weight changes, nausea, vomiting or diarrhea. No skin lesions or headaches. No other visual complaints. PE: NAD, alert & oriented x 3, central obesity, VSS B/P 124/76mmHg LA sitting, P 80bpm, R 18bpm, T 98.2F Oral, BMI 27.4 kg/m2 HEENT - PERRLA, EOMI's, no retinal hemorrhages or exudates, TM's pearly gray, nose and throat clear without exudates or lesions Neck - supple, no lymphadenopathy, no thyromegaly, no JVD Lungs - CTA bilat, CV - RRR, no murmurs, gallops or rubs Abd - soft, nontender, normal BS, no hepatosplenomegaly EXT - peripheral pulses 2+ throughout, no lesions, edema or lesions Neuro - CN II-XII intact, feet with normal vibratory and light touch sensation and monofilament-normal Na - 141, K - 4.0, Cl - 96, CO2 - 22 BUN - 20, Cr - 1.1, Ca - 9.9, Phos - 3.2 AST - 21, ALT - 15, Alk Phos - 45, T. bili - 0.9 Fasting glucose - 162 (high) HgA1C - 6.8% (high) Fasting lipids - Chol - 280 (high) LDL - 193 (high) HDL - 27 (low) Trig - 302 (high) UA - 1+ glucose, neg ketones, neg protein, neg microalbumin

What are the diagnoses? Type 2 Diabetes, Overweight, Hyperlipidemia w/ low HDL What medications should be considered? Metformin for glucose control as well as a statin for cholesterol control What other interventions should be considered? Pt should be counseled on lifestyle changes. Discuss with patient regarding dietary and exercise choices. Healthy lifestyle changes. Possible referral to dietician, nutritionist or even a diabetic center that can aid with healthy choices/support. Metformin (Glucophage, Glucophage XR) T2DM - Initial: 500 mg by mouth every 12 hours or 850 mg by mouth daily with meals; increase every 2 Wks Maintenance: 1500-2550 mg/day PO divided q8-12hr with meal Extended release - Glucophage XR: 500 mg by mouth once daily with dinner; titrate by 500 mg/day every week; not to exceed 2000 mg/day T2DM Prevention - 850 mg by mouth 1-2x daily Contraindications: CHF, metabolic acidosis, severe renal impairment (GFR <30), alcoholism Adverse Effects: GI (nausea, diarrhea, dyspepsia, constipation, vomiting) Interaction with CT contrast media - no metformin for 48 hrs after the study - may cause renal damage

Sarah, a 45 y/o Caucasian female, presents requesting to have her blood sugar tested - "it must be running low because I don't feel well." PMHx: T2DM x 6 months, HTN x 12 yrs, dyslipidemia x 12 yrs, morbid obesity x 20 yrs Meds: Metformin 1000 mg po BID with food, Lisinopril 20 mg po once a day, pravastatin 40 mg po once a day All: Penicillin - hives SHx: married, 2 teenagers, sales associate, quit smoking 10 yrs ago, moderate alcohol - 3 to 4 beer or wine a week ROS: Daily nocturia, polyuria, polydipsia. No nausea, vomiting or diarrhea, no signs or symptoms of hypoglycemia, no paresthesia's, weakness or tingling. No chest pain or dyspnea. She admits to a 10 lb weight gain over the past 6 months. PE: NAD, alert & oriented x 3, marked obesity, VSS BP 154/90mmHg LA sitting, P 90bpm, R 18bpm, T 98.0F Oral, Wt 265 lbs, BMI 36kg/m2 HEENT - PERRLA, EOMI, no retinal hemorrhages or exudates, nose and throat clear without exudates or lesions Neck - supple, no lymphadenopathy, no thyromegaly Lungs - CTA bilat CV - RRR, no murmurs, gallops or rubs ABD - soft, obese, nontender, normal BS EXT - pulses 2+ throughout, no edema Neuro - CN II-XII intact, DTR's 2+ throughout, feet with normal vibratory and light touch sensation (monofilament normal) Labs: Na - 138, K - 3.7, Cl - 103, CO2 - 31 BUN - 16, Creat - 0.9, CA - 9.4, Phos - 3.3 AST - 16, ALT - 19, Alk Phos - 62, T. Bili - 0.4 Random glucose - 243 (high) HgA1C - 10.0% (high); Previous HgA1C - 8.9 (6 months ago) Fasting lipids - Chol - 244 (high) LDL - 141 (high) HDL - 58 Trig - 225 (high) UA - 2+ glucose, neg ketones, 1+ protein What are the dx's?

What are the diagnoses? Uncontrolled Type 2 diabetes mellitus Uncontrolled hypertension Hyperlipidemia not at goal Morbid obesity What other therapies should be considered? Discuss with patient lifestyle changes. How is her diet? Is she exercising? Is she taking her mediation properly? Is she compliant taking it daily? What are the adverse effects of these medications? Pending on what dosages can be adjusted and added will determine the adverse effects of the medications. When do you need to see this patient back again? Patient to return in 2-4 weeks

CC - " My throat hurts" A 6-year-old girl presents to your clinic complaining of a sore throat and fever x 4 days. Fever has ranged between 100-102F Tympanic. Mom notes that she hasn't been eating well. Denies any cough, nasal congestion, headache, facial pain. She immigrated to the US from Russia with her parents about 4 months ago. She has not had regular medical care and her immunization status is unknown ■PMHx - unknown ■Allergies - None ■Medications - None ■Vitals - BP 102/68 mmHg RA sitting, HR-114bpm, RR-20 bpm, T-101.5°F Tympanic WT-20kg, HT - 46" ■Alert & anxious, appears ill ■HEENT: NC/AT, Conj-clear, PERRLA, Ears-wnl, Nose-patent, sinuses-nontender, pharynx-erythematous, tonsils 3+, gray membrane coating the tonsils, uvula and soft palate. ■NECK: FROM, supple, + large, tender lymph nodes throughout her cervical chain ■LUNGS: + tachypnea, CTA bilat ■CV: + tachycardia, RR w/o murmurs, gallops or rubs ■ABD: Normal bowel sounds x 4, soft, nontender, nondistended, no hepatosplenomegaly. ■SKIN: No rashes

What is the differential for this patient? - Tonsillitis, Diphtheria, Strep Throat What is the most likely diagnosis? - Diphtheria (Corynebacterium diphtheria) What studies should be ordered? - Bronchoscopy for removal and culture of What other care does this patient need? Diphtheria antitoxin (DAT) Erythromycin (better eradication of bacteria vs PCN) Tracheotomy if needed 30-50mg/kg/day every 6-8 hours Max 4g/day Pt weighs 20kg - 20*50 = 1000mg or 1g per day. Due to infection and severity, higher end of dosing was chosen Erythromycin comes in 250mg/5mL dosage 1000mg/4 = 250mg - Therefore, patient should be given 4 dosages of 5mL to get the daily dosage recommended ■Diphtheria ■Corynebacterium diphtheria ■Diphtheria antitoxin (DAT) ■PCN or erythromycin ■Brochoscopy to remove membrane ■Tracheotomy as needed

CC - "I have a headache" A 19-year-old male presents at the emergency room complaining of a severe headache (9/10), vomiting, and a stiff neck with pain running down his back. Pt is a football player at UF. He notes he was at a party this past weekend with his girlfriend and other cheerleaders. Pt states that he has a rash that just started within the last day or 2. During triage, his temperature is 102°F. The young man appears to have trouble hearing during the nurse's interview and seems to have trouble concentrating. He reports that he thought he was getting a cold the past few days. ■PMHx - Unremarkable ■Medictions - None ■Allergies - None ■Social Hx - He denies any illicit drug use, drinking alcohol, or smoking. ■Vitals -BP 132/78mmHg RA lying flat, HR 110bpm, RR 16bpm, Temp 102F Oral ■On physical exam, the skin exam reveals several areas of small purplish spots over his back, chest, abdomen, thighs and arms. ■Due to rash and other symptoms lumbar puncture ordered...

What is the most likely diagnosis? - Bacterial meningitis Which microorganism is the causative agent of this? - Neisseria meningitides How would you treat this patient? - Antibiotics ■Cefotaxime 2gm IV every 4-6 hours OR Ceftriaxone 2gm IV every 12 hours PLUS ■Dexamethasone 0.15mg/kg IV every 6 hours for 2-4 days -Give with or just before first dose of antibiotic ■Vancomycin 15-20mg/kg IV every 8-12 hours What will happen if he is not treated? - Death, Impaired mental status, Increased intracranial pressure and cerebral edema, Seizures, Focal neurologic deficits (eg, cranial nerve palsy, hemiparesis), Cerebrovascular abnormalities, Sensorineural hearing loss Intellectual impairment What other bacteria can cause this disease? - Streptococcus pneumoniae patients over 50 years of age or those who have deficiencies in cell-mediated immunity, Listeria monocytogenes

CC - "It hurts when I pee" A 24-year-old woman presents to your clinic complaining of dysuria and frequency x 3 days. She complains of mild lower abdominal pain and an abnormal odor to urine. Denies any visible blood in urine. No fever, chills, nausea, vomiting, or diarrhea. No back or flank pain. No vaginal discharge or discomfort. No rashes. She is sexually active and uses oral contraception. She has never been pregnant. ■NAD, alert, VSS ■LUNGS: CTA bilat ■CV: RRR w/o murmurs, gallops or rubs ■ABD: Normal bowel sounds x 4, soft, nondistended, + mild suprapubic tenderness. No CVA tenderness. No guarding, rebound or rigidity ■GYN: Deferred while awaiting UA results ■UA: Nitrite +, LE +, pH-6.0, WBCs - 20-25 per hpf, 3+ bacteria

What is the most likely diagnosis? - Cystitis, Urinary Tract Infection Which microbe is most likely? - E. coli What other microbes may be implicated? - Proteus mirabilis, Klebsiella pneumonia What is the best treatment for this patient? - Bactrim DS What could happen if she does not receive treatment? - Cystitis, Urinary Tract Infection, pyelo

CC - "I have vomiting and diarrhea" A 17-year-old male presents to the ER in Florida on the fourth of July complaining of acute onset nausea, vomiting, and diarrhea shortly after returning home from a pool party at his friend's house. Pt note he ate hamburgers, hotdogs, potato salad with baked beans and lemonade. States that the food was poolside and he was there for about 6 hours. Denies anything tasting spoiled. States has seen via social media 3 other guests are also sick. Notes that his symptoms started right before the fireworks (~4 hours after eating). Denies any fevers, hematochezia, melena. Admits to a crampy pain "all over" his abdomen (7/10) that improves with diarrhea or vomiting (3/10). Pt admits since starting symptoms has vomited x 4 and had diarrhea x 5. States he is unable to keep anything down. ■Pt has vomited x 1 since arriving to the ER ■PMHx - None ■Medications - None ■Allergies - None ■Vitals - 110/70mmHg RA lying flat, RR 16bpm, HR 108bpm, Temp 99F Oral ■NAD, A&O x 3, VSS, actively vomiting ■HEENT: NC/AT, PERRLA, conj-clear, TM's-pearly gray, nose-patent, pharynx-wnl ■NECK: FROM, supple, No lymph nodes ■LUNGS: CTA bilat ■CV: RRR w/o murmurs, gallops or rubs ■ABD: Normal bowel sounds x 4, soft, nondistended, + mild diffuse tenderness throughout; no guarding, rebound or rigidity ■SKIN: No rashes

What is the most likely diagnosis? - Food Poisoning Which organism is implicated, and what does it look like on gram stain? - Staph Aureus - gram positive cocci What are the tx options? - BRAT diet, Increase fluids, pt will eat when he is hungry - because no blood, pt can use immodium if needed Name other organisms that can cause similar disease after eating contaminated food (albeit bloody diarrhea) - Campylobacter jejuni, Vibrio cholera, Vibrio parahaemolyticus, Vibrio vulnificus, E. coli variants, Salmonlla typhi, Shigella dysenterea, Clostridium perfringens

CC - "I have a rash" A 53-year-old male farmer presents to your clinic complaining of a rash on his arm x 7 days. States it started as "itchy red bumps" that blistered and then ruptured. States has felt "feverish" but denies checking for a temperature. Denies any cough, congestion, sore throat, nausea, vomiting diarrhea. Denies any new lotions, soaps, detergents. Denies any recent contact with anyone. Denies having chicken pox as a child. Is not under any excessive stress. Pt admits to running a livestock farm. Currently has cows, horses, goats, sheep and chickens. ■PMHx - unremarkable ■Medications - None ■Allergies - None ■Social Hx - pt is a farmer. Denies any smoking, drinking or illicit drug usage ■Vitals - BP 130/82mmHg LA sitting, HR 78bpm, RR 16bpm, Temp 98.6F Oral ■NAD, A&O x 3, VSS ■SKIN: Clear with exception of R upper arm - 4.5 cm circular black hardened plaque surrounded by several blisters (vesicles) and edema. ■NODES: + tender axillary lymph node enlargement on R ■HEENT: WNL ■LUNGS: CTA bilat ■CV: RRR w/o murmurs, gallops or rubs

What is the name of this patient's disease? - Cutaneous anthrax What is the causative organism of this disease? - Bacillus anthracis What does it look like on gram stain? - Large Gram Positive Rod What other diseases does this organism cause? - Pulmonary, GI or oropharynx anthrax How would you treat this patient? - Ciprofloxacin

CC: "I feel depressed x 1 month" HPI: 26-year-old female presents to your office stating she has been feeling depressed for the last month. Notes that she is feeling sad and hopeless. Pt isn't sleeping at all (3 hours a night) and is constantly fatigued. States that depression started after having her baby 2 months ago but over the last month has increased significantly. Pt is breastfeeding and is up all night with her child who feeds every 3 hours. Denies any hx of depression/anxiety. Is eating well. Denies any suicidal ideation, denies any thoughts of harming her child or anyone else. Concerned because she feels like she should enjoy being a mom, but states that she is having a hard time adjusting/bonding with her baby and just wants to stay in bed. PMHx: none - healthy Surg Hx: C-section (child 2months old) Meds: Multivitamins daily Allergies: NKDA Social Hx: Married for 2 years, 1 child (boy 2 months old) single family home, safe, no abuse, no tobacco, illegal drugs or alcohol VS BP 102/72mmHg RA sitting HR 68bpm RR 14bpm Temp 98.4F Oral PE: Pt crying during exam ENT - Eyes, conjunctiva pink w/o lesions; Thyroid, non-enlarged, w/o nodules or tenderness CV - S1 & S2 RRR w/o murmurs, rubs, gallops RESP - CTA BL Labs: Electrolytes, CBC, TSH/FT4 - NL

What is your diagnosis? Post-Partum Depression What? If any, medication would you start the patient on? Sertraline (Zoloft) Why? It is safe for breastfeeding What would you take into consideration when writing medication? Pt is breastfeeding. We would want something that is safe for baby. What else would you do for patient? Refer to counseling

A 68 yo female presents to your clinic for 1 month follow up after starting Lantus stating her blood sugar isn't as good as she wants it to be. She currently is taking 40 units of Lantus at night. BG in the am averages 180-200 and 220-250 after meals. Patient is not exercising and isn't really monitoring what she eats. Last HgbA1c 1 month ago was 8.8%. Intake vitals shows: T: 98.7F oral P: 86bpm BP: 112/78mmHg L arm Wt: 86kg Ht 5'5" MI 31.5kg/m2 PE cont: GEN: ANOx3, sitting on exam table CV: S1 S2, no MGR Pulm: CTA B/L GI: normoactive BS Skin: Acanthosis Nigricans noted on nape of neck You review labs: HgbA1c - 8.8% FBS - 182 All other labs within normal limits

What is your diagnosis? Uncontrolled DM2 What are your steps in treatment? Increase Lantus and add a rapid acting insulin with meals Let's talk insulin and cost Humalog and Novolog are comparable ◦Humalog $68 per vial ◦Novolog $71 per vial ◦Apidra $217 per vial

CC: "F/U for depression" HPI: A 29-year-old male presents to the family practice clinic for his 2-week follow-up after starting his depression medication. At his last appt, he was diagnosed as moderately depressed and Fluoxetine 20mg was prescribed. He notes that he has taken it daily as directed. Pt denies any improvement with his insomnia, anhedonia, loss of appetite and decreased interest in going to work. He states there has been no change. His boyfriend who is accompanying him notes that he hasn't noted a change either. Denies any suicidal or homicidal ideation. He is tolerating the medicine well. PMHx: none - healthy Surg Hx: circumcision at birth Meds: Multivitamins daily, Prozac 20 mg daily Allergies: NKDA Social Hx: Feels safe, no abuse, no tobacco/alcohol/illegal drug use VS BP 124/80mmHg LA sitting, HR 80bpm, RR 12bpm, Temp 98.6F Oral

What is your next step? Increase dosage of his fluoxetine and have him return in 4 weeks. Advised patient it can take at least 2-4 weeks to note a change in mood. Make partner aware to monitor for changes as well. If no improvement at next office visit, at that time discuss possibly changing medication. As always, counsel the patient about the risk of suicidal thoughts occurring or depression worsening and at that time he would be able to contact the office, call 911 or go to the ER. What else would you do for patient? Make sure that patient is following up with a counselor. If he isn't, refer and get him the information of one.

CC: "F/U for depression" HPI: 35-year-old female presents to the Family Practice Clinic for her follow-up appointment for depression. States 14 weeks ago was prescribed Fluoxetine 20mg and over the course of 14 weeks has been increased to 80mg. States that she has slight improvement. Notes that she hasn't been crying as much and her family/friends tell her she seems happier, but she isn't noticing much of a difference. States she is still having difficulty going to work as is still having difficulty sleeping. Denies any suicidal or homicidal ideation and notes that she is tolerating the medication well. PMHx: None - healthy Surg Hx: tonsillectomy (child 7 y/o) Meds: Multivitamin daily, Prozac 80 mg daily Allergies: NKDA VS BP 128/78mmHg RA sitting, HR 82bpm, RR 14bpm, Temp 98.6F Oral

What is your next step? Pt is currently at the highest dosage of Fluoxetine. Therefore, I would switch her to another anti-depressant vs adding another medication due to her wishes to remain on as "little" as possible. Duloxetine (Cymbalta) is a great choice -SNRI. Due to the half life of Fluoxetine and the ability to stay in the system for up to 5 weeks, Cymbalta will be started at a low dosage. Due to the severity of depression, it is not feasible to wean patient off current SSRI prior to starting SNRI What else would you do for patient? I would verify and make sure that patient is seeing counseling and if not, refer her to a counselor. Make a follow up appt with patient to follow up on the efficacy of the medication.

CC - "I'm here for a work physical" A 22-year-old male is a first year PA student and presents to your clinic for a routine physical and PPD screening. Pt denies any complaints. Denies any recent travel, night sweats, fever or cough for more then 2 weeks. Pt was born in the US and did not received BCG vaccine as a child. ■PMHx - None ■Allergies - None ■Medications - None ■Vitals: BP 112/64mmHg RA sitting, HR 70bpm, RR 10bpm, Temp 98.4F Oral Wt: 85kg Ht: 70inches BMI 26 kg/m2 ■You correctly administer his PPD injection, and the patient returns 48 hours later with a 15mm area of induration at the injection site. You repeat his entire history, including his social history, and he states he forgot to tell you before that he occasionally volunteers at a homeless shelter. ■You order a chest x-ray. It returns with no significant radiographic findings.

What test would you order next (after PPD)? - CXR What is this patient's dx? - Latent TB What is the indicated tx? - 3 options **Isoniazid alone x 9 months **Isoniazid and Rifampin **Isoniazid and Rifapentine What will happen if he does not receive or finish tx? - Latent Tuberculosis can become an active infection. If treatment is not completed, resistance can be created to the treatment. ■Talk to the patient regarding cost as well as compliance due to length of treatment ■Pt is a PA student, so let's say he is SUPER compliant - then maybe pick the most cost affordable ■Let's say patient tells you that he hates taking medication, can barely remember and has awesome insurance or he doesn't care because he will just use his student loans to cover it. Well, then maybe pick the shortest therapy ■First - what dosages do our medication come in? -Isoniazid = 150mg and 300mg tablets -Rifampin - 300mg capsules -Rifapentine - 150mg capsules ■Second - what dosage is needed? -Isoniazid (300mg daily x 3 or 9 months or 900mg weekly x 3 months) ■If daily x 9 months, for better adherence write 90day rx with 2 refills = 9 months a total of #270 tablets ■If daily x 3 months, then 300mg max and only #90 needed ■If weekly x 3 months - 900mg max. Therefore, needing to take 3 tabs (3*300mg = 900mg) weekly x 12 weeks = 3 tabs *12 weeks = 36 tablets -Rifampin 300mg capsules x 3 months ■600mg daily needed. Therefore, 300mg x 2 capsules = 600mg ■2 capsules x 90 days = 180 capsules -Rifapentine 150mg capsules weekly x 3 months ■900mg daily needed. Therefore 150mg x 6 capsules = 900mg ■6 capsules x 12 weeks = 72 capsules

A 34 y/o 80 kg male presents to the ER with fever and vomiting/diarrhea secondary to gastroenteritis. He appears dehydrated and is unable to tolerate oral fluid replacement. You decide to admit him for rehydration. •HR 120 bpm, BP 95/68 mmHg, T 39C What fluids should we use? •LR, NS, 1/2NSD5W? How much should we use? •A little? A lot?

What type of fluid bolus should he receive? NS How much should he receive? 1 L to start or 20mL/kg (for this pt it would be 1600 mL or 1.6L) - Biggest bag size fluids come in are 1L

A 34 year old 80 kg male presents to the ER with fever and vomiting/diarrhea secondary to gastroenteritis. He appears dehydrated and is unable to tolerate oral fluid replacement. You decide to admit him for rehydration. •HR 120 bpm, BP 95/68 mmHg, T 39C What type of fluid bolus should he receive? How much should he receive?

What type of fluid bolus should he receive? NS, with KCI (10 mEq) •D5 Ns with 20 mEq/L KCl@ 125mL/hr IV •Bolus: 1L NS IV once over 30 min or 1L NS IV bolus once How much should he receive? Maintenance rate can use 4/2/1 rule 40 first 10 20 next to 10 1 for every one left 125 mL/hr

CC - "I have a sore throat" HPI: A 5-year old male child presents to your walk-in clinic with his mother. He has had a fever (102F tympanic) at home and has been complaining of a sore throat for about 24 hours. The mother states he has been very cranky and will not eat any food she gives him. She has been making him drink Pedialyte and notes she gave him Tylenol last night and fever improved to 100F tympanic. ■PMHx None ■Meds None ■Allergies NKDA ■Vitals BP 100/68mmHg LA sitting HR 120bpm RR 30bpm Temp 103F Oral, WT 20kg, HT 40inches ■On exam, the boy has considerable swelling and redness of the pharynx. His tonsils are enlarged and coated with a white exudate. He has swollen anterior cervical lymph nodes bilaterally. His ears are clear. His lungs are clear. His skin is warm and dry, but he has a fine, papular rash on his trunk.

What would be your presumptive diagnosis for this child? Why? Scarlet Fever (Beta Strep Group A) Dx due to symptoms and findings upon exam (skin rash + sore throat) What diagnostic testing would be indicated, if any? - Rapid Strep with culture and sensitivity What is the best treatment for this illness? - Penicillin V What will happen if this child is not treated? - At risk for Rheumatic Fever ■AHA recommendations -<27kg - 250mg 2-3x daily x 10 days ->27kg - 500mg 2-3x daily x 10 days - ■Special Considerations... -The child is 5yo so we want liquid -Liquid is 250mg per 5mL -5mL x 3x daily = 250mg x 3x daily- So, 5mL x 3 = 15mL daily 15mL x 10 days = 150mL total dispensed Notes: make sure to ask when they took Tylenol last

1 year old 12 kg child presents to the ED presents after suffering an anaphylactic reaction secondary to peanut exposure. •HR 160 bpm, BP 73/42, RR 40 bpm What's the first step to manage this pt? Meds? Fluids?

What's the first step to manage this patient? epi. 0.12 mg IM once Epi cardiac arrest dose 1 g Meds? Benadryl 1mg/kg (12 mg IV once) steroids methylprednisolone 1mg/kg (12 mg IV once) Fluids? (they are hypotensive and tachy) •Bolus : 240 mL NS •Maintenance: 44 ml/hr NS

CC - "spider bite" HPI - A 21-year-old male presents to your urgent care with a spider bite x 3 days in the left groin. Pt did not see the spider bite him, but states he has some "big ones" around his apt. States that it occurred 5 days ago, but over the last 3 days has worsened in size, redness and pain. Pain is constant pressure and was a 3/10 and is now a 9/10. Vicodin makes the pain better (4/10). His cousin supplied him with 1 tablet and pt notes "it'd be cool if you'd write me some of those" Pt notes overall feeling fatigued and nauseas. States he has had a fever x 2 days but has not checked his temperature. PMHx None Meds: None Allergies: NKDA Soc Hx: Single, student, No tobacco/illicit drugs, 2-4 beers on the weekends VS: BP 124/76mmHg sitting RA HR76bpm RR16bpm O2Sat 100% RA Temp 98.4F PE: The patient has a fluctuant, erythematous 2 x 4cm abscess in the left groin that is oozing a scant amount of yellowish-white pus. No surrounding cellulitis. He has enlarged inguinal lymph nodes on the left. The patient's skin is otherwise intact and warm and dry.

Which microorganisms could have caused this infection? - Staph Aureus (MRSA & MSSA) Which one is most likely responsible? -MRSA Which antibiotic(s) would you use to treat this patient? Why? - Bactrim DS - coverage for MRSA and pt is uncomplicated What other care does this patient need? - I&D of area What other prescriptions does he need? - Pain What kind of patient counseling needs to done? - Keeping area clean while healing

CC - "I have anxiety" A 23-year-old female presents to your office with new onset anxiety and panic. Notes that she is currently having issues at home and that she overall is feeling overwhelmed. States that she will have "attacks" where she feels as though her heart is racing and she can't breathe. Denies any chest pain. Pt notes that symptoms worsen when she has a fight with her significant other, when she is driving in traffic, when she is in large crowds. Symptoms started years ago and have worsened as she has gotten older. Currently notes having "attacks" at least 3x per week. Prior only had them maybe 3x per year. Currently is doing yoga and breathing to aid with her symptoms with minimal relief. Denies any depression or fatigue. Is currently being treated for depression with sertraline and states it is working really well. Denies any suicidal ideation or homicidal ideation. PMHx - Depression Medications - Sertraline 50mg daily, compliant Allergies - None Vitals - BP 110/72mmHg sitting LA, HR 72bpm, RR 16bpm, TEMP 98.2F oral

Would you want to write an Rx? Yes If so, what would you write? I would recommend Buspirone as well as Vistaril as needed. Patient is already on Sertraline which can aid with anxiety, so due to increase in attacks and current symptoms buspirone to be added. Any other treatments? I would recommend a referral to counseling or psychology/psychiatry for assessment due to increased amount of "attacks" Follow up? Bring patient back to office in 2-4 weeks to reassess anxiety and usage of medication. Adjust dosages as needed at that time. Follow up on if she has been seen by specialist Notes: Others Xanax, BB, Benzo

A 56 yo male with Type II DM presents to the ER after waking up in the morning to find that he had a swollen, red, painful big toe of his left foot. He had recently been on a cruise to the Bahamas and spent much of the time eating and drinking beer more than usual. An X-ray of his left foot shows no acute fracture. Vital signs are within normal limits. Blood work shows an increase in uric acid, but all other labs are otherwise unremarkable. The patient states he cannot tolerate NSAIDs due to a history of GI bleeding. NKDA. Once Acute Attack Controlled Pt is now following up with PCP Uric acid labs completed, & levels are at 8 mg/dL What to do next?

Write for allopurinol and continue to recheck levels every 1-2 weeks until level below 6mg/dL. Then maintenance dosage will be written

Good Prescribing Practice: Directions for use

Write out in full English rather than use Latin abbreviations •Latin abbreviations - more convenient, more potential for mistakes •Avoid Dangerous Abbreviations (see next slides) Provide clear and specific directions •Avoid "Take as Directed."

A 57-year-old Caucasian Female comes into the office. She was recently diagnosed with a DVT within the last 2 weeks. Patient went on vacation overseas and was on a 22hour flight. After returning home noted pain, swelling and redness in her right leg. Patient was seen in the ER after returning home, stayed for 3 days, stabilized and had Lovenox in the hospital. Discharged on 5mg daily of warfarin (Coumadin). Overall, patient is feeling well. Notes the leg swelling/pain has improved and denies any shortness of breath. She was released from the hospital 1 week ago and has come to the office for an INR check. Allergies: None Medications: Coumadin 5mg daily (Compliant), Lisinopril 10mg daily (Compliant), Levothyroxine 75mcg daily (Compliant) PMHx: Hypothyroid, HTN, DVT SocHx: Denies any tobacco, alcohol, illicit drug usage Vitals: BP 114/72mmHg RA Sitting, Pulse 72bpm, RR 14bpm, O2 99% RA, Temp 98.4F Oral LMP: 2 years ago (menopause) Physical Exam All within normal. Right leg, no swelling, pain, tenderness, erythema. Neg Homan's sign What do you do?

You discuss with her the 4D's. Diet: Any major changes in the diet? Vit K rich foods? Drugs: Any new medicines? OTC meds? Ibuprofen? Motrin? ASA? Dose: Has she taken medication daily? Any missed dosages? Disease: Any recent illness, fever, N/V/D, significant pain or stress? Goal is between 2-3 because of Dx DVT. So, how do we adjust? Basic rule is to increase by 1 daily dosage over the course of the week Therefore --> Increase by 5mg over the week. So, pick 2 days and have them take 1.5 tablets or 7.5mg - Mon-Fri 5mg @ 5pm and Sat/Sun 7.5mg @ 5pm But...if pt was a 37-year-old female, what do you need to do prior to starting on coumadin? - Pregnancy test Notes: Goal for mechincal heart valve 2.5-3.5 Low INR = blood too thick


Ensembles d'études connexes

Consumer Behavior Chp 16 (Exam 2)

View Set

Abnormal Psych. CH.9 Eating Disorders

View Set

Network+ Guide to Networks, Exam 3 (ch 9-12) study guide, CIT 45

View Set

NURS 204 - Exam 2 - Chapters 14, 15, 16

View Set

Chapter 6 - Nursing Care of Mother and Infant During Labor and Birth

View Set

Digital Marketing 4650: Google Ads

View Set